What intuitively obvious mathematical statements are false?

๐ŸŽ™๏ธ horsefeathers1123 ยท 1129 points ยท Posted at 01:20:53 on November 21, 2015 ยท (Permalink)

Saved comment

Lopsidation ยท 1168 points ยท Posted at 02:36:28 on November 21, 2015 ยท (Permalink)

If a girl called Eve listens to absolutely everything you and your friend say to each other, then you can't tell each other secrets without Eve finding out too.

anonymousproxy404 ยท 530 points ยท Posted at 02:53:54 on November 21, 2015 ยท (Permalink)

How is this untrue?

UlyssesSKrunk ยท 5813 points ยท Posted at 03:45:28 on November 21, 2015 ยท (Permalink)*

Take your message, treat it as a number and multiply it by a bunch of primes.

Send it to me. I will then multiply by a bunch of primes too.

I send it back to you. You then divide by all of your primes.

Send it back to me. I divide by all of my primes and get the original message.

It may be easier to think of the message as a box and the primes as locks.

You want to send a box to me without Eve getting at what's inside. So you put a lock on it and send it to me.

Now neither Eve nor I can open it because it's locked. I add my own lock because fuck you and your stupid lock. I send it back to you.

Now you can't open it and it's locked so it's worthless, therefor you take your precious lock back and send the now worthless piece of shit back to me.

Eve is still like "WTF?" All she has seen so far is the same box going back and forth with locks she can't open.

So now I get the box with my lock on it and I take my lock off. Now the box is unlocked and I can take your shit.

Iudicious ยท 2327 points ยท Posted at 04:30:17 on November 21, 2015 ยท (Permalink)

Your description of cryptography just made my night.

eaglejdc117 ยท 921 points ยท Posted at 16:08:56 on November 21, 2015 ยท (Permalink)

It's a great analogy. If you'd like to see more like this, check out The Code Book, by Simon Singh. In fact, he uses this very analogy in his public key chapter.

It's an absolutely fantastic read. I can't keep my hands on it- I keep giving my copy away to share it with people, then buying a new one.

Imapseudonorm ยท 952 points ยท Posted at 20:34:17 on November 21, 2015 ยท (Permalink)

That book quite literally saved my life. I was at a real low point in my life, and wanted to write a suicide note that was hard to figure out, but not TOO hard (yeah, I was a dramatic little fuck), so I started reading up on how cryptography worked throughout the ages.

Got so engrossed in the book I decided to learn even more about modern crypto. I spent the next few months reading everything I could about crypto and number theory, and by the time I emerged, I wasn't suicidal anymore.

shut-up-dana ยท 374 points ยท Posted at 21:17:25 on November 21, 2015 ยท (Permalink)

You should tell this to Simon Singh.

OfMugsAndDoughnuts ยท 101 points ยท Posted at 01:26:49 on November 22, 2015 ยท (Permalink)

Please do that is an amazing story. Also Simon Singh's a really nice guy (he gave a talk at my school)

StripeyC ยท 11 points ยท Posted at 08:59:36 on November 22, 2015 ยท (Permalink)

Same here, I've got a signed copy of the book that day from him.

a3wagner ยท 4 points ยท Posted at 06:04:21 on November 23, 2015 ยท (Permalink)

I saw a poster at my school that said he was going to give a talk, and I got really excited. Even better, I hadn't already missed the date -- it was going to be the following week!

Imagine my disappointment when I learned it was being given at a completely different university. Not even the same country. WHY DO WE EVEN HAVE THAT POSTER.

RobbieGee ยท 11 points ยท Posted at 13:41:02 on November 22, 2015 ยท (Permalink)

I found his webpage and sent him a link to this thread :)

Imapseudonorm ยท 2 points ยท Posted at 19:42:14 on November 22, 2015 ยท (Permalink)

Awesome. I've loved all of his books, and if it helps him to know how much one of his books helped someone, I'm all for it. Thanks for doing the legwork!

kriskingle ยท 55 points ยท Posted at 21:23:52 on November 21, 2015 ยท (Permalink)

That story is a bit similar to another story in another book by Simon Singh, The Fermat enigma. Paul Wolfskehl, an Austrian industrialist, was depressed over a love affair and ready to commit suicide at midnight, and to pass the time until then, began working on solving Fermat's last theorem. He didn't manage to solve it, but became so excited at identifying a way to a possible solution that he gave up his suicide attempt and established the Wolfskehl Prize, to be awarded to the person who proved the theorem.

[deleted] ยท 3 points ยท Posted at 23:08:15 on November 21, 2015 ยท (Permalink)

So was the dude a millionaire?

kriskingle ยท 4 points ยท Posted at 03:07:35 on November 22, 2015 ยท (Permalink)

He was, and he was quite successful too, if he was to endow a prize of some considerable value.

bryster126 ยท 82 points ยท Posted at 21:07:05 on November 21, 2015 ยท (Permalink)*

Check out computerphile on youtube

edit: https://www.youtube.com/user/Computerphile

Imapseudonorm ยท 20 points ยท Posted at 21:09:59 on November 21, 2015 ยท (Permalink)

Will do, thanks!

Zahand ยท 136 points ยท Posted at 22:09:31 on November 21, 2015 ยท (Permalink)

Other cool youtube channels:

Math/Numbers: Numberphile
Physics: Veritasium/Sixty Symbols
General knowledge: VSauce, CGPGrey
Programming: Derek Banas

Those are some of my favorite youtube channels :)

Plecks ยท 12 points ยท Posted at 04:39:04 on November 22, 2015 ยท (Permalink)

I'd also recommend:

Smarter Every Day (Physics)
Periodic Videos(Chemistry)
Engineer Guy (Engineering)
The Brain Scoop (Biology and Zoology)

metaStatic ยท 31 points ยท Posted at 22:16:54 on November 21, 2015 ยท (Permalink)

also Vihart

[deleted] ยท 12 points ยท Posted at 22:48:47 on November 21, 2015 ยท (Permalink)*

[deleted]

_N_O_P_E_ ยท 5 points ยท Posted at 02:44:56 on November 22, 2015 ยท (Permalink)

You mean Dirk?

[deleted] ยท 1 points ยท Posted at 11:38:06 on November 22, 2015 ยท (Permalink)

Who, Duhrk?

orangemaen ยท 3 points ยท Posted at 23:02:27 on November 21, 2015 ยท (Permalink)

Add in Crash Course for history and astrology as well.

kataanglover1 ยท 6 points ยท Posted at 23:36:40 on November 21, 2015 ยท (Permalink)

I think you meant astronomy. Astrology is quackery.

Don't want you getting torn apart by neckbeards. Common mistake. All the best.

orangemaen ยท 4 points ยท Posted at 00:06:05 on November 22, 2015 ยท (Permalink)

Spoken like a true leo.

kataanglover1 ยท 1 points ยท Posted at 01:13:15 on November 22, 2015 ยท (Permalink)

Almost! Virgo. Good try though.

THANKFUL_DUDE ยท 1 points ยท Posted at 22:29:16 on November 21, 2015 ยท (Permalink)

Pbs space time is the best YouTube channel if you want to be blown away

socialisthippie ยท 1 points ยท Posted at 23:20:03 on November 21, 2015 ยท (Permalink)

Another few:

Product Engineering and funny Mechanical/Machine shop stuff (through teardowns): AvE

Clock building: Clickspring

[deleted] ยท 1 points ยท Posted at 14:36:19 on November 22, 2015 ยท (Permalink)

Clickspring!

Imapseudonorm ยท 1 points ยท Posted at 23:31:21 on November 21, 2015 ยท (Permalink)

Less serious, but still awesome, have you seen vi hart's stuff?

I especially like this one: https://www.youtube.com/watch?v=4mdEsouIXGM

NagNella ยท 1 points ยท Posted at 01:24:34 on November 22, 2015 ยท (Permalink)

Great list

hand0fkarma ยท 1 points ยท Posted at 03:17:39 on November 22, 2015 ยท (Permalink)

Sweet list

californication101 ยท 1 points ยท Posted at 03:52:35 on November 22, 2015 ยท (Permalink)

Programming: Derek Banas Thanks, these are some awesome channels. Any others you can suggest, science, math, cryptology?

pete101011 ยท 1 points ยท Posted at 08:05:47 on November 22, 2015 ยท (Permalink)

What about SmarterEveryDay?

Zahand ยท 1 points ยท Posted at 10:08:26 on November 22, 2015 ยท (Permalink)

shit, forgot about Destin

Pilesos ยท 1 points ยท Posted at 12:40:52 on November 22, 2015 ยท (Permalink)

Wow these channels are awesome. Thank you very much!

[deleted] ยท 1 points ยท Posted at 13:10:26 on November 22, 2015 ยท (Permalink)

My procrastinating side thanks you deeply.

Otroletravaladna ยท 1 points ยท Posted at 11:25:08 on November 24, 2015 ยท (Permalink)

Chemistry: Periodic Table of Videos

[deleted] ยท 1 points ยท Posted at 22:06:44 on November 21, 2015 ยท (Permalink)

Thanks for this channel!

bryster126 ยท 2 points ยท Posted at 22:33:16 on November 21, 2015 ยท (Permalink)

No problem!

Friskyinthenight ยท 1 points ยท Posted at 23:40:35 on November 21, 2015 ยท (Permalink)

Just looked him up again on wikipedia, glad to see that that chiropractic garbage has been sorted now. Poor guy.

pled ยท 19 points ยท Posted at 21:29:31 on November 21, 2015 ยท (Permalink)

There's always more cool shit you haven't heard about before. That's exactly why suicide is always the wrong answer.

I'm glad you're better!

Max_Insanity ยท 8 points ยท Posted at 13:27:48 on November 22, 2015 ยท (Permalink)

What is the act of killing one self called?

Hope you never get that question on a quiz show.

Muchashca ยท 11 points ยท Posted at 21:13:24 on November 21, 2015 ยท (Permalink)

That's awesome! It's easy to fall into depression when you don't have something to be passionate about, never a bad idea to rekindle that fire from time to time with something new :)

dysphere ยท 13 points ยท Posted at 22:51:03 on November 21, 2015 ยท (Permalink)

Are you me? This happened with me and crypto too, only it was Cryptonomicon, and I read The Code Book after I got into crypto.

Imapseudonorm ยท 1 points ยท Posted at 23:17:23 on November 21, 2015 ยท (Permalink)

Heh, to be fair, I had read the Cryptonomicon long before all this happened, so I was sufficiently primed for the code book (see what I did there?)

PixInsightFTW ยท 1 points ยท Posted at 00:57:21 on November 22, 2015 ยท (Permalink)

Cryptonomicon didn't save my life, but it is among my absolute favorite books. That scene toward the end, when Randy programs the keyboard lights... amazing.

Every year or so, I'll think of a part of that book, go back and read it, and just keep on reading to the end from there. So good. Now I think I'll go read it again.

bipocni ยท 1 points ยท Posted at 10:18:56 on November 22, 2015 ยท (Permalink)

Dude that book was hard enough the first time around. I got almost 300 pages into it before I gave up, and had to come back later to finish it.

Dont get me wrong, I love that book. But once was enough.

oh_no_aliens ยท 11 points ยท Posted at 02:14:21 on November 22, 2015 ยท (Permalink)

Yours is a great example of how suicidal feelings do eventually subside. It's like an illness, of the mind, which takes time to heal.

Imapseudonorm ยท 71 points ยท Posted at 02:45:59 on November 22, 2015 ยท (Permalink)

I've always believed that suicide is a fundamental right we have, but it needs to be a truly autonomous decision, and any sort of temporary state (or neurochemical imbalance) that precludes making a rational decision means that decision isn't really yours to make.

That rule has helped me through a few of my darkest hours; it's my right to kill myself, but it CANNOT be an impulsive act, and CANNOT be based on any temporary states. Thus far, I've never regretted staying around.

I can honestly say, all of the worst moments of my life were also my best ones, inasmuch as they inevitably led me to much better circumstances.

Flash-man ยท 2 points ยท Posted at 18:29:51 on November 23, 2015 ยท (Permalink)

Wow this is sort of a weird catch 22

Imapseudonorm ยท 1 points ยท Posted at 19:11:08 on November 23, 2015 ยท (Permalink)

I see what you did there ;).

But yeah. I'm a firm believer in autonomy, but I also recognize that things like abnormal brain chemistry can be addressed medically, but until they are you can't really be acting autonomously, because you're being driven by some curable flaws, which means there's no legitimate reason to take a permanent step (suicide).

Of course, I'm also known for the absurd amount of recursion in my thought processes, so for some reason this all makes sense in my head.

Flash-man ยท 2 points ยท Posted at 22:56:03 on November 23, 2015 ยท (Permalink)

What you're saying makes a lot of sense. This idea that taking your own life is well within your right to decide, but only if you are in a correct state to make that decision, which you never/seldom would be in if feel that suicide is an option.

TotesMessenger ยท 1 points ยท Posted at 05:52:43 on November 22, 2015 ยท (Permalink)

I'm a bot, bleep, bloop. Someone has linked to this thread from another place on reddit:

If you follow any of the above links, please respect the rules of reddit and don't vote in the other threads. (Info / Contact)

[deleted] ยท 1 points ยท Posted at 20:22:18 on November 22, 2015 ยท (Permalink)

I have a similar thought process about it. I noticed that the times I got close to doing anything were at night, usually around 1-3 am. So I simply made the deal with myself that if I ever do it, it will be outside when the sun is shining.

[deleted] ยท -1 points ยท Posted at 21:33:46 on November 22, 2015 ยท (Permalink)

[deleted]

Imapseudonorm ยท 5 points ยท Posted at 21:43:26 on November 22, 2015 ยท (Permalink)

Eh, I disagree. Having been in the throes of pretty deep depression, and in more pain than I'd care to describe, at some point the altruistic idea of "must continue with this pain, lest I cause others more pain" stops working. You don't will yourself out of depression; you don't get better by just "getting over it."

That being said, I know that certain realities of my life have dealt me a specific hand in terms of the crazy that goes on in my head. I also know that the crazy in my head is NOT something I'm willing to let myself act on. So if suicide is a decision made outside of my "crazy" then I accept it as a rational act. Thus far, I cannot say that I've ever been able to make that rational choice, and I doubt I ever will.

But to look at someone else who is in pain (and if you're thinking about suicide, you're probably in a lot of pain), and to say simply "stay around, other people need you" in my experience just makes the pain worse. When I've talked other people down (including myself), I try to remove the "other" component, and look at it purely in terms of the consequences to the individual who is thinking about the act.

Generally speaking, there's enough going on in the self to find a reason to continue, sometimes all it takes is for someone to help you see it.

paperbackwriter73 ยท 3 points ยท Posted at 02:56:16 on November 23, 2015 ยท (Permalink)

May you always have someone to help you see it, friend.

starfirex ยท 1 points ยท Posted at 09:12:51 on November 22, 2015 ยท (Permalink)

They should call it suicitis

amwreck ยท 7 points ยท Posted at 22:10:40 on November 21, 2015 ยท (Permalink)

This would be an epic Amazon review! Glad you found something to work on and make you happy. May you stay happy for the remainder of your days.

aldld ยท 7 points ยท Posted at 13:39:57 on November 22, 2015 ยท (Permalink)

Reminds me of Bertrand Russell: "There was a footpath leading across fields to New Southgate, and I used to go there alone to watch the sunset and contemplate suicide. I did not, however, commit suicide, because I wished to know more of mathematics."

Bubo_scandiacus ยท 3 points ยท Posted at 22:12:11 on November 21, 2015 ยท (Permalink)

That's actually an amazing story. I'm glad you got through it, in a REALLY cool way too!!

Muskwatch ยท 3 points ยท Posted at 18:44:13 on November 22, 2015 ยท (Permalink)

I loved this book as a teenager - managed to solve the first four or five levels of his crypto challenge at the end using pencil and paper. it was really one of the funnest things I ever did and played a role in me becoming a linguist today.

Imapseudonorm ยท 1 points ยท Posted at 19:37:39 on November 22, 2015 ยท (Permalink)

All I hear is how cunning you are...

puzl ยท 2 points ยท Posted at 21:22:44 on November 21, 2015 ยท (Permalink)

Admit it, your passion for crypto lead you to mining bitcoin in the early days and now you're a millionaire!

Imapseudonorm ยท 12 points ยท Posted at 21:35:12 on November 21, 2015 ยท (Permalink)

Not Bitcoin. Doge! Much satisfaction! Many saves!

puzl ยท 3 points ยท Posted at 22:00:08 on November 21, 2015 ยท (Permalink)

For reals sheeb?

Imapseudonorm ยท 1 points ยท Posted at 23:19:55 on November 21, 2015 ยท (Permalink)

Feel free to check post history. /r/dogecoin is the whole reason I actually registered for a reddit account. :D

+/u/dogetipbot 500 doge

puzl ยท 1 points ยท Posted at 00:00:42 on November 22, 2015 ยท (Permalink)

Woo magic internet money!

PM_ME_UR_ASS_GIRLS ยท -1 points ยท Posted at 23:26:58 on November 21, 2015 ยท (Permalink)

That shit is still a thing?

puzl ยท 1 points ยท Posted at 00:01:05 on November 22, 2015 ยท (Permalink)

Do you ever get any PMS?

jeremyjava ยท 2 points ยท Posted at 06:36:09 on November 22, 2015 ยท (Permalink)

That was incredibly touching and inspiring. Thank you.

[deleted] ยท 2 points ยท Posted at 19:36:17 on December 17, 2015 ยท (Permalink)

Wow, hope you're doing well now mate

gronke ยท 1 points ยท Posted at 21:49:31 on November 21, 2015 ยท (Permalink)

And now he works at the CIA!

Imapseudonorm ยท 1 points ยท Posted at 23:21:00 on November 21, 2015 ยท (Permalink)

Heh, no, now he manages a specific IT department at a university. And that University's name? Albert Einstein.

gronke ยท 1 points ยท Posted at 23:26:26 on November 21, 2015 ยท (Permalink)

And the dean walked up to him and handed him a crisp $100 bill and whispered into his ear, "Welcome to the Republican Party"

SeaMenCaptain ยท 1 points ยท Posted at 23:00:47 on November 21, 2015 ยท (Permalink)*

[deleted]

What is this?

Imapseudonorm ยท 1 points ยท Posted at 23:16:31 on November 21, 2015 ยท (Permalink)

I've been an IT generalist for the past 15 years or so. There's been a couple of times where my interest in cryptography has paid off in terms of conversation, but it didn't really affect my career.

astrawnomore ยท 1 points ยท Posted at 04:39:06 on November 22, 2015 ยท (Permalink)

I never knew about any other books by Simon Singh, but I really enjoyed his book called Big Bang in my teens โ€” pretty much began my interest in astrophysics. I'm due to graduate this spring with a degree in physics.

TotesMessenger ยท 1 points ยท Posted at 04:49:47 on November 22, 2015 ยท (Permalink)

I'm a bot, bleep, bloop. Someone has linked to this thread from another place on reddit:

If you follow any of the above links, please respect the rules of reddit and don't vote in the other threads. (Info / Contact)

ParadoxSe7en ยท 38 points ยท Posted at 18:49:24 on November 21, 2015 ยท (Permalink)

Cryptonomicon by Neal Stephenson is also a pretty good read. http://www.amazon.com/Cryptonomicon-Neal-Stephenson/dp/0060512806

RyePunk ยท 42 points ยท Posted at 19:13:51 on November 21, 2015 ยท (Permalink)

I enjoyed the 5 page description of eating captain crunch to ensure it doesn't get soggy.

Imapseudonorm ยท 21 points ยท Posted at 20:34:59 on November 21, 2015 ยท (Permalink)

That and the "mapping london by the sound of raindrops" were my two favorite thought experiments in that book.

xxxStumpyGxxx ยท 8 points ยท Posted at 22:17:02 on November 21, 2015 ยท (Permalink)*

what about the bit where they "read" (spy) the erotic musings about boning on antique furniture and a stocking fetish for about 5 pages. i was so confused. i think it was about the inherent immorality and uselessness of most spying, or something, maybe. But i was seriously baffled by that entire chunk.

edit: van eck phreaking, reading the em field from the monitor on the other side of a wall and "seeing" whats on the monitor

LetThereBeR0ck ยท 8 points ยท Posted at 21:20:57 on November 21, 2015 ยท (Permalink)

I loved the incredibly long analogy where he describes the oral surgeon that removes his severely impacted wisdom teeth and likens him to America Shaftoe.

IsaacJDean ยท 2 points ยท Posted at 19:37:23 on November 21, 2015 ยท (Permalink)

I was laughing throughout that part. I didn't think this book would have so many parts that were hilarious

MtBakerScum ยท 12 points ยท Posted at 19:57:13 on November 21, 2015 ยท (Permalink)

I don't remember this part. I do for some reason remember the part about him optimizing his work output relating to the last time he masturbated though. Strange how the mind works....

yolo-swaggot ยท 3 points ยท Posted at 00:39:33 on November 22, 2015 ยท (Permalink)

He had a fetish for stockings, and his ex wife for dead relatives expensive furniture.

PixInsightFTW ยท 1 points ยท Posted at 00:59:58 on November 22, 2015 ยท (Permalink)

So many great parts like that. Randy's letter about the Phillippines jungle gold, the relatives laying out the furniture on the big axes... all the little vignettes that just make the story so rich and good.

[deleted] ยท 2 points ยท Posted at 06:50:46 on November 22, 2015 ยท (Permalink)

Nobody's mentioned my favorite. Bobby likening the Vickers machine gun to the band saw. Also, bonus mention to the Galvanic Lucifer, and how Lawrence puts away his little flashlight in shame when it is turned on.

riceshrug ยท 14 points ยท Posted at 20:43:31 on November 21, 2015 ยท (Permalink)

Anathem is my favorite math story

PedroFPardo ยท 16 points ยท Posted at 20:06:07 on November 21, 2015 ยท (Permalink)

Great, I'm going to get that book but in Spanish because English is not my first langua... Fuck that! 768โ‚ฌ??? I'll get the English version.

sn0r ยท 12 points ยท Posted at 20:46:02 on November 21, 2015 ยท (Permalink)

What the..? How? Why? Are the pages lined with platinum or something?

MangoBitch ยท 28 points ยท Posted at 22:05:08 on November 21, 2015 ยท (Permalink)*

My understanding is that some of the 3rd party sellers on Amazon use algorithms to automatically set and adjust prices. They tend to work pretty well and be stable if Amazon is also selling the book, since these prices tend to depend on what other people are selling for and Amazon's prices set a more reasonable and stable baseline.

There was a story about a textbook being sold for something like $32 million because two third party sellers were in an unintentional arms war to be the second cheapest seller. So the book started off at, say $100, but then they both kept increasing the price by, say, $1 each time the other one adjusted theirs. If that's not bad enough, imagine the price being incremented by a percentage with no cap, then you have exponential growth and we're all doomed.

This isn't a perfect example, but take a look at these colored pencils. They were sold by Amazon itself (not FBA) and were something like $12 or $13. Since then, they sold out. Although I can't figure out when exactly that was (other than between Oct 30th and earlier this week), this price tracker shows some minor instability (probably caused by inventory fluctuations), followed by a huge jump to a price no one would pay for those colored pencils even accounting for scarcity.

This is also what's going on when you see something going for $50 and with "9 used from $78.00."

I've heard it can help to message sellers and tell them that the price is ridiculous, because they could have very well not noticed what happened and will fix it.

RakeattheGates ยท 2 points ยท Posted at 16:37:59 on November 22, 2015 ยท (Permalink)

That is really interesting, thanks! There are now 3-4 people selling the pencis for ~$12 and then like 8 all priced at $45 so it all makes sense now.

JohnEffingZoidberg ยท 1 points ยท Posted at 23:38:35 on November 21, 2015 ยท (Permalink)

How is it not obvious to Amazon that something is messed up with their algorithms?

MangoBitch ยท 9 points ยท Posted at 00:12:45 on November 22, 2015 ยท (Permalink)

Amazon's are fine. It's third party sellers that mess it up.

[deleted] ยท 4 points ยท Posted at 21:26:31 on November 21, 2015 ยท (Permalink)

Agreed. I could understand if it had to be translated into Esperanto or some Masai clicking language...BUT SPANISH?!...it's a very widely spoken language.

Koshatul ยท 1 points ยท Posted at 02:35:38 on November 22, 2015 ยท (Permalink)

Translated into Base 64, RSA signed.

Or maybe just rot13 the whole book.

digoryk ยท 1 points ยท Posted at 00:57:59 on November 22, 2015 ยท (Permalink)

found the language nerd ^_^

almondmilk ยท 16 points ยท Posted at 21:03:54 on November 21, 2015 ยท (Permalink)

I just bought The Code Book over a week ago along with a few others. People in /r/math were talking about the documentary based on the book The Man Who Knew Infinity and how the book is better and less sensational. Through that I came across Fermat's Enigma, also by Simon Singh and which I'm currently reading, and The Code Book, as well as Journey Through Genius, which is about many mathematicians throughout the years and seems to be a mini-biography of each. Also just finished re-reading The Drunkard's Walk and convinced my mom to start reading it since I'm reading a book she bought for me. So there's some recommendations for anyone looking for some reading material.

Thanks for getting me excited to read The Code Book. I'll make sure it's next on my queue!

misplaced_my_pants ยท 3 points ยท Posted at 03:12:31 on November 22, 2015 ยท (Permalink)

Add Chaos, Genius, Isaac Newton, and The Information (all by James Gleick) in that order to your list.

spottyPotty ยท 1 points ยท Posted at 09:01:38 on November 22, 2015 ยท (Permalink)

Once you're at it you might as well add all feyman books: surely you're joking mr feynman, six easy pieces, tuva or bust, and others whose titles escape me right now.

[deleted] ยท 1 points ยท Posted at 21:46:35 on November 21, 2015 ยท (Permalink)

mom.

almondmilk ยท 2 points ยท Posted at 21:56:43 on November 21, 2015 ยท (Permalink)

Zip up your jacket, honey; it's cold out!

gr00ve88 ยท 53 points ยท Posted at 17:12:05 on November 21, 2015 ยท (Permalink)

let me know if you ever buy another copy, i'd love to have it! :)

rangeo ยท 8 points ยท Posted at 18:29:23 on November 21, 2015 ยท (Permalink)

Fuck Eve!

Scarrzz ยท 1 points ยท Posted at 01:48:45 on November 22, 2015 ยท (Permalink)

Is she hot?

evildonald ยท 11 points ยท Posted at 21:06:17 on November 21, 2015 ยท (Permalink)

Also, Cryptonomicon explains crypto in real-world examples (bike chains and u-boats) and is also a great fiction read!

lains-experiment ยท 4 points ยท Posted at 19:19:46 on November 21, 2015 ยท (Permalink)

Why are the real book versions cheaper than the digital copy?

Daniel15 ยท 9 points ยท Posted at 20:11:29 on November 21, 2015 ยท (Permalink)*

Different business models. I read a great blog post about it a few years ago but can't find it again. Here's a different post on it: http://booksavenue.co/2013/12/17/why-are-e-books-more-expensive-than-printed-books/

Edit: Here's the post I was thinking about! http://blog.nathanbransford.com/2011/03/why-some-e-books-cost-more-than.html

Natanael_L ยท 3 points ยท Posted at 19:30:01 on November 21, 2015 ยท (Permalink)

Publishers, that's why

Not_An_Ambulance ยท 1 points ยท Posted at 23:13:20 on November 21, 2015 ยท (Permalink)

How much money/space unsold copies take up and how much it costs to destroy an unsold copy.

VCavallo ยท 3 points ยท Posted at 19:08:07 on November 21, 2015 ยท (Permalink)

I begrudgingly passed over this book the other day at a used book store. I'm going back to buy it right now! Thanks!

BadMrFrostyCZ ยท 3 points ยท Posted at 20:49:24 on November 21, 2015 ยท (Permalink)

Simon Singh.

Look up Numberphile on youtube, Mr Singh and many other interesting chaps have contributed. The videos on mathematical paradoxes are my personal favs.

Ateisti ยท 3 points ยท Posted at 20:51:24 on November 21, 2015 ยท (Permalink)

Thanks for the recommendation. Just ordered a used copy from Amazon.de (they are practically free there, so you only have to pay for postage).

Evilandlazy ยท 3 points ยท Posted at 22:40:37 on November 21, 2015 ยท (Permalink)

I want one.

dsfox ยท 2 points ยท Posted at 20:32:08 on November 21, 2015 ยท (Permalink)

I'm not sure it's even an analogy.

KariTether ยท 3 points ยท Posted at 16:20:57 on November 21, 2015 ยท (Permalink)

Haha, me too, I was gifted it, read it and gave it away. I have bought it 3 times since!

batnastard ยท 2 points ยท Posted at 17:39:23 on November 21, 2015 ยท (Permalink)

I've only loaned mine out twice, so I still have it!

dubineer ยท 16 points ยท Posted at 19:42:04 on November 21, 2015 ยท (Permalink)

I loaned mine out too, but got it back encrypted. :(

randombits ยท 8 points ยท Posted at 19:58:30 on November 21, 2015 ยท (Permalink)

Now put your lock on it and send it back.

6ickle ยท 1 points ยท Posted at 00:43:43 on November 22, 2015 ยท (Permalink)

Maybe I'm really slow, but in the analogy who would be the one taking their lock back so that you can now unlock it?

PixInsightFTW ยท 1 points ยท Posted at 00:55:28 on November 22, 2015 ยท (Permalink)

LOVE that book. I want to create a class (I'm a teacher) and use that book as the text.

xjhnny ยท 1 points ยท Posted at 02:01:08 on November 22, 2015 ยท (Permalink)

I picked this up after watching the Imitation game. Cryptography is such an interesting subject

Yserbius ยท 1 points ยท Posted at 03:41:32 on November 22, 2015 ยท (Permalink)

The Code Book is a must read. Virtually every laymans description I've seen about cryptography from the last eight years is based off of something from that book.

edsobo ยท 1 points ยท Posted at 04:08:13 on November 22, 2015 ยท (Permalink)

I had the same thought while reading the description. Just read that book a little earlier this year and thoroughly enjoyed it.

HighSpeed556 ยท 1 points ยท Posted at 13:38:35 on November 22, 2015 ยท (Permalink)

Why the fuck is it cheaper to buy the paperback than it is to download the kindle version?

roy2593 ยท 1 points ยท Posted at 13:43:12 on November 22, 2015 ยท (Permalink)

Would it still be a good read for someone has read anything on it before?

[deleted] ยท 1 points ยท Posted at 16:11:36 on November 22, 2015 ยท (Permalink)

If you want to give another copy away... Here I am

Cyrus296 ยท 1 points ยท Posted at 20:19:24 on November 22, 2015 ยท (Permalink)

I love that book so much. Thanks for introducing it to a bunch of people. :)

The_Ballsagna ยท 1 points ยท Posted at 18:53:18 on November 21, 2015 ยท (Permalink)

Thanks for the recommendation - just bought a copy!

NuclearRobotHamster ยท -2 points ยท Posted at 18:31:04 on November 21, 2015 ยท (Permalink)

I read that book as background for my interview for Cambridge. Brilliantly interesting read.

Kreative_Katusha ยท -2 points ยท Posted at 22:51:26 on November 21, 2015 ยท (Permalink)

Cryptography should be a felony to use by he normal person except when doing online banking and such.

[deleted] ยท 32 points ยท Posted at 07:44:48 on November 21, 2015 ยท (Permalink)

Now give him a final, no cheat sheet, and fucking multitudes of encryption schemes.

Plutor ยท 19 points ยท Posted at 19:46:00 on November 21, 2015 ยท (Permalink)

This back and forth isn't really how any modern cryptographic system works, but it's neat anyhow.

HugeMallett ยท 3 points ยท Posted at 20:04:18 on November 21, 2015 ยท (Permalink)

but that requires you creating a code before she can listen to you... so she hasnt heard everything. you might as well recommend coming up with a new language and speaking in that language. its the same

Syrdon ยท 8 points ยท Posted at 21:31:40 on November 21, 2015 ยท (Permalink)

Once you suspect she is listening, you can make your last clear text message "multiply the following by a large prime, then send it back and divide my response by your prime". It does require that Eve not be able to send a message along the same channel though.

HugeMallett ยท 4 points ยท Posted at 22:10:38 on November 21, 2015 ยท (Permalink)

but eve knows her prime.... because when the second person sends it back she can do simple division to find her prime. its so easy

Syrdon ยท 6 points ยท Posted at 23:40:37 on November 21, 2015 ยท (Permalink)

Other people addressed that concern in more detail. The short version is thy this example is usefully wrong. It explains the basic idea, but isn't a functioning algorithm. Real encryption uses functions whose inverse is significantly harder to perform than the function itself.

rya_nc ยท 2 points ยท Posted at 05:57:29 on November 22, 2015 ยท (Permalink)

I slightly more detailed, but still fairly simple description:

Alice chooses two very large prime numbers (hundreds of digits long), p and q. The product of p * q is N. Alice chooses e as 65,537, a standard value for this purpose.

Alice tells Bob that he can send her a message by encoding it as a number, raising it to the eth power, dividing the result by N and sending her the remainder.

Bob does this, and Alice can use her knowledge of p and q (which neither Bob nor Eve know) to recover Bob's message. Recovering the message is somewhat more complicated.

Alice first calculates a value called phi equal to (p - 1) * (q - 1). Next, Alice uses the extended euclid algorithm to find a number called d, which when multiplied by e then divided by phi will give a remainder of 1.

The math happens to work out that if Alice raises number Bob sent her to the d'th power, divides the result by N and takes the remainder, she gets Bob's message.

Eve can't decrypt the message because without p and q (which Alice keeps secret) she can't calculate d, and the time required to figure them out with just N and e is effectively forever.

This is how the RSA cryptosystem works. in practice, there are a few more steps done for efficiency, and to prevent Eve from being able to guess what Bob's message was and then check if she's right.

Clasm ยท 0 points ยท Posted at 23:27:25 on November 21, 2015 ยท (Permalink)

This is only a basic example though. Using a larger matrix would drastically increase the complexity of the lock.

HugeMallett ยท 1 points ยท Posted at 14:04:01 on November 22, 2015 ยท (Permalink)

increased complexity doesnt mean its impossible to break especially not when she hears you dicuss how it will work

Clasm ยท 2 points ยท Posted at 16:37:57 on November 22, 2015 ยท (Permalink)

Of course it doesn't, that's not the point. The point is to make it so complex that, even if she does know the method, by the time she does manage to break it, the information is no longer relevant.

HugeMallett ยท 0 points ยท Posted at 18:06:10 on November 22, 2015 ยท (Permalink)

must i really spell everything out so obviously before you get it. look at the whole point of this

If a girl called Eve listens to absolutely everything you and your friend say to each other, then you can't tell each other secrets without Eve finding out too.

Clasm ยท 1 points ยท Posted at 18:09:53 on November 22, 2015 ยท (Permalink)

You're talking in circles now, and have brought nothing new to the conversation. I applaud your ability to be completely obtuse about how encryption is supposed to work.

HugeMallett ยท 0 points ยท Posted at 21:47:15 on November 22, 2015 ยท (Permalink)

oh my lord, theres nothing to bring up thats new. im just proving my point.

my last comment was proving you wrong christ

Clasm ยท 1 points ยท Posted at 21:59:44 on November 22, 2015 ยท (Permalink)

Except your 'proof' was just a flawed observation of yours. Saying it again doesn't change how wrong it is.

HugeMallett ยท 2 points ยท Posted at 22:10:04 on November 22, 2015 ยท (Permalink)

WHAT?

Of course it doesn't, that's not the point. The point is to make it so complex that, even if she does know the method, by the time she does manage to break it, the information is no longer relevant.

you agreed with me. you pretty much said, of course the example doesnt work. can you not see that now ive also shown you the original comment and yours? the comment says eve wont know it, your saying she will but it will take her a while ( which it wont, its two calculations lol)

Clasm ยท 1 points ยท Posted at 22:17:12 on November 22, 2015 ยท (Permalink)

Since you still don't quite grasp how computation complexity affects this, lets just say that, while any number crunching can eventually break the encryption, if the encryption is strong enough, the breaking process can take thousands of years. So, even though it can be broken, and Eve was the one who started the process, she will be long dead before that point.

HugeMallett ยท 1 points ยท Posted at 22:43:19 on November 22, 2015 ยท (Permalink)

no crunching required in the given example. none at all. she can calculate it if she hears how it will be encrypted which she fucking will as that is the point.

im aware its possible to have better encryption but i can prove that its easy to break NO MATTER WHAT instantly

Clasm ยท 1 points ยท Posted at 00:01:21 on November 23, 2015 ยท (Permalink)*

Instantly, huh? Try this one then:

4476 4678 3318 5316 3367 4384

5992 6939 5756 6774 5313 6585

5745 6205 5457 5719 4447 4102

3697 4208 3641 4083 3018 4545

3497 3768 3100 3782 1960 2950

2348 2176 2784 1200 2852 3792

As per the example, this message is sent with the instruction to the recipient to multiply it by a sixth order matrix of their choosing and send it back so that I can cancel out my encryption. The link expires in 15 minutes. Edit: Well, 15 minutes are up, plenty of time to establish a more secure method of communication between parties and you missed it.

HugeMallett ยท 1 points ยท Posted at 02:53:55 on November 23, 2015 ยท (Permalink)

what? did you want me to send it back... i cant play eve and one of the two people.

anyway ill explain:

first person has message 'm', primes making 'a' while other persons prime create 'b'... first message is sent: ma, returned message is sent: mab, third message sent:mb.

eve knows mb, mab,ma so she can very easily find m. childsplay. every other reply agrees

Clasm ยท 1 points ยท Posted at 03:58:03 on November 23, 2015 ยท (Permalink)*

Fine, here's mAB:

1854416 1087615 1221006 1380726 1297230 1346122

2700665 1590836 1875055 2083796 1899214 2007585

2291340 1347584 1497112 1696920 1643543 1641259

1690443 993192 1180575 1315488 1149725 1260636

1402065 806447 871830 1017533 959263 990274

1097156 700920 927296 967612 702556 890664

and mB:

26308 2722 -33908 33234 29631 -51271

42877 3085 -64659 64587 49426 -91274

31548 3691 -39671 38267 36447 -61268

27602 1502 -42352 42765 31174 -58837

18430 2210 -20343 19548 20263 -33263

23211 -653 -45391 47750 26123 -55729

Keeping with this notation, the first message was mA, and your goal as Eve is to find m. I will edit the post in 15 min.

Edit: Time is up. Again, A and B got away with securing communications outside of Eve's control.

HugeMallett ยท 1 points ยท Posted at 10:45:14 on November 23, 2015 ยท (Permalink)

idiot i was asleep. and you only have me two of the numbers.

eve would hear ma, mab, mb. if you cant see how basic that s without an example thats sad.

ma*mb/mab=m. all it takes is a calculaor which would work it out just as quickly as the second person could work out m. christ

Clasm ยท 1 points ยท Posted at 17:23:06 on November 23, 2015 ยท (Permalink)

I said mA was the first message, and with it and the two I sent later, you would have all three. However, that is besides the point. Your continuing excuses do not dispute the fact that you did not have time to circumvent or disrupt any covert operation that may have taken place, even though you had the same message three times.

The encryption scheme I used can barely count as such. It was a simple numerical substitution passed through a matrix. That was it. Such a message does not even have to be passed in text form and there are a myriad of ways obfuscate it to where Eve would not detect it in time, even if she was monitoring all forms of communication up until that point.

Fraye ยท 4 points ยท Posted at 00:40:19 on November 22, 2015 ยท (Permalink)

No, you can use public key encryption. Basically you have different primes for lock and unlocking, but the math works out so that you can give away the locking prime, and only you can unlock it still. Therefore people wanting to send you a message can just use your locking prime and send it.

Of course making sure that you know who the message came from is an entirely different problem :)

ThirdFloorGreg ยท 2 points ยท Posted at 21:44:05 on November 21, 2015 ยท (Permalink)

No it doesn't. Just because she knows you are going to encode your message, doesn't mean she can decode it.

HugeMallett ยท -4 points ยท Posted at 22:08:10 on November 21, 2015 ยท (Permalink)

yes it does if she gets to listen to EVERYTHING you both say. she would hear you discuss how you will encode and then apply that to decode.

ThirdFloorGreg ยท 5 points ยท Posted at 22:34:25 on November 21, 2015 ยท (Permalink)

Part of the method of encoding is "do something reversible to the message but don't tell me what it was."

Zagaroth ยท 3 points ยท Posted at 22:17:18 on November 21, 2015 ยท (Permalink)

If you choose the right numbers, she'd literally die before she could guess the right combination of primes.

HugeMallett ยท 1 points ยท Posted at 14:06:06 on November 22, 2015 ยท (Permalink)

no guessing is involved. when the second person send the number back alice can easily calculate what the number was multiplied by. very easily, literally the second number divided by the one she was sent. alice then divides the final number by this calculated number

flexmuzik ยท 2 points ยท Posted at 03:29:36 on November 22, 2015 ยท (Permalink)

Nope not with private/public key cryptography. Only the public keys are communicated but they don't do the data thief much good because the data is encrypted with the public key, then decrypted with the private key.

Watch this

bystandling ยท 1 points ยท Posted at 21:35:35 on November 21, 2015 ยท (Permalink)

but even if you did, you don't have to share the primes you're multiplying, so she might know the rule but not the specifics she needs to decode the message. And with every message you can change your primes. No real problem here imo.

HugeMallett ยท 2 points ยท Posted at 22:09:08 on November 21, 2015 ยท (Permalink)

she knows the rule so she just works out what you did to the number when you send it between each other..... then reverts it

Zagaroth ยท 5 points ยท Posted at 22:16:13 on November 21, 2015 ยท (Permalink)

These are very large prime numbers. You are forced to guess which ones were used. It takes a very very very very long time.

mallian ยท 5 points ยท Posted at 00:11:33 on November 22, 2015 ยท (Permalink)*

Maybe I'm wrong, but I don't think she needs to know the primes used if she has all three iterations of the message(which we are assuming she does in this scenario).

Product of the primes=P1 and P2
Message=M

The first iteration would be P1 * M
The second: P1 * P2 * M
The third: P2 * M

Multiplying the first two and last together would be P1 * P2 * M 2
Then dividing the result by the second iteration would cancel the square of M, P1 and P2 , leaving M. I think.

roboticon ยท 4 points ยท Posted at 02:31:05 on November 22, 2015 ยท (Permalink)

You are correct.

In reality, we aren't multiplying and then dividing. Straight-up multiplication doesn't work because the inverse (division) is just as easy. Instead we use a function that is simple to run, but outputs something really, really difficult to invert. Even if you know the function that was run, you don't know what the input was and you can't just run the function "backward" to get there.

mallian ยท 2 points ยท Posted at 03:36:37 on November 22, 2015 ยท (Permalink)

Ah, okay. Thanks. I couldn't see anything wrong with the math, but wasn't sure if I was missing something.

Zagaroth ยท 2 points ยท Posted at 03:09:54 on November 22, 2015 ยท (Permalink)

In a straight forward multiplication or XOR operation, you would be correct. In actuality, what they do for encryption is much more complicated, which is why the lock analogy fails when you try to apply it directly.

What you actually do (minus some details) is at least one end has generated a key pair from a very complicated formula that requires the input of two very, very large primes. They have then published one of those keys as public (this involves trust chains and verification, which is slightly a different topic, so we are starting with a known--good public key).

The other party then establishes communication, says "I'm going to give you a shared key", encrypts that shared key in the other party's public key, which can then only be decrypted by the matching private key. All further communication is then done with the shared key crypto (which is a LOT less computationally intensive and smaller for the same level of security. Which is why the primes are so very, very big to begin with).

There's a variation of this for elliptic curve cryptography, but I don't understand it well enough to describe it.

HugeMallett ยท 1 points ยท Posted at 14:06:19 on November 22, 2015 ยท (Permalink)

no guessing involved in the example used.

[deleted] ยท 1 points ยท Posted at 22:41:54 on November 21, 2015 ยท (Permalink)

[deleted]

mallian ยท 2 points ยท Posted at 00:26:31 on November 22, 2015 ยท (Permalink)

The thing is, I don't think she needs to guess when (as we're assuming) she has all three forms of the message that they are sending back and forth.

You can simply multiply the messages with only a single person's prime together to get a new message, and then just divide that new product by the one with the two peoples' primes together. No guessing necessary.

I wrote out what I mean here. Maybe I'm missing something?

WriteOnlyMemory ยท 1 points ยท Posted at 01:44:58 on November 22, 2015 ยท (Permalink)

Math looks right to me.

HugeMallett ยท 1 points ยท Posted at 14:04:44 on November 22, 2015 ยท (Permalink)

she doesnt need to guess. she can tell by how the numbers change when the two other people are replying to one another. no guessing is involved

mascaron ยท 1 points ยท Posted at 06:00:54 on November 23, 2015 ยท (Permalink)

It's not difficult at all to do this specific instance. Let's say my message to you is 57141913. Your message back to me is 111369588437. I message you again 44961481. If Eve is listening to us say the encoding method laid out by /u/UlyssesSKrunk above, she'll know the following:

X * the message = 57141913; (note: it doesn't matter how many primes you use, or even that you use prime numbers. If you multiply them together, it is still a value X).

57141913 * Y = 111369588437; (this is you putting the 2nd lock on)

111369588437 / X = 44961481; (this is me taking my lock off)

44961481 / Y = the message; (this is you taking your lock off)

From here, she just needs to solve for Y and plug it into the above formula to get the message:

Y = 111369588437 / 57141913 = 1949

44961481 / 1949 = the message = 23069 (reference check: x = 111369588437 / 44961481 = 2477. 2477 * 23069 = 57141913)

The more secure method would be to use an unspecified manner of strong encryption on both ends.

drpinkcream ยท 2 points ยท Posted at 00:21:17 on November 22, 2015 ยท (Permalink)

description of cryptography

cryptscription

Ubergeeek ยท 1 points ยท Posted at 23:20:31 on November 21, 2015 ยท (Permalink)

This same analogy was used in the children's educational uk series "how 2" on the 90s. Worth a watch.

745631258978963214 ยท 1 points ยท Posted at 20:05:59 on November 21, 2015 ยท (Permalink)

He actually took it from a common example in books and whathaveyou. Except the locks are usually color coded as well to make it easier to keep track of when explaining the different encryptions.

[deleted] ยท 1 points ยท Posted at 05:17:23 on November 22, 2015 ยท (Permalink)

[deleted]

rawling ยท 2 points ยท Posted at 10:03:32 on November 22, 2015 ยท (Permalink)

Seriously? You ask developers to come up with an encryption scheme that reveals both the message and both private keys to anyone listening?

Jasper1984 ยท 0 points ยท Posted at 19:54:06 on November 21, 2015 ยท (Permalink)

Diffie-Hellman exchange, roughly, right? There are different ways to do public key cryptography, another one is eliptic curve-based, and there is an relatively inefficient secure-hash-only based on too, not that i am that well-learned in this stuf...

Note that there are caveits, and "implementation concerns". And if Eve can change the messages, she do a man-in-the-middle attack.

christian-mann ยท 3 points ยท Posted at 20:12:09 on November 21, 2015 ยท (Permalink)

No, it's the three-pass protocol.

Laoracc ยท 3 points ยท Posted at 20:15:07 on November 21, 2015 ยท (Permalink)

The description is more of an XOR Cipher, but the first sentences made it sound like it was going to be DHE.

GemOfEvan ยท 223 points ยท Posted at 04:45:26 on November 21, 2015 ยท (Permalink)

I think I'm missing something. Alice has a message m and a product of primes a. She sends Bob the product ma. Bob has the product of primes b and sends back the product mab. Alice divides by a and sends back mb. Eve has heard the products ma, mab, and mb. (ma)(mb)/(mab) = m, so Eve now has the message.

assliquorr ยท 186 points ยท Posted at 05:47:42 on November 21, 2015 ยท (Permalink)

These type of cryptographic constructions are known as three-pass protocols. You're right, integer multiplication three-pass protocols are completely insecure, because multiplication is about as computationally intensive as its inverse, and so the plaintext is trivially reconstructed from the three transmitted messages. I guess integer multiplication three-pass is pedagogically useful, though, because you get an intuition that your three-pass operation must be commutative, and, as you've deduced, asymmetric in some way, so that it's not so easy to calculate the inverse.

Real three-pass protocols use commutative operations that are computationally asymmetric, like exponentiation modulo a large prime, or exponentiation in the Galois field. Computing the inverse of these operations would effectively be equivalent to solving the discrete logarithm problem.

kspacey ยท 35 points ยท Posted at 21:38:08 on November 21, 2015 ยท (Permalink)*

But computationally difficult is different from impossible. This makes it HARD for Eve to discern the message, but given sufficient time she has everything she needs to acquire the information.

Edit: lord you people are persistent. I know about P != NP and the fact that the level of difficulty in cracking the message is absurd. The issue is you may have obscured the message but you have not completely hidden it. As mentioned elsewhere that would require a one time pad, which Eve would hear.

The point is the statement is actually true unless you add in assumptions (like computation time) that fall outside the 'seems obvious' that was the mandate of the thread.

zKITKATz ยท 34 points ยท Posted at 21:48:07 on November 21, 2015 ยท (Permalink)

True, but the assumption we're making here is that the amount of time required to figure it out is so much that the message is more or less worthless by the time it can be figured out.

krimin_killr21 ยท 38 points ยท Posted at 23:06:40 on November 21, 2015 ยท (Permalink)

For example, a 2048 bit RSA key would take 6.4 quadrillion years to factor on a desktop computer. It's just not feasible.

Baloroth ยท 16 points ยท Posted at 01:35:15 on November 22, 2015 ยท (Permalink)

But just because it's not practical doesn't mean it's not possible, so technically the OP''s statement is actually true, not false (and in fact there is no way to communicate with theoretically unbreakable communication if Eve can read everything: even quantum cryptography only tells you that something is being intercepted).

causmeaux ยท 43 points ยท Posted at 01:51:28 on November 22, 2015 ยท (Permalink)

But if you strip away all practical constraints of time, then no secret can be kept by anyone, because you can just guess every possible message forever until you get the right one.

Baloroth ยท 10 points ยท Posted at 02:13:52 on November 22, 2015 ยท (Permalink)

You can guess, but the guess would be meaningless without some communication to verify it against (as an analogy, you could create the works of Shakespeare with a random number generator, but without the actual works themselves you'd never know you actually had the works of Shakespeare). One-time pads, for example, are truly unbreakable, even without any time constraint whatsoever (because even when you guess the message you have no means of verifying it is the message).

causmeaux ยท 4 points ยท Posted at 02:32:14 on November 22, 2015 ยท (Permalink)

You can't know you decrypted ANY message fully/correctly unless you can verify it was correct. Like if I decrypt a message from a spy using an infinite amount of time and for some reason the message is still relevant and everyone is still alive, and the decrypted message is not garbled, there may still be multiple layers of obfuscation in place and I can't know I understood the message communicated without verification.

bbctol ยท 1 points ยท Posted at 05:24:59 on November 22, 2015 ยท (Permalink)

To be fair, Eve can't be sure that her decryption is correct either, if we're willing to be as open defining "sure" as we were with "possible."

ZeroNihilist ยท 2 points ยท Posted at 12:12:10 on November 22, 2015 ยท (Permalink)

Since you would have no way of knowing which was correct, you would never actually gain any information from your random guesses. You can't in any meaningful sense know a secret using that method.

MauledByPorcupines ยท 1 points ยท Posted at 08:42:15 on November 22, 2015 ยท (Permalink)

A one-time pad would still be cryptographically secure against brute force.

kspacey ยท 1 points ยท Posted at 12:43:40 on November 22, 2015 ยท (Permalink)

No that's not the same, guessing that I have two fingers up behind my back won't help you if you also guess 1 3 4 5 6... N. There's a difference between being able to read out the message and gaining any information.

Jonny0Than ยท 1 points ยท Posted at 19:54:11 on November 22, 2015 ยท (Permalink)

A one-time pad is absolutely unbreakable (assuming the pad was shared securely and is used correctly). It cannot be broken because any attempt at brute force will return every possible message of the same length. Most will be gibberish, and one of them will be the real message, but there's no way to know which one it is.

The_Serious_Account ยท 4 points ยท Posted at 06:15:28 on November 22, 2015 ยท (Permalink)

Common confusion about quantum key distribution (quantum cryptography is a research field). You use it to share a key, check to see no one listened and then use it for one time pad, which is in fact theoretically unbreakable.

inio ยท 1 points ยท Posted at 02:32:40 on November 22, 2015 ยท (Permalink)*

even quantum cryptography only tells you that something is being intercepted

Detecting data leakage is sufficient to provide a truly secure channel. Alice sends bob random bits, and bob sends back a bitmask of which bits made it through undetected. Once bob has gotten enough secret bits, Alice XORs her message with those bits and sends that.

Baloroth ยท 1 points ยท Posted at 02:34:32 on November 22, 2015 ยท (Permalink)

Our hypothesis presumed that Eve can see everything sent between the two. That means none of the bits are secret.

inio ยท 1 points ยท Posted at 03:04:05 on November 22, 2015 ยท (Permalink)

This was replying to the last phrase of the parents message, referring to quantum "cryptography".

Jagjamin ยท 1 points ยท Posted at 14:10:55 on November 22, 2015 ยท (Permalink)

If she can't decode it during her lifetime even if she can convert all the matter in the universe into computronium, and live until the sun goes nova, then it's not possible for her to know what you've said.

kspacey ยท 1 points ยท Posted at 00:16:18 on November 22, 2015 ยท (Permalink)

That assumption was made nowhere in the original statement, and it defies the concept of the original question which is something that seems obvious at first but turns out to be incorrect.

Putting arbitrary computational limits in (no matter how large the limit) still takes this beyond intuitive.

DamonTarlaei ยท 8 points ยท Posted at 22:10:32 on November 21, 2015 ยท (Permalink)

What you state is true for all current crypto systems. In general, they are designed off asymmetric operations (functions where the inverse is orders of magnitude harder to compute than the function itself) and choosing a search space large enough that brute forcing takes too long to get the message out in useful length of time.

[deleted] ยท 1 points ยท Posted at 01:49:03 on November 22, 2015 ยท (Permalink)*

[deleted]

DamonTarlaei ยท 2 points ยท Posted at 08:12:37 on November 22, 2015 ยท (Permalink)

Sorry, I will clarify further and expand on what I said.

All operations in crypto-systems are reversible/invertible. This is what distinguishes them from hashing systems, which are inherently one way. The asymmetry in the operation that I was describing, is in the difficult of performing the operation in a given direction. I should have chosen better terms, but I had only recently woken up, so you might forgive me a lack of linguistic aptitude. Cryptographic operations are chosen such that, given an operation, a set of initial information and an input, both encryption and decryption are easy, and that, given the operation, the input and a LACK of some given initial information (the key), decryption is difficult.

The reason why this is important is that for a crypto system to be usable, you must be able to encrypt easily within the useful time of the message, to a level that makes cracking it within the useful time of the message very difficult/expensive to the point of it not being worth the effort to do so. If someone has a 0.000000001% chance of successfully cracking a message within a day and it is about what time I am having dinner with you tonight, then they're probably not going to bother cracking it. If it takes me 24 hours to encrypt however, there's no reason for me to do that, as by the time I've encrypted it, you'll have missed the lovely dinner.

That is the asymmetry I am talking about. Unfortunately, a lot of the methods that we are currently using are requiring way more significant investments to encrypt for diminishing returns on how difficult it is to crack.

Tl;dr - terminology issue. I am claiming that multiplication (and other operations) is an asymmetric operation only due to the fact that its inverse is way more computationally complex.

duskhat ยท 1 points ยท Posted at 04:47:45 on November 23, 2015 ยท (Permalink)

Well, candidates for asymmetric operations. Any provably asymmetric operation would show the existence of the one-way function

I'm not disagreeing with what you say though, more or less adding something to what's essentially true

[deleted] ยท 6 points ยท Posted at 01:49:26 on November 22, 2015 ยท (Permalink)

We're talking "it would take multiple lifetimes of the universe" difficult

Ideaslug ยท 4 points ยท Posted at 01:11:11 on November 22, 2015 ยท (Permalink)

Yeah but that's the basis of all our information security. All of it. If that concerns you, I would discontinue use transactions over the internet.

"Sufficient time" with most decryption hacks is literally many, many times the expected lifespan of our Sun.

kspacey ยท 3 points ยท Posted at 12:47:21 on November 22, 2015 ยท (Permalink)

You guys can be so condescending. Yes I understand how all of this works, my boyfriend spends a lot of time working on cryptography and I personally have a fairly notable mathematics background so NP hard problems aren't new to me.

But OP wanted statements that were 'intuitively obvious' but end up incorrect, and the response that eve has all she needs to evesdrop is factually true, so the response that spawned this thread is fundamentally incorrect (unless you add non-obvious constraints)

[deleted] ยท -1 points ยท Posted at 04:11:47 on November 22, 2015 ยท (Permalink)

Read up on P != NP for a better idea of how computational difficulty plays into encryption.

Wanderlustfull ยท 1 points ยท Posted at 23:35:09 on November 21, 2015 ยท (Permalink)

What is the discrete logarithm problem?

qhp ยท 2 points ยท Posted at 06:32:27 on November 22, 2015 ยท (Permalink)
Wanderlustfull ยท 2 points ยท Posted at 09:15:40 on November 22, 2015 ยท (Permalink)

Thanks very much! Lost me a bit in places, but I get the gist.

wont_start_thumbing ยท 1 points ยท Posted at 07:50:00 on November 22, 2015 ยท (Permalink)

Thank you.

[deleted] ยท -18 points ยท Posted at 15:11:24 on November 21, 2015 ยท (Permalink)

hu?

AcellOfllSpades ยท 29 points ยท Posted at 15:35:47 on November 21, 2015 ยท (Permalink)

Multiplying and dividing are easy so it doesn't work well for keeping secrets. In real life they use something that is easy one way but really hard the other so it's hard to find out the secrets.

fleshtrombone ยท 1 points ยท Posted at 19:31:35 on November 21, 2015 ยท (Permalink)

There was one thing that my discrete math prof never explained: why is it easy to determine if a huge number is prime? Like a number with 100+ digits?

Octopuscabbage ยท 11 points ยท Posted at 19:41:27 on November 21, 2015 ยท (Permalink)

There are algorithms that can remove a huge number of factors at once (like the sieve of erasthanos). They woke by figuring out that if one number isn't a factor then a bunch of others must also not be.

Laoracc ยท -1 points ยท Posted at 20:19:48 on November 21, 2015 ยท (Permalink)*

You can also see if it's divisible by all the integers from 1 to sqrt(prime). If none of those numbers cleanly (no remainder) divide the prime in question, then it's prime.

Edited to reflect this is always true.

almightySapling ยท 3 points ยท Posted at 20:56:29 on November 21, 2015 ยท (Permalink)

If none of those numbers cleanly (no remainder) divide the prime in question, theres a high likelihood it is in fact prime.

Yes, it is highly likely that prime numbers are prime.

Octopuscabbage ยท 2 points ยท Posted at 20:25:41 on November 21, 2015 ยท (Permalink)

Well usually both methods are combined

malnourish ยท 1 points ยท Posted at 20:42:00 on November 21, 2015 ยท (Permalink)

In what cases would that not be true?

Laoracc ยท 1 points ยท Posted at 00:35:32 on November 22, 2015 ยท (Permalink)

There aren't, my bad; edited my comment to reflect that.

All possible factors greater than the sqrt would need to be multiplied by a factor smaller than the square root. If both integers were greater than, the product would be larger than the prime in question.

mrbaggins ยท 1 points ยท Posted at 20:42:00 on November 21, 2015 ยท (Permalink)

Isn't that the definition of prime? It's not going to have two factors bigger than the square root

fleshtrombone ยท 1 points ยท Posted at 23:52:37 on November 21, 2015 ยท (Permalink)

don't you mean sqrt(n)?

grelphy ยท 5 points ยท Posted at 20:54:18 on November 21, 2015 ยท (Permalink)

Primality testing is relatively efficient. The Fermat primality test is probabilistic, but has a running time only barely slower than O(logยฒ(n)) in the magnitude of the number being checked (or O(nยฒ) in the number of digits). (Obviously you should check multiple times to ensure a very high probability of primality, and you should check that you didn't find a Carmichael number, but it's still quite fast even for multi-kilobit numbers, and not the only fast primality test.)

Given that we can check for primality relatively easily, and given that primes are relatively common (approximately one in log(n) near n, by the prime number theorem), finding gigantic primes is entirely computationally feasible.

fleshtrombone ยท 2 points ยท Posted at 23:47:52 on November 21, 2015 ยท (Permalink)

Oh wow O(log n) I didn't know it was that efficient, I'll have to check out the algorithm.

[deleted] ยท -44 points ยท Posted at 15:43:54 on November 21, 2015 ยท (Permalink)

Yeah I got that part thanks.

UncleMeat ยท 8 points ยท Posted at 16:10:18 on November 21, 2015 ยท (Permalink)

This pattern of crypto relies on one "direction" of the operation being way harder than the other without access to some secret. This is why Eve cannot just "undo" the locks. OP's example using multiplication was bad because it doesn't have this property. Instead we use fancier stuff.

Laoracc ยท 3 points ยท Posted at 20:22:02 on November 21, 2015 ยท (Permalink)

Also known as Asymmetric, or Public-Key Cryptography for those interested.

[deleted] ยท 1 points ยท Posted at 16:23:31 on November 21, 2015 ยท (Permalink)

Can you give an example?

clickstation ยท 9 points ยท Posted at 17:12:18 on November 21, 2015 ยท (Permalink)

I'm not a math wizard, but I'm imagining a function f(x) such that it's easy to calculate f(x) if we know x.. but it's not easy to calculate x if we know f(x).

In the example, if Alice passes "15" to Bob, and Bob passes "45" to Alice, we know that Bob multiplied the number by three, because the function is a simple multiplication: f(x) = y.x.

Because we know x = 15 and f(x) = 45, then y is simply 3.

But if f(x) = a.x5 + b.x4 + c.x3 + d.x2 + e.x... finding a, b, c, d, and e if you know only x and f(x) would be a lot harder.

Family-Duty-Hodor ยท 11 points ยท Posted at 17:06:16 on November 21, 2015 ยท (Permalink)

Take two (very large) prime numbers and multiply them. Boom, easy!
Now you get the product of those two primes. Try to tell me what numbers I used to get that product. Protip: you can't. Cause it's FREAKING HARD (that's a technical term).

mjk1093 ยท 138 points ยท Posted at 05:39:55 on November 21, 2015 ยท (Permalink)

It doesn't work exactly like OP suggested. The message is actually scattered around a modulo group so it's not discernible what the actual product is.

The metaphor of the two locks is genius though, that's a good way to explain cryptography to non-math people.

[deleted] ยท 27 points ยท Posted at 07:47:05 on November 21, 2015 ยท (Permalink)

It's a riddle in the crypto course I took, part of the first assignment. Bob wants to send Alice a ring through the mail, but everything gets stolen. He can send a safe, and the safe has a hasp that can hold any number of locks. With Alice's participation, as he can call her, how does he get the ring to her? Keys would also get stolen.

AMathmagician ยท 40 points ยท Posted at 16:28:52 on November 21, 2015 ยท (Permalink)

Until Eve is a jealous bitter rival who adds her own lock. If she can't be happy no one can.

sothisislife101 ยท 15 points ยท Posted at 18:45:00 on November 21, 2015 ยท (Permalink)

Eve can look, but she can't touch.

cem3394 ยท 1 points ยท Posted at 04:37:57 on November 22, 2015 ยท (Permalink)

No wonder she's jealous

[deleted] ยท 0 points ยท Posted at 18:58:54 on November 21, 2015 ยท (Permalink)

But would that stipulation make the analogy work for the real world?

sothisislife101 ยท 2 points ยท Posted at 19:03:12 on November 21, 2015 ยท (Permalink)

Not really, only in a broader/generic sense. Otherwise, it would depend on the method of communication and message transmittal.

I'm no expert though, so I can't really say much more confidently.

Rick0r ยท 4 points ยท Posted at 20:41:18 on November 21, 2015 ยท (Permalink)

Ransomware!

745631258978963214 ยท 4 points ยท Posted at 20:07:50 on November 21, 2015 ยท (Permalink)

But then eve has made it obvious that someone is tampering with the safe, so the two people are now on alert.

meltingdiamond ยท 5 points ยท Posted at 01:33:29 on November 22, 2015 ยท (Permalink)

But the rules are that everything in the mail gets stolen, so you are already alerted.

[deleted] ยท 1 points ยท Posted at 05:09:58 on November 22, 2015 ยท (Permalink)

Yes but she still can't open the safe because someone else's lock is still on it.

[deleted] ยท 14 points ยท Posted at 09:09:03 on November 21, 2015 ยท (Permalink)

Why wouldn't the safe get stolen?

univalence ยท 57 points ยท Posted at 10:39:23 on November 21, 2015 ยท (Permalink)

Too heavy. No one wants to carry that

Andrenator ยท 36 points ยท Posted at 10:43:01 on November 21, 2015 ยท (Permalink)

That is logical, you live up to your flair.

[deleted] ยท 14 points ยท Posted at 20:52:35 on November 21, 2015 ยท (Permalink)

Except the poor mailman that no one ever considers.

745631258978963214 ยท 3 points ยท Posted at 20:07:00 on November 21, 2015 ยท (Permalink)

Because the only thing in it is a spider web.

Publius82 ยท 2 points ยท Posted at 20:07:29 on November 21, 2015 ยท (Permalink)

Because it's useless, you can't open it. And only the sender knows what's actually in the safe; it might not be valuable at all.

110011001100 ยท 3 points ยท Posted at 19:26:38 on November 21, 2015 ยท (Permalink)

TBH, putting it that way makes the solution seem trivial

[deleted] ยท 7 points ยท Posted at 19:27:44 on November 21, 2015 ยท (Permalink)

Certainly wasn't fucking simple when I did it. You can see the solution, but you've been given the answer. I think only a few people in the class figured it out, without googling it.

110011001100 ยท 1 points ยท Posted at 19:32:38 on November 21, 2015 ยท (Permalink)

Well, true, but its a variant on the swap 2 integers without using a 3rd one problem...

Ofcourse, maybe I got this analogy cause I saw the answer as well

OperaSona ยท 3 points ยท Posted at 20:24:03 on November 21, 2015 ยท (Permalink)*

It takes a lot of steps to do it the first time, but if you're clever, you can make it so that anything you exchange after that only takes one mail (plus maybe another one to mail the safe if you want to send a message while Alice still has the box). You need 4 sets of lock+key for that though (maybe just 3?).

Edit: yeah I think 3 works.

[deleted] ยท 3 points ยท Posted at 20:55:37 on November 21, 2015 ยท (Permalink)

Two locks, bob puts the ring in the safe, locks it, sends it to alice. She puts her own lock on the hasp, sends it back. Bob takes his lock off, sends it to her, where she can take her own lock off at will.

Two locks, three mailings.

OperaSona ยท 6 points ยท Posted at 00:25:20 on November 22, 2015 ยท (Permalink)

3 mailings for 1 item to send. If you want Alice to answer once she gets that item by sending another item to Bob, you need 3 other mails. You have a rate of 1/3 in terms of items/mail, by using two locks.

Now with 3 locks: Bob puts an item and lock1 plus one copy of key1 into the box. He locks it with lock2 and sends it to Alice. Alice puts lock3 on the box and sends it back. Bob removes lock2 from the box and sends it to Alice. Alice removes lock3 from the box and opens it. She gets the first item and lock1+key1 from the box. She puts the second item in the box and locks it with lock1, sends it. Bob can open lock1 because he also has a copy of key1, so he gets the second item. He puts the third item in the box and locks it, once again, with key1. Etc. In the end, you have a rate that goes to 1 instead of 1/3.

If you don't like the fact that they share their lock/key, you can make both Alice and Bob send locks (without a key) that they can open, and that the other has to use to lock the box when answering. You still need the 3-message "handshake" part of the protocol early on, but you end up properly establishing a rate-1 connection with private/public key pairs: you just have to send your public key (the lock you can open) along with all your messages.

[deleted] ยท 3 points ยท Posted at 05:14:15 on November 22, 2015 ยท (Permalink)

Except if one person is unknowingly compromised. Then the encryption is broken.

OperaSona ยท 2 points ยท Posted at 05:30:10 on November 22, 2015 ยท (Permalink)

Without more specification on what a party being "unknowingly compromised" means, I think it can break pretty much any common encryption protocol. I mean in "real life", if a guy doing a man-in-the-middle attack knows your private key, he can read messages addressed to you and send messages as if he were you. The only difference between the scheme I discuss and the one with one 3 exchanges is that you compromise a longer sequence of messages (or items) by not generating new keys and doing a new handshake for each message. That's it.

[deleted] ยท 2 points ยท Posted at 05:58:54 on November 22, 2015 ยท (Permalink)

Your right. My example is invalid because if one person's method of communication is compromised (meaning the ability to read any file opened and also has a key logger) then anything that person sends or receives is also compromised. Making more hand shakes does nothing.

flexmuzik ยท 1 points ยท Posted at 03:33:12 on November 22, 2015 ยท (Permalink)

Bob sends Alice the lock but keeps the key for himself. She puts the ring in the safe and clicks the lock shut, then Bob opens it with his key once he gets it.

745631258978963214 ยท 0 points ยท Posted at 20:09:24 on November 21, 2015 ยท (Permalink)

Put a combination lock on it and tell her what the combo is.

That was too easy. :/

grelphy ยท 5 points ยท Posted at 20:56:45 on November 21, 2015 ยท (Permalink)

Eve has tapped the phone lines as well.

745631258978963214 ยท 7 points ยท Posted at 22:08:34 on November 21, 2015 ยท (Permalink)

Ugh, reminds me of those childhood games.

"So you're going to rob a bank, and there are three cops standing under a chandelier and you just have one laser beam shot. What do you do if the laser beam can destroy anything?"

"Well... if I have a laser gun, the military would pay me top dollar, so I'd just avoid shooting anyone and just make my money that way."

"NO, YOU CAN'T DO THAT. LET'S SAY YOU ALREADY ROBBED THE BANK."

"Well... I'd laser beam my way out of the bank by shooting through a wall... I don't want to kill the cops."

"NO, YOU CAN'T ESCAPE, YOU HAVE TO KILL THE COPS."

"WTF is the point of this game if I have to use the obvious answer of 'shoot the top of the chandelier so it crushes them'?!"

"HA WRONG. YOU'RE SUPPOSED TO SHOOT IN A STRAIGHT LINE SO IT HITS ALL THREE COPS."

"Fuck this shit, I'm gonna go drink my juice."

grelphy ยท 5 points ยท Posted at 22:16:01 on November 21, 2015 ยท (Permalink)

Because it's a dubious metaphor. ;) There's many fewer gotchas when the problem is posed the way it actually works: your goal is to communicate securely over an insecure channel with an adversary that will intercept but not modify all communications. You may assume that you and your communication partner share a language (that is also understood by your adversary) to negotiate whatever secure communication scheme you choose to use, but nothing else (in particular, no shared secrets). Go.

Bonus question: what if the adversary can make modifications?

[deleted] ยท 1 points ยท Posted at 23:43:58 on November 21, 2015 ยท (Permalink)

I'm genuinely interested. If the adversary can make modifications then you need a way to know what modifications were made in order to decrypt the original message. Right? Or is there a way around that? Ooh! Could the original sender factor out the original message, leaving just the added information? But then the original sender would have to communicate that information back to the recipient and that information wouldn't be useful unless you could be certain that the same modification was being made every time. If it was different, repeating the process would just throw you into a loop.

Can I get a hint?

grelphy ยท 2 points ยท Posted at 01:10:14 on November 22, 2015 ยท (Permalink)

To the best of my knowledge, there is no known solution to the man-in-the-middle case. I strongly suspect, though I do not know if it has been proven, that no solution is possible without a side channel. This is why it's a bonus question. ;)

Fortunately, in reality side channels exist, and so we can use them to transmit small amounts of information (enough to verify that we are talking to who we expect to be talking to and not a malicious intermediary) in ways the adversary cannot reasonably intercept. This is effectively what the certificate authority system does for the web: CA certificates are transmitted to you when you install your browser or operating system, and can then be used (modulo some assumptions about how much you trust the various authorities) to verify the identity of your endpoint. Webs of trust in PGP work similarly, albeit in a decentralized manner.

Of note is that these systems aren't necessarily perfect even if the adversary doesn't control the side channel (e.g. they're not poisoning the list of CAs in the version of Chrome you downloaded), because they involve a trusted intermediary verifying the identities independently, and that trust can always be misplaced. There's no perfect solution to thisโ€”in the extreme case, the person you're communicating with could be malicious and forwarding all your messages to the NSA. Trust, it turns out, is complicated!

ralgrado ยท 1 points ยท Posted at 23:50:55 on November 21, 2015 ยท (Permalink)

The first part is easy: I send my adversary my public key. He uses it to encrypt his message to me or we make the key exchange the other way around and I send him a message.

Bonus: I guess you need a way to exchange keys maybe in person to be able to sign messages so you can detect modifications. So all that's possible is to deny communication. Not sure if there is a better way. Modification at least should give that much.

745631258978963214 ยท -1 points ยท Posted at 22:24:28 on November 21, 2015 ยท (Permalink)

Then just say fuck it and use UPS or Fed Ex instead.

[deleted] ยท 2 points ยท Posted at 23:33:48 on November 21, 2015 ยท (Permalink)

what if the adversary can make modifications?

If the US government wants to tamper with your mail, how the fuck would using UPS and FedEx solve anything?

You're the annoying kid who always has to be right and never gets the point of the goddamn question.

[deleted] ยท 2 points ยท Posted at 20:10:11 on November 21, 2015 ยท (Permalink)

They aren't allowed, only simple locks and keys.

Edit: This is supposed to be like soviet Russia or something.

Pit-trout ยท 1 points ยท Posted at 21:02:56 on November 21, 2015 ยท (Permalink)

Itโ€™s more like youโ€™re supposed to assume that everything outside your own houses could be infiltrated be Eve.

skztr ยท 3 points ยท Posted at 19:50:59 on November 21, 2015 ยท (Permalink)

I think the "two locks" metaphor has a serious problem right now, though, in that everyone is used to "TSA Approved" locks, which the government has easy access to

EggShenVsLopan ยท 3 points ยท Posted at 01:49:51 on November 22, 2015 ยท (Permalink)

And the physical world mimics the digital world... Pictures of the TSA master locks were released so now anyone can open them. There are calls for the government to have backdoors in encryption and this is why it's a bad idea.

Ar-Curunir ยท 1 points ยท Posted at 22:09:32 on November 21, 2015 ยท (Permalink)

Even when the underlying field is Z/pZ for some cryptographic p, taking inverses in Z/pZ is easy.

To make this hard you have to work in a group where taking inverses is hard; namely groups where DDH is difficult.

Take a look at DH key exchange.

mjk1093 ยท 1 points ยท Posted at 01:37:56 on November 22, 2015 ยท (Permalink)

You are beyond my level of expertise here.

shadowjudge ยท 0 points ยท Posted at 20:58:28 on November 21, 2015 ยท (Permalink)

But with this example isn't it still susceptible to man in the middle attacks?

Person a sends to person b but eve intercepts and puts her own lock. Person a unlocks and sends again intercepted by eve which unlocks her lock and now has the original. To avoid detection eve sends a .... Ah I see where this falls down. Because eve doesn't have person a response to person b, the messages would have to come from eve for person a to get something they understand thus the variance in the messages could be detected.

mjk1093 ยท 3 points ยท Posted at 21:03:40 on November 21, 2015 ยท (Permalink)

But with this example isn't it still susceptible to man in the middle attacks?

I don't think so. If the recipient receives the message with two locks on it already, he will know that something fishy is going on.

More realistically, since the "lock" we're talking about is really the Generalized Euclidean Algorithm, trying to decrypt the message at the endpoint if there are too many locks on it will leave a message that is still garbled.

In other words, a middleman attack could destroy the message, but not steal it.

shadowjudge ยท 0 points ยท Posted at 21:07:31 on November 21, 2015 ยท (Permalink)

Yup I realized it as I followed the transaction towards the end with person b. Very good analogy. I'm impressed.

pfreedy ยท 12 points ยท Posted at 06:35:08 on November 21, 2015 ยท (Permalink)

Diffie helman exchange is an example of what he is describing: https://en.wikipedia.org/wiki/Diffie%E2%80%93Hellman_key_exchange#Description As one of the other commenters mentioned, it ustilizes the fact that the discrete log problem is difficult to solve (i.e. what Eve has to do to decode the message).

SkepticalOfOthers ยท 3 points ยท Posted at 20:56:09 on November 21, 2015 ยท (Permalink)

Actually, diffie-hellman relies on the hardness of the similar diffie-hellman problem. It hasn't been proven that discrete log and diffie-hellman are equivalent hardness, so it could be easier to solve than discrete log. (ie if you can solve discrete log, you can solve diffie-hellman, but we don't know if you can't solve discrete log given that you can solve diffie hellman)

Riffler ยท 26 points ยท Posted at 20:16:58 on November 21, 2015 ยท (Permalink)

Ignore the maths; it's just a bad example; also ignore the process, because that's wrong too. All that's any good is the analogy.

There are a number of encryption techniques known as public-key encryption. The most common involves very large prime numbers. This involves 3 numbers - 2 very large primes, and their product. There is a method of encrypting a message using the product of the primes in such a way that it can only be decrypted in a reasonable amount of time by someone who knows the original primes. Finding the primes from the product is possible, but not in a reasonable amount of time.

Alice has 2 very large primes, and knows their product. Bob wants to send her a message, and tells her so. Alice sends Bob her public key (the product) - these 2 crucial steps are missed out in the above simplistic example. Bob uses this to encrypt his message, and sends it to Alice. Alice can decrypt it using her private key (the 2 large primes). Eve knows everything that has passed between Alice and Bob but cannot decrypt the message because she doesn't have the private key. There is no need for Alice and Bob to meet, or communicate securely at any point, which is what makes public key encryption so immensely useful.

[deleted] ยท 3 points ยท Posted at 20:59:53 on November 21, 2015 ยท (Permalink)

[deleted]

Riffler ยท 11 points ยท Posted at 21:07:10 on November 21, 2015 ยท (Permalink)

Because factoring primes is very time-consuming. Large primes, in this context, generally means 128 bits, about 30 digits or so. You can derive the primes from their product, but it will take the most powerful modern computers thousands of years or more.

Personally, I'm concerned about someone finding my credit card details tomorrow. I'm pretty relaxed about them finding them a thousand years from now, as the card will have expired, and I'll be dead.

aguycalledmax ยท 7 points ยท Posted at 21:17:05 on November 21, 2015 ยท (Permalink)

I'm still confused as to why the 2 primes are needed at all. If the product is public, why cant eve divide by the product to get the original? why are the two primes necessary for decryption?

speedything ยท 13 points ยท Posted at 22:21:25 on November 21, 2015 ยท (Permalink)*

Because its an asymmetric algorithm. It's a little bit complicated but RSA does something along these lines...

  1. Generate two large prime numbers.
  2. Do a series of calculations with them that results in two public numbers
  3. You now have two private primes and two public numbers.
  4. Someone sending you a message can encypt it to cyphertext with this 'simple' algorithm:

    cyphertext = messagepublicKey1 mod publicKey2

    The clever bit is that this is not reversable. Even if you know publicKey1 AND publicKey2 it is very hard to calculate the message (i.e. would take 1000s of years of essentially guessing)

  5. Even more cleverly you CAN easily decrypt it if you know the primes that generated the public numbers:.

    message = cyphertextprivateKey1 mod publicKey2

So, for Eve to decypher the message they either need to guess the original primes or guess the message. Its an easier task to guess the primes but we're still talking years, and if they're big enough then Eve's grandchildren will be long dead before the computer correctly guesses them.

Note: I've left out calculations in step 2 as they go a little above my head and I don't think are necessary to explain the concept.

Zagaroth ยท 7 points ยท Posted at 22:35:53 on November 21, 2015 ยท (Permalink)

You are using large primes too make the numbers hard to guess. As a simple example, if such an equating was run using 11 as one of the primes, nothing but an 11 will do for cracking the code. If you use twelve, the code can also be cracked with 2, 3, 4, and 6. Since it involves 2 large primes, you have to guess both of them to come up work the same pair of keys.

The keys are equal in purpose, so public and private are arbitrary but can never change. This allows you to sign things to. If you create a hash of the message you are sending and encrypt that hash with your private key and send the message with the encrypted hash, the other person can use your public key to decrypt it( verifying you sent it), then compare the decrypted hash with a new hash they made of the Dane message. If the hashes match they know the message hasn't been altered.

ferwick ยท 1 points ยท Posted at 11:35:03 on November 22, 2015 ยท (Permalink)

The video linked in this comment explains the math better. It involves exponentian of the primes, not multiplication. Factoring the exponential result of two very large primes is significantly more difficult.

TheCannonMan ยท 1 points ยท Posted at 15:09:06 on November 22, 2015 ยท (Permalink)

Actually those are sizes typical of a symmetric crypto system, and way too small for rsa. RSA asymmetric crypto uses keys on the order of 2048 bits, 1024 and 4096 also see some use. 22048 is about 3*10616 So primes that are literally hundreds of digits long.

[deleted] ยท 4 points ยท Posted at 20:51:16 on November 21, 2015 ยท (Permalink)*

[deleted]

Riffler ยท 11 points ยท Posted at 21:17:19 on November 21, 2015 ยท (Permalink)

Because the algorithm used to encrypt the message requires primes, and because prime factorisation is unique. There is only one way to describe a number as the product of primes. Fortunately, there are various techniques for quickly checking that large numbers are prime, so finding suitable large primes is not hard.

If m = pq where p and q are prime, there is no other way of writing m as a product of numbers other than 1, p, q and m (except trivially qp).

babuji83 ยท 8 points ยท Posted at 21:10:58 on November 21, 2015 ยท (Permalink)

Because we don't have very efficient algorithms for prime factorization. The only real solution is still brute-forcing the answer (that is, guessing all possible answers until we get the right one). So we use really really big numbers so that it takes a long, long time to guess all the possible answers.

capilot ยท 1 points ยท Posted at 01:50:39 on November 22, 2015 ยท (Permalink)

d'oh!

Ok, now read about Diffie-Hellman key exchange. That's how it's really done.

Swiftzor ยท 1 points ยท Posted at 04:59:54 on November 22, 2015 ยท (Permalink)*

Yes, however typically encryption isn't quite this simple. A better example might be like this:

Alice has 2 sets of numbers aPub and aPriv, such that they are complements of each other. Bob has a similar set called bPub and bPriv. Both aPriv and bPriv are ONLY known by Alice and Bob respectively, while aPub and bPub are known by everyone.

Alice sends a message to Bob that looks like m * bPub. Bob then can read the message by doing m * bPub / bPriv. But it's important to remember that bPub =/= bPriv and is only really able to be decoded by the decryption algorithm that is being used that knows BOTH numbers. Or to go back to the locks analogy Bob hands locks out to everyone that only he has they key for.

***OR***

Alice would then send a message to Bob that looks like m * aPriv * bPub that could then be decrypted with m * bPriv * aPub. This however requires a bit of forethought on Alice and Bobs part as they need to have a previously established form of communications to build a decryption algorithm, but this method is MUCH more secure.

[deleted] ยท 1 points ยท Posted at 10:00:33 on November 22, 2015 ยท (Permalink)

Good question! I was thinking exactly this.

[deleted] ยท 1 points ยท Posted at 01:29:32 on November 22, 2015 ยท (Permalink)

You're not. This doesn't work. Don't trust anything you read in this sub.

[deleted] ยท 37 points ยท Posted at 09:56:05 on November 21, 2015 ยท (Permalink)
Brandonlucky ยท 2 points ยท Posted at 21:55:18 on November 21, 2015 ยท (Permalink)

Thank you.

mcgrotts ยท 26 points ยท Posted at 22:44:15 on November 21, 2015 ยท (Permalink)

The message is 2

I multiply it by 3 making it 6

You get it and multiply it by 4 giving you 24

I get the 24 and try dividing it by my 3 but only get 8

You get the 8 from me and divide that by your 4

You now have the message which is was 2.

jfb1337 ยท 52 points ยท Posted at 11:15:56 on November 21, 2015 ยท (Permalink)

Can't Eve still perform a MITM attack though? If Alice sends a locked box to Bob, but Eve intercepts it, and adds her own lock and sends it back to Alice, who removes her lock (thinking the other lock is Bob's) and sends it back, Eve can unlock the box and read it. Then she can go through the motions of locking it and unlocking it to get it to Bob without him suspecting anything, as he thinks they are Alice's locks.

Tillerino ยท 96 points ยท Posted at 11:37:57 on November 21, 2015 ยท (Permalink)

You're thinking of Mallory. Eve is tetraplegic and mute.

smog_alado ยท 44 points ยท Posted at 13:37:37 on November 21, 2015 ยท (Permalink)

Public key crypto assumes that Alice and Bob know how each other's locks look like before they start communicating.

In the analogy, the locks are the public keys and, as you correctly figured out, you need to exchange the public keys through a trusted (but not necessarily secret) medium before you start encrypting. You might meet up face to face beforehand or delegate the trust to a third party who knows both the public keys.

BlueFireAt ยท 5 points ยท Posted at 16:38:35 on November 21, 2015 ยท (Permalink)

How do they do it in general on the internet? Say I want to send an encrypted message to you, what trusted broker could we use?

jfb1337 ยท 15 points ยท Posted at 17:10:07 on November 21, 2015 ยท (Permalink)

SSL uses certificates signed by Certificate Authorities (CAs), and the list of CAs to trust is chosen by the developer of your browser or OS, or the manufacturer of your device, which you are assumed to trust by the fact that you are using their product.

More info: https://youtu.be/-enHfpHMBo4

BlueFireAt ยท 7 points ยท Posted at 17:37:34 on November 21, 2015 ยท (Permalink)

What if a CA gets compromised? I guess I can go in and update the list, right? And an OS update could probably remove it from the list, too?

gellis12 ยท 30 points ยท Posted at 19:55:51 on November 21, 2015 ยท (Permalink)

Lenovo and Superfish did just that one year ago.

They went out of their way to create a compromised CA, and have it running on every single laptop sold by Lenovo. Superfish then stepped in and performed man in the middle attacks on webpages that users loaded, and injected ads into them.

The worst part was that the private key that made this attack possible was the same on every single Lenovo computer, which meant that anyone could grab it and start using it to perform even worse man in the middle attacks on Lenovo users en masse.

The fact that Lenovo not only considered, but also went ahead with something as incredibly stupid and selfish as this, has convinced me to never ever buy anything from Lenovo in my life. If they destroyed users security for their profit once, what makes you think they'd ever think twice about doing it again?

death_hawk ยท 1 points ยท Posted at 05:59:35 on November 25, 2015 ยท (Permalink)

Dell literally just did it yesterday or the day before as well.

gellis12 ยท 2 points ยท Posted at 15:57:41 on November 25, 2015 ยท (Permalink)

Yep, I saw the thread about that. What a complete shitstorm Dell has created...

death_hawk ยท 1 points ยท Posted at 19:40:51 on November 25, 2015 ยท (Permalink)

I seriously want to punch whoever thought that was a good idea. Like seriously?

gellis12 ยท 2 points ยท Posted at 20:22:45 on November 25, 2015 ยท (Permalink)

It's Dell... Did anyone really have high expectations?

ErroneousFunk ยท 1 points ยท Posted at 07:02:49 on November 22, 2015 ยท (Permalink)

I bought a Lenovo laptop once. After about a week I just wiped it and reinstalled Windows, which was much better. Working with it felt... kind of like buying a new house that was not only furnished, but had, like, a sink full of dirty dishes and a 10 year old TV you didn't want.

Needless to say, that whole Superfish thing was shocking, but shouldn't have been terribly surprising to most people who have used their laptops...

gellis12 ยท 1 points ยท Posted at 12:42:55 on November 22, 2015 ยท (Permalink)

I'd have nuked it and installed Arch on day 1, personally...

pion3435 ยท 0 points ยท Posted at 20:41:02 on November 21, 2015 ยท (Permalink)

Nope, just the budget line. Thinkpads didn't have it.

gellis12 ยท 1 points ยท Posted at 22:56:25 on November 21, 2015 ยท (Permalink)

Source?

pion3435 ยท 1 points ยท Posted at 23:36:22 on November 21, 2015 ยท (Permalink)

Your own link.

stratys3 ยท 1 points ยท Posted at 00:02:36 on November 22, 2015 ยท (Permalink)

Yeah - but I think his point is if the company is willing to do it on their budget line today, will the ThinkPads have this type of issue tomorrow?

pion3435 ยท 1 points ยท Posted at 03:15:29 on November 22, 2015 ยท (Permalink)

I don't care. I was simply correcting a factual error.

langlo94 ยท 4 points ยท Posted at 17:52:08 on November 21, 2015 ยท (Permalink)

When CA's are compromised it is a big big problem. There's no practical solution as if yet, google "Trusting Trust" for more info.

jfb1337 ยท 3 points ยท Posted at 17:45:18 on November 21, 2015 ยท (Permalink)

Yeah, I'd imagine an OS update would remove it. I'm not sure how to update the list manually, but there's probably a way.

The video I linked mentioned a few cases where this has happened, and the CAs in question were bankrupted.

[deleted] ยท 6 points ยท Posted at 22:35:11 on November 21, 2015 ยท (Permalink)*

[deleted]

BlueFireAt ยท 1 points ยท Posted at 01:24:51 on November 23, 2015 ยท (Permalink)

Since Level 1 ISPs are roots in the Internet trees, are they the CAs you mean?

smog_alado ยท 3 points ยท Posted at 17:32:54 on November 21, 2015 ยท (Permalink)

Each web browser is bundled with a hardcoded list of certificate authorities

teh_maxh ยท 7 points ยท Posted at 01:03:12 on November 22, 2015 ยท (Permalink)

It's not really hardcoded; you can modify it if you want. There's usually not much reason to, but it's entirely possible.

Doomjunky ยท 1 points ยท Posted at 08:54:29 on November 22, 2015 ยท (Permalink)

The assumption was:

Eve listens to absolutely everything

The assumption was not:

Eve listens to absolutely everything and modify it.

Man in the Middle attacks (MITM) require assumption 2.

mveinot ยท 21 points ยท Posted at 18:43:52 on November 21, 2015 ยท (Permalink)

I add my own lock because fuck you and your stupid lock.

Had me chuckling to myself in McDonald's.

ikahjalmr ยท 17 points ยท Posted at 19:35:42 on November 21, 2015 ยท (Permalink)

That's insane. I thought I understood encryption after discrete math but this makes it so obvious

Siriacus ยท 7 points ยท Posted at 05:32:04 on November 22, 2015 ยท (Permalink)

Now neither Eve nor I can open it because it's locked. I add my own lock because fuck you and your stupid lock. I send it back to you.

Fucking lost it here.

dwimber ยท 12 points ยท Posted at 18:46:55 on November 21, 2015 ยท (Permalink)*

This is a great explanation... but now I'm curious. If the same box is seen going back and forth, couldn't this Eve chick easily figure out your prime number?

Let's say I want to use your analogy to send you a "4." I multiply it by my super-secret prime key (7.) Now I send you a "28." You multiply it by your key (11) and return to me a "308." I divide by my prime and return to you a "44." At this point, Eve would have seen the same message go back and forth and could tell that your key was an 11, that mine was a 7, and then read my original message... right?

edit I just realize that this very question was already addressed by /u/assliquorr . Thanks /u/assliquorr. Now, here's to hoping that I never have to type your name again! shudder

Natanael_L ยท 9 points ยท Posted at 19:35:57 on November 21, 2015 ยท (Permalink)

In reality math problems like RSA are used. They're strongly resistant to that analysis

[deleted] ยท -2 points ยท Posted at 19:17:30 on November 21, 2015 ยท (Permalink)

[deleted]

dwimber ยท 4 points ยท Posted at 20:42:19 on November 21, 2015 ยท (Permalink)

I'm aware that 4 isn't prime, thanks. In my example, that was the contents of the "message," and the primes that I used were 7 and 11. Also, I understand that much larger primes are used, but in my example I used small ones just as a way to get to my point. Unfortunatly, I couldn't think of a 30 digit prime number off of the top of my head, and I didn't want to guess at one because some random person would then feel the need to correct me on it.

[deleted] ยท -2 points ยท Posted at 00:56:33 on November 22, 2015 ยท (Permalink)

[deleted]

dwimber ยท 1 points ยท Posted at 01:56:36 on November 22, 2015 ยท (Permalink)

Thanks for googling those for me. I guess that would have been just as easy as using a simple example like I did.

umop_apisdn ยท 2 points ยท Posted at 20:04:46 on November 21, 2015 ยท (Permalink)

The size doesn't matter, the explanation is utter garbage, you don't just multiply by large primes, because division is very easy.

[deleted] ยท 1 points ยท Posted at 20:21:58 on November 21, 2015 ยท (Permalink)

[deleted]

umop_apisdn ยท 1 points ยท Posted at 18:43:21 on November 22, 2015 ยท (Permalink)

You seem to be hard of thinking. Eve sees that Alice sent 4. Bob multiplied it by his super secret prime and sent back 28. Eve just divides 28 by 4 to discover that the super secret prime is 7. No matter how big the numbers are, you just divide one by the other.

jefeperro ยท 1 points ยท Posted at 02:05:41 on November 23, 2015 ยท (Permalink)

Does she have xray vision ?

Zagaroth ยท 1 points ยท Posted at 22:38:49 on November 21, 2015 ยท (Permalink)

You have to divide by the same prime, which is why the size matters. There are lots of primes that big, and so you have to guess which primes they are to divide. And you don't know either prime, so you have to guess both. That is a hard problem in that it takes a lot of repetition and there is no short cut to guessing.

umop_apisdn ยท 1 points ยท Posted at 18:39:56 on November 22, 2015 ยท (Permalink)

No guessing is required at all. If you see that I sent 4, then they multiplied it by some secret prime and sent back 28, it takes no time whatsoever to work out what their secret prime was. You don't have to try lots of numbers, you just divide 28 by 4.

ForeskinLamp ยท 1 points ยท Posted at 21:35:19 on November 22, 2015 ยท (Permalink)

The size of the numbers doesn't matter. If you take the numbers out, you will see the following exchange of messages, where n is the original message:

  1. na = x (the original message sent, where a is person 1's prime)
  2. nab = y (the message that is sent back, where b is person 2's prime)
  3. nb = z (the third message sent where nab has been divided through by person 1's prime a)

If you listen in, you should know x, y, and z, so you have a system of 3 equations with 3 unknowns (a, b, n). At that point, it doesn't matter what the numbers are or how big they are, you can always determine the original message and the keys that were used by rearranging the equations and solving them simultaneously.

Grabthelifeyouwant ยท 1 points ยท Posted at 03:58:14 on November 22, 2015 ยท (Permalink)

A 30 digit number is not large for a computer. To my knowledge computational operations that require primes use primes around 120 digits in size.

Not trying to be a dick, I just think encryption is neat.

jefeperro ยท 2 points ยท Posted at 04:01:17 on November 22, 2015 ยท (Permalink)

I also think its neat but dont rely on a computer to encrypt. Ive got an ovaltine decoder ring that works just fine

DynaBeast ยท 7 points ยท Posted at 21:36:27 on November 21, 2015 ยท (Permalink)

What if eve puts on her own lock and sends it back?

bld2527 ยท 6 points ยท Posted at 03:47:51 on November 22, 2015 ยท (Permalink)

I FINALLY GET IT!!! OMG!

v3ctorman ยท 5 points ยท Posted at 06:46:47 on November 22, 2015 ยท (Permalink)

I add my own lock because fuck you and your stupid lock.

ty. Made me lul

dirice87 ยท 5 points ยท Posted at 06:58:20 on November 22, 2015 ยท (Permalink)

I've programmed for a few years and I never really got the mechanism of how it was done until now. Thank you, be a teacher!

karmaticforaday ยท 7 points ยท Posted at 17:37:18 on November 21, 2015 ยท (Permalink)

Question: if she knew what was up, couldn't she divide the second number by the first? Then she knows your set of primes, so when dude 1 sends it a second time, she divides that by your set of primes and has what ever was there originally? I understand the process of cryptography, but if she's been listening in, and say for example, communicating the method for cryptography was not encrypted, then she would have a means for breaking the code, yes?

ambrux ยท 17 points ยท Posted at 20:51:01 on November 21, 2015 ยท (Permalink)

I'm going to use the analogy example explain this, but here are the variables

M a b
2015 217645199 492876847
32452867 275604547
236887691 179424673
982451653 694847539

M : This is your message

a : These are your locks

b : These are the recipient's locks

You lock the box {M} by multiplying it with your locks. This makes locked box {Ma} with a value of {3312309379967778134280375206895560885}. You send this to the recipient.

Then the recipient adds their locks making the box {Mab} with a value of {56095416572385525154713578876611339168291668429150410898641475603328355}. This is returned to you.

Now you undo your locks creating locked box {Mb} with a value of {34124911482289254484502370986393738345}.

Finally the recipient unlocks their locks leaving an open box {M} with the value {2015}.

The weakness lies in that a Man-in-the-Middle (MitM) would have seen {Ma}, {Mab} and {Mb}. So now they have all the tools to reverse your locks and the recipient's locks.

Mathematically, {Mab}/{Ma} creates {b-composite} with a value of {16935439941582756567991251109872823}.

MitM does not know the values of {b}, but does not need them to unencrypt. Now take {Mb} and divide by {b-composite} to create {M} with the value, as ever, {2015}.

Strong encryption knows this weakness and therefor does not use straight multiplication, but by this analogy you are indeed correct. If the MitM misses any transmission though, the contents are secured

[deleted] ยท 1 points ยท Posted at 02:08:10 on November 22, 2015 ยท (Permalink)

Have you got an explanation for PGP? From what I understand of it, it's sort of like what you've described, where you're locking your message with their locks and the message contains yours, but how does the whole public-private key system work? How can you form a encrypted message from a public key that can't be decrypted with that same public key?

knightcrusader ยท 1 points ยท Posted at 04:43:06 on November 22, 2015 ยท (Permalink)

Strong encryption knows this weakness and therefor does not use straight multiplication

If I remember correctly from crypto class in college, isn't it a combination of using exponents and modulus?

SkepticalOfOthers ยท 6 points ยท Posted at 20:59:13 on November 21, 2015 ยท (Permalink)

yes. The described system is completely useless. If you'd like I can describe a system that actually works, but it's a little more complicated.

wings_like_eagles ยท 1 points ยท Posted at 04:02:20 on November 22, 2015 ยท (Permalink)

Please do!

SkepticalOfOthers ยท 5 points ยท Posted at 05:05:15 on November 22, 2015 ยท (Permalink)

So there's one important mathematical concept that's necessary to understand this, and it's called modular arithmetic. Most people already have a intuitive understanding of this idea from how they add hours on a clock. If I add 6 hours to 9:00, it becomes 3:00. The mathematical way of saying this is 9+6 = 3 mod 12 (because there are 12 hours on a clock). The more formal way of stating this is that 3 is the remainder when we divide (9+6) by 12.

When doing modular arithmetic, we have to choose what's called a "modulus". In the case of a clock that modulus is 12. In cryptography we usually use a very large prime number because primes yield certain properties in modular arithmetic that often make them easier to work with/more secure. The useful fact about modular arithmetic is that, with regards to some simple operations (addition, multiplication, and exponentiation), it works more or less exactly as it does with regular arithmetic.

So with this knowledge we can construct what's called the Diffie-Hellman key exchange. The premise is that two people, Alice and Bob, want to agree on a secret key with which they can encrypt messages, without Eve learning this secret key.

They start by agreeing on a large prime p, and a number g between 1 and p, called a generator. The "g is a generator" means that if we compute g1 mod p, g2 mod p, g3 mod p, etc. we will eventually get every number 0,1,2,...,p-1. This is important for maximizing the security of the system.

Next, Alice and Bob choose secret random values, a and b respectively, between 1 and p-1, and they compute ga mod p, and gb mod p. They then exchange these numbers. Alice them computes (gb )a =gba mod p, and Bob computes (ga )b = gab mod p. But, as I said, the rules of modular arithmetic work the same for exponentiation as they do with just regular numbers, so gab = gba mod p. Thus, Alice and Bob have agreed on the secret key gab .

Meanwhile, Eve has: p, g, ga , and gb . In order to break their encryption, Eve will have to find gab from ga and gb . This is called the Diffie-Hellman problem, and is believed to be computationally difficult (by "believed", I mean that a lot of people have worked over many years to try and find an efficient algorithm for solving it, but as of yet nobody has come up with one good enough.)

Most modern crypto protocols rely on some sort of mathematical operation that's easy to compute in one direction, but hard to do in the other direction without some sort of secret info (like a lock and a key. Anybody can lock a padlock, but only the person with the key can unlock it). Some of these problems are the discrete-log problem (which is closely related to the Diffie-Hellman problem), and another is integer factorization.

wings_like_eagles ยท 1 points ยท Posted at 21:41:07 on November 22, 2015 ยท (Permalink)

Thanks! So is using modular arithmetic basically the same as converting to a different base system? Or is there a significant difference? The analogy with the keys and the math makes a lot of sense. Is it possible and/or likely that major governmental code breaking institutions (i. e. the NSA) have figured out some way to solve it and are just trying to cover that up? I realize that at this point we start to get into the question of whether or not N = NP...

SkepticalOfOthers ยท 2 points ยท Posted at 23:47:01 on November 22, 2015 ยท (Permalink)

is using modular arithmetic basically the same as converting to a different base system?

It's pretty different. It's basically a way we can do math with finitely many whole numbers. Say I'm working with the set of numbers modulo 7. What this means is the only numbers I'm working with are {0,1,2,3,4,5,6}. If I add 2 numbers, say 5 and 4, I would normally get 9, but I want to stay in my group modulo 7, so I subtract copies of 7 from 9 until I get back into my group (in this case, I only need to subtract 7 once, 9-7=2). Multiplication is the same. Normally 54=20, but to get back into my group I subtract of 2 copies of 7: 20-(72)=20-14=6. Exponents are the same way.

The reason modular arithmetic is used is because it makes some problems that would normally be very easy, hard. For example, if I give you the number 2, and the number 1024, and asked you "solve 2x =1024" you could easily solve that with a computer. We have efficient algorithms for computing logarithms on regular numbers. Even if it was a much much bigger number than 1024, you could still easily solve it. However, if I gave you the number 2, and the number 5, and told you "solve 2x = 12 mod 13" you would have a much harder time. The usual methods to compute logarithms don't work here. Your best bet is to guess and check:

22 = 4 mod 13 23 = 8 mod 13 24 = 16 = 16-13 = 3 mod 13 25 = 224 = 23 = 6 mod 13 26 = 2*6 = 12 mod 13

And so there, 6 is the solution to "2x = 12 mod 13." Now we've come up with some pretty clever algorithms over the years to make the "guessing and checking" work as efficiently as possible, but it still basically boils down to guessing and checking.

Is it possible and/or likely that major governmental code breaking institutions (i. e. the NSA) have figured out some way to solve it and are just trying to cover that up? I realize that at this point we start to get into the question of whether or not N = NP...

Possible? sure. Likely? not at all. Keep in mind that government agencies all use these protocols. If they had discovered some easy method to defeat them, there's no reason to think they would continue to use them and to recommend them to other government agencies. Doing so would be a huge risk. If word about the algorithm got out, mountains of top secret encrypted data would become vulnerable.

Interestingly, P=NP isn't hugely relevant to the discrete log problem, because the discrete log problem isn't known to be NP-complete (ie, it could be that P!=NP, but discrete log could still be in P).

One thing that will be interesting in the coming decades is the advent of Quantum Computing. Many of these important problems on which these protocols rely (specifically, integer factorization (for RSA), and discrete log problem (for diffie-hellman key exchange, and El-Gamal)) are in a complexity class called BQP, meaning they can be solved in polynomial time with a quantum computer. This means quantum computers can break most known encryption systems. This will be a very real problem, and the NSA is already beginning plans to transition to quantum resistant encryption algorithms.

Unfortunately, many current post-quantum algorithms are relatively new, and haven't been studied much, so the guarantees about their hardness is much smaller.

daytodave ยท 1 points ยท Posted at 06:19:18 on November 23, 2015 ยท (Permalink)

How is g determined? Is there a formula that, given a large prime p, returns all the valid generatiors < p?

What scale of numbers are we talking about here? If Eve were to take an average ga and try to break the encryption by guessing and checking every number 1 < n < p until using gan mod p starts giving messages that make sense, how long would we expect it to take?

Are there any encryption/encoding systems that use "decoy keys", where decrypting using more than one key will give you coherent messages, but only one gives the message that was actually sent?

SkepticalOfOthers ยท 1 points ยท Posted at 09:58:36 on November 23, 2015 ยท (Permalink)

How is g determined? Is there a formula that, given a large prime p, returns all the valid generatiors < p?

No, no general formula. Generally you just pick a number and check to see if it's a generator. From what I understand, though, the way things are typically done is a bit different.

Firstly, there's no need to get a new p and g each time you perform the exchange. having a random a and b is more important. So typically p and g are shared and re-used among everyone. This is useful because some primes are better than others. Also, there's technically no need for g to be a generator. You just want the order of g to be sufficiently big (so that the number of possible keys is as big as possible), and a generator gives you the largest possible order. So oftentimes what is actually done is to take a large prime q, and compute p=iq-1 for i =2,3,... until p is prime. Then we can choose any random number to be g, and it will be, with high probability, of order some multiple of q. So if q is chosen to be big enough, then g will have large enough order as well.

What scale of numbers are we talking about here? If Eve were to take an average ga and try to break the encryption by guessing and checking every number 1 < n < p until using gan mod p starts giving messages that make sense, how long would we expect it to take?

Numbers hundreds of digits long. The NSA is recommending agencies use 3072-bit moduli, which are nearly 1000 digits long. Now if you actually wanted to try to break this you wouldn't want to start guessing "n" values and testing gan. You would instead try to find a, by solving the discrete log problem ga = A mod p for a. Once you have a, just compute (gb )a and you have the key, and you know it's the right key. You can do this using something like Index Calculus, or Pollard's Rho algorithm. How long this will take depends a lot on what your computing power is. It actually turns out to be pretty hard to estimate, but it's at least on the order of decades for a 3072-bit prime (at least until quantum computers gain ground).

Are there any encryption/encoding systems that use "decoy keys", where decrypting using more than one key will give you coherent messages, but only one gives the message that was actually sent?

I'm assuming you're asking this with regards to your previous questions about how you "check and decrypt until you start getting messages that make sense?" That doesn't really apply, since, as I mentioned, you can know for sure that you have the right key.

ArosHD ยท 1 points ยท Posted at 19:01:47 on November 21, 2015 ยท (Permalink)

She doesn't know the primes used though. There is no 'key' attached to the box for her to use.

jefeperro ยท -3 points ยท Posted at 19:17:58 on November 21, 2015 ยท (Permalink)

no

prydek ยท 6 points ยท Posted at 22:46:15 on November 21, 2015 ยท (Permalink)

Actually RSA and private key encryption don't work like this. RSA is more like having a lock that anyone can lock but only you can unlock, and trying to break it open would take so much time it's just not even worth the trouble because you'd be dead before it opens.

Private key encryption is basically only good once and has to be prearranged. So having two keys for the same lock, you lock it and send it to your friend and they unlock it, but then your lock is essentially useless so you throw it out and buy a new lock.

superbeastdj ยท 6 points ยท Posted at 04:10:55 on November 22, 2015 ยท (Permalink)

I'm finishing my associates in computer tech and specifically am taking a class called secure communications right now. This quote has finally made me grasp the whole shared / private key, general cryptography concept of everything. Thank you so much. And fuck these retarded books

Perpetualinvalidity ยท 3 points ยท Posted at 19:06:15 on November 21, 2015 ยท (Permalink)

A fundamental lack of understanding of how encryption works has been the base of my lack of trust in it. Nobody has been able to explain it like this before. Thank you.

LasagnaAttack ยท 3 points ยท Posted at 23:51:04 on November 21, 2015 ยท (Permalink)

Wait wait wait wait... Holy shit. This makes tons of sense. But why didn't I understand there are two locks when I studied it?

shoguante ยท 3 points ยท Posted at 01:00:22 on November 22, 2015 ยท (Permalink)

Best explanation I've seen. Thanks.

[deleted] ยท 3 points ยท Posted at 09:35:48 on November 22, 2015 ยท (Permalink)

Oh man! I work in IT and this has made more sense than any other description I've seen

shnicklefritz ยท 3 points ยท Posted at 09:59:31 on November 22, 2015 ยท (Permalink)

RSA: I give everyone a copy of my lock. Only I have the key

Fig1024 ยท 7 points ยท Posted at 18:33:02 on November 21, 2015 ยท (Permalink)

why does it have to be primes?

Natanael_L ยท 3 points ยท Posted at 19:33:13 on November 21, 2015 ยท (Permalink)

Doesn't have to be. RSA uses them, but ECC and NTRU and McEliece are also options

thorle ยท 2 points ยท Posted at 19:54:48 on November 21, 2015 ยท (Permalink)

For that given experiment it doesn't have to be primes, but because every number can be defined by primes and they also play a key role in cryptography, people tend to use them in such examples.

GuyBelowMeDoesntLift ยท 3 points ยท Posted at 21:39:35 on November 21, 2015 ยท (Permalink)

They're way harder to guess is basically what it boils down to

SkepticalOfOthers ยท 0 points ยท Posted at 20:57:26 on November 21, 2015 ยท (Permalink)

doesn't in this case. In fact, the example given is totally bogus and insecure. Decrypting the message without given only ma, mb, and mab is exactly as hard as "encrypting" them in the first place.

Luminarie ยท 2 points ยท Posted at 16:15:22 on November 21, 2015 ยท (Permalink)

I was having a hard time explaining encryption to a coworker without getting into the math. This is just beautiful.

umop_apisdn ยท 3 points ยท Posted at 20:01:29 on November 21, 2015 ยท (Permalink)

Unfortunately OP doesn't know much, has half remembered a very well known analogy, and made a statement of how it can be used ("by multiplying by a bunch of primes") that isn't remotely secure. Cryptography doesn't work like this at all.

But I guess that not many people have heard about the two locks technique for exchanging something securely and think it is really cool.

SybariticLegerity ยท 2 points ยท Posted at 17:07:15 on November 21, 2015 ยท (Permalink)

Wow. I was planning on posting to ELI5 asking for an explanation of cryptography, but you just explained it. You explained what I didn't get, anyway.

[deleted] ยท 2 points ยท Posted at 19:03:39 on November 21, 2015 ยท (Permalink)

That makes so much sense holyshit

elyisgreat ยท 2 points ยท Posted at 20:08:28 on November 21, 2015 ยท (Permalink)

a bunch of primes

Wait, so a composite number? What do the primes have to do with it?

rekoil ยท 2 points ยท Posted at 22:53:34 on November 21, 2015 ยท (Permalink)

The result of two prime numbers multiplied together can only be factored back (factoring = finding the original multiplied numbers) to those two primes. Multiply two non-prime numbers together, and there are multiple solutions to the factoring problem.

rawling ยท 1 points ยท Posted at 10:12:43 on November 22, 2015 ยท (Permalink)

But you don't have to care what the original primes were in order to break this scheme.

Dirty_Pretzel_ ยท 2 points ยท Posted at 20:17:42 on November 21, 2015 ยท (Permalink)

Thank you so much for this. I knew the mechanics, but it didn't make sense to me until your description. Not even the box part, just the top part was gold!

DeliveredByOP ยท 2 points ยท Posted at 20:32:53 on November 21, 2015 ยท (Permalink)

Question: since in practice it's not locks, but numbers, can't an observer divide the coded number sequence from the receiver to the sender by the second coded numbers sequence from the sender to the receiver to crack the sender's key? Then, take the first coded numbers sequence from the sender and use the sender's key to decode the message. Am I missing something?

Bman409 ยท 1 points ยท Posted at 23:37:47 on November 21, 2015 ยท (Permalink)

Exactly my question... Eve should easily crack this....

Truthier ยท 2 points ยท Posted at 20:57:22 on November 21, 2015 ยท (Permalink)

Why primes?

greenzr ยท 1 points ยท Posted at 21:17:21 on November 21, 2015 ยท (Permalink)

Because prime number divide roundly with only one combination. It keeps people from guessing the encryption key.

Truthier ยท 1 points ยท Posted at 21:19:47 on November 21, 2015 ยท (Permalink)

I see, thanks. How would they know they've guessed the right numbers, with or without primes ?

greenzr ยท 1 points ยท Posted at 21:34:06 on November 21, 2015 ยท (Permalink)

If box unlocks, you have the right key. If the number divides, the message partially decrypts. You can use those numbers and the message to decrypt the rest of it.

Truthier ยท 0 points ยท Posted at 21:47:54 on November 21, 2015 ยท (Permalink)

My question is, you have some input data, and you apply some transformation - how do you know the transformed data shows the key has been guessed? Is it by analyzing whether the decrypted data "looks like it should"? Like seeing if English text comes out? Or are there mathematical ways of knowing ?

greenzr ยท 1 points ยท Posted at 23:57:36 on November 21, 2015 ยท (Permalink)

There are mathematical ways of knowing. If you divide a number by another number, the computer can tell you if there's a remainder. When decrypting, we are only interested in the answer if our algorithm had no remainder.

Truthier ยท 1 points ยท Posted at 00:21:02 on November 22, 2015 ยท (Permalink)

I see. That makes sense. Thanks

LtWorf_ ยท 1 points ยท Posted at 14:25:50 on November 22, 2015 ยท (Permalink)

? No. Don't answer if you don't know the answer and say some random jargon.

ilrasso ยท 2 points ยท Posted at 21:17:21 on November 21, 2015 ยท (Permalink)

Why primes?

LtWorf_ ยท 1 points ยท Posted at 14:26:42 on November 22, 2015 ยท (Permalink)

Go look how RSA works. In this instance it makes absolutely no difference, the encryption does not work whatever numbers you use.

mikegustafson ยท 2 points ยท Posted at 21:23:20 on November 21, 2015 ยท (Permalink)

Thank you for this.

[deleted] ยท 2 points ยท Posted at 21:33:56 on November 21, 2015 ยท (Permalink)*

[deleted]

trynagetaway ยท 0 points ยท Posted at 02:29:40 on November 22, 2015 ยท (Permalink)

Exponentation is technically multiplication.

Lorf30 ยท 2 points ยท Posted at 21:51:27 on November 21, 2015 ยท (Permalink)

Stupid question but why are primes used? Can any integer be used as long as the number is divided by the aforementioned integer? Bio major here...

rekoil ยท 0 points ยท Posted at 22:46:11 on November 21, 2015 ยท (Permalink)*

Primes are used because the algorithm depends on factoring - figuring out which two whole numbers were multiplied together to arrive at another number. If you have the result and one of the factors, determining the other is simple - you just divide the result by the factor you have - but if you don't, it's impossible to determine what the original factors were without brute-force guessing (which for large numbers, takes a LONG time).

Since primes have only two factors (one and itself), a number that is result of multiplying two primes can only be the result of multiplying those two primes - no two other numbers could result in the same product. If one or both of the numbers is not prime, then there are multiple solutions to the factoring problem, making it far easier to "crack".

For example: 7 times 11 equals 77, but because 7 and 11 are both prime numbers, there are no other numbers (other than 1 and 77) that can be multiplied together to get that result. On the other hand, 7 times 10 equals 70, but because 10 is not prime - it's divisible by 2 and 5 - it's also true that 5 times 14 equals 70, and 35 times 2 equals 70 as well.

rawling ยท 2 points ยท Posted at 10:16:47 on November 22, 2015 ยท (Permalink)

You can break this entire encryption scheme without ever knowing or caring what the primes were.

WallHop ยท 2 points ยท Posted at 23:01:03 on November 21, 2015 ยท (Permalink)

Awesome explanation

GameStunts ยท 2 points ยท Posted at 23:13:22 on November 21, 2015 ยท (Permalink)

You've just solved in a handful of sentences something I've never understood about encryption.

Namely how the target computer can know how to decrypt what's being said without first being sent the key, which would surely mean someone listening in would get the key as well.

Really thank you for such a simple and effective explanation.

actuallyserious650 ยท 2 points ยท Posted at 23:17:05 on November 21, 2015 ยท (Permalink)

Can't person A deduce B's key because they have both the original message and the message with primes multiplied into it?

Kappadar ยท 2 points ยท Posted at 23:40:46 on November 21, 2015 ยท (Permalink)

Wow this is a good example

UlyssesSKrunk ยท 0 points ยท Posted at 03:21:23 on November 22, 2015 ยท (Permalink)

Yeah. Of all the examples I have ever heard in all my years of schooling, this one struck me as the most intuitively eye opening.

[deleted] ยท 2 points ยท Posted at 23:44:14 on November 21, 2015 ยท (Permalink)

This is the answer to the programming interview question the 2 people, 2 locks, and a box being shipped.

Sanity_fading ยท 2 points ยท Posted at 23:46:19 on November 21, 2015 ยท (Permalink)

Pure

Brilliance

Haaaarry ยท 2 points ยท Posted at 00:22:09 on November 22, 2015 ยท (Permalink)

That's really cool!

capilot ยท 2 points ยท Posted at 01:47:55 on November 22, 2015 ยท (Permalink)*

That is such a wonderful description.

I just want to add a tweak: don't multiply by several small primes; those can be factored. It's much better to multiply by one really big prime.

Oh, and the original message should be a really big prime too. So add a lot of gibberish to the end if you have to, and tune the gibberish so the final result is a prime.

Orc_ ยท 2 points ยท Posted at 01:48:56 on November 22, 2015 ยท (Permalink)

+1

Abuv ยท 2 points ยท Posted at 02:02:34 on November 22, 2015 ยท (Permalink)

By far the best, funniest, and easiest way to explain encryption.

LiveNeverIdle ยท 2 points ยท Posted at 02:39:39 on November 22, 2015 ยท (Permalink)

This is the best description of this I've ever seen. Thanks man!

flexmuzik ยท 2 points ยท Posted at 03:27:33 on November 22, 2015 ยท (Permalink)

I'm a noob so this might be all wrong but...

I don't like this analogy because it involves the package being sent 3 times, but in reality with public key cryptography it only ever gets sent once. Sure the public keys have to be exchanged, but after that, every package only travels one time. From sender to recipient.

asharwood ยท 2 points ยท Posted at 03:33:18 on November 22, 2015 ยท (Permalink)

"I can take your shit" aka I can see what's in the box. Great explanation otherwise.

booleanerror ยท 1 points ยท Posted at 04:33:54 on November 22, 2015 ยท (Permalink)

What's in the fucking box?!

TheF0CTOR ยท 1 points ยท Posted at 05:37:17 on November 22, 2015 ยท (Permalink)

a cat

booleanerror ยท 1 points ยท Posted at 06:44:36 on November 22, 2015 ยท (Permalink)

Dead or alive?

Hanse00 ยท 1 points ยท Posted at 11:35:27 on November 22, 2015 ยท (Permalink)

Both, or neither.

andersonpaac ยท 2 points ยท Posted at 04:33:49 on November 22, 2015 ยท (Permalink)

You just explained PGP

Kants_Pupil ยท 2 points ยท Posted at 05:22:43 on November 22, 2015 ยท (Permalink)

While correct, I feel as though you aren't a mathematician (bestof claims you are) as there is neither an Alice nor a Bob in your story.

SwimmingNaked ยท 4 points ยท Posted at 05:47:32 on November 22, 2015 ยท (Permalink)

Fucking Mallory sets the box on fire.

Encyphus ยท 2 points ยท Posted at 06:01:32 on November 22, 2015 ยท (Permalink)

Wait... AES encryption uses prime numbers too?

apathyzeal ยท 2 points ยท Posted at 06:53:09 on November 22, 2015 ยท (Permalink)

This nearly brought a tear to my eye.

donrhummy ยท 2 points ยท Posted at 08:49:20 on November 22, 2015 ยท (Permalink)

that's a fabulous explanation of the exchange of keys!

[deleted] ยท 2 points ยท Posted at 10:02:41 on November 22, 2015 ยท (Permalink)

[deleted]

speedyjohn ยท 1 points ยท Posted at 21:48:33 on November 22, 2015 ยท (Permalink)

Multiply it by whatever you want. Any number is the product of primes.

MyUnhappySecret ยท 2 points ยท Posted at 13:06:10 on November 22, 2015 ยท (Permalink)

This is so great

randytang89 ยท 2 points ยท Posted at 14:47:41 on November 22, 2015 ยท (Permalink)

well damn, that explanation of encryption was like a good glass of wine. that's pretty awesome.

Danda_Nakka ยท 2 points ยท Posted at 13:35:11 on November 27, 2015 ยท (Permalink)

Holy shit. This is one of the greatest explanations I have ever read in my entire lifetime

PM_ME_MESSY_BUNS ยท 2 points ยท Posted at 23:30:46 on December 9, 2015 ยท (Permalink)

holy shit

i finally understand

this is incredible

i'm a really computer-savvy person but i've never understood perfectly how this works

anonymousproxy404 ยท 1 points ยท Posted at 04:45:23 on November 21, 2015 ยท (Permalink)

That's really clever! Why does it have to be primes though?

Astrith ยท 11 points ยท Posted at 04:54:42 on November 21, 2015 ยท (Permalink)*

I believe the idea is that primes are harder to "accidentally" divide by. For example, if you multiply by 60, there are many ways to divide that. 6x10, 2x3x10, 60x1, 30x2, 15x4 and such. If it's a prime then there's only one way to divide it and so the chance of Eve accidentally finding a way to unlock the lock is lesser. (I have no idea about cryptography though, this may be totally wrong)

eegabooga ยท 7 points ยท Posted at 08:20:53 on November 21, 2015 ยท (Permalink)

This is correct. The Fundamental Theorem of Arithmetic states that each number has one unique prime factorization. Finding the prime factors of a numbers is hard (there's no quick shortcut for finding them) so with huge numbers, it'll take a really long time.

mormotomyia ยท 2 points ยท Posted at 13:53:15 on November 21, 2015 ยท (Permalink)

finding primes is in NP or?

masklinn ยท 6 points ยท Posted at 15:40:12 on November 21, 2015 ยท (Permalink)

It is in NP and co-NP. It's currently assumed to be NP-Intermediate.

mormotomyia ยท 13 points ยท Posted at 15:45:14 on November 21, 2015 ยท (Permalink)

Okay. I have no idea what this means but thanks.

Natanael_L ยท 2 points ยท Posted at 19:37:56 on November 21, 2015 ยท (Permalink)

Computational complexity. In other words, given how long the numbers are, how much time will it take at most to crack?

mormotomyia ยท 1 points ยท Posted at 19:43:13 on November 21, 2015 ยท (Permalink)

No I meant what is the difference between NP and NP-Intermediate

masklinn ยท 1 points ยท Posted at 15:24:34 on November 22, 2015 ยท (Permalink)

NP is the union of P, NP-complete and NP-intermediate. So NP-intermediate is anything NP which is neither P nor NP-complete.

hotoatmeal ยท 1 points ยท Posted at 18:34:40 on November 21, 2015 ยท (Permalink)

With a quantum computer it's in P.

mormotomyia ยท 0 points ยท Posted at 18:42:02 on November 21, 2015 ยท (Permalink)

Uhm. its still in NP. Even in a Quantum computer. In quantum logic howhever its P. The problem remains in the same class for our logic. But not for a logic that goes beyond that.

hotoatmeal ยท 1 points ยท Posted at 19:13:35 on November 21, 2015 ยท (Permalink)

Shor's algorithm is O(n3)

aldld ยท 4 points ยท Posted at 05:37:43 on November 21, 2015 ยท (Permalink)

My knowledge of cryptography is limited, but so far nobody has found an efficient algorithm for factoring integers. (Well, without using quantum computers.)

LtWorf_ ยท 1 points ยท Posted at 14:24:02 on November 22, 2015 ยท (Permalink)

Factoring a number is hard because basically the algorithm is to try to divide for every prime number, and the ones used are really large; thus the operation is expensive.

That said, the example does not show RSA, but some very silly and crackable in 3seconds scheme.

After seeing the messages ax and abx you know the value of "b", then when you see the message "bx" you know the value of x. Prime numbers or not, nothing makes that scheme working.

The_Highlife ยท 3 points ยท Posted at 17:33:38 on November 21, 2015 ยท (Permalink)

Great analogy, but why only primes?

gagnonca ยท 1 points ยท Posted at 22:13:04 on November 21, 2015 ยท (Permalink)

To make the math work and make the problem hard to solve. Here's a much better explanation that doesn't get the problem wrong like OP: http://youtu.be/YEBfamv-_do. (Important part starts around 3 minutes with a much better analogy)

zacker150 ยท 1 points ยท Posted at 22:39:21 on November 21, 2015 ยท (Permalink)

Because otherwise, you could just brute force divide out each prime factor of a or b

The_Highlife ยท 2 points ยท Posted at 23:05:47 on November 21, 2015 ยท (Permalink)

I'll take your word for it. I'm not comfortable enough with number theory to intuitively understand this.

LtWorf_ ยท 1 points ยท Posted at 14:19:10 on November 22, 2015 ยท (Permalink)

Neither does he. In reality the operation used is exponentiation/modulo. With multiplication it makes no difference at all. Also, the example is a very silly scheme that can easily be broken.

If x is the message, these are going on

ax abx bx

But having all these, you can just solve for a and b and obtain x.

MadTux ยท 2 points ยท Posted at 17:58:58 on November 21, 2015 ยท (Permalink)

It's a wonderful algorithm and everything, but it's still mathematically possible for Eve to crack it. She just needs a lot of processing power.

addandsubtract ยท 5 points ยท Posted at 22:05:38 on November 21, 2015 ยท (Permalink)

Given enough time, everything is solvable.

MadTux ยท 3 points ยท Posted at 22:11:32 on November 21, 2015 ยท (Permalink)

Except that with a cipher like the one-time pad, you actually can't crack it without the key, since the encrypted text is literally completely random, so the plaintext could be anything with that specific length.

[deleted] ยท 2 points ยท Posted at 19:45:14 on November 21, 2015 ยท (Permalink)

That's the struggle man, the encyptors v the decryptors.
A never ending battle

LtWorf_ ยท 1 points ยท Posted at 14:20:02 on November 22, 2015 ยท (Permalink)

A lot? That thing is not RSA, it would take perhaps 3 seconds to crack it.

ztsmart ยท 3 points ยท Posted at 19:27:41 on November 21, 2015 ยท (Permalink)

That is the best description of cryptography I've ever read

BoodieTraps ยท 2 points ยท Posted at 17:36:30 on November 21, 2015 ยท (Permalink)

I want to up vote this more. I totally get it now.

tuffus ยท 1 points ยท Posted at 18:48:19 on November 21, 2015 ยท (Permalink)

Kahn Academy?

745631258978963214 ยท 1 points ยท Posted at 20:14:33 on November 21, 2015 ยท (Permalink)

KKKKAAAAAHHHHNNNNN!!!!!!

(Also, it's khan academy)

cjluthy ยท 3 points ยท Posted at 20:21:37 on November 21, 2015 ยท (Permalink)

(Also, it's the Wrath of Khan.)

745631258978963214 ยท 2 points ยท Posted at 20:22:29 on November 21, 2015 ยท (Permalink)

Ah, damn. A taste of my own medicine, I see.

dangil ยท 1 points ยท Posted at 19:46:11 on November 21, 2015 ยท (Permalink)

Thats diffie-hellman right?

sberrys ยท 1 points ยท Posted at 19:48:28 on November 21, 2015 ยท (Permalink)

But what if Eve found a way to unlock the type of lock you have and she intercepts the box before it gets to your friend?

teruma ยท 1 points ยท Posted at 20:43:39 on November 21, 2015 ยท (Permalink)

Is this how data encryption works?

Umbrall ยท 1 points ยท Posted at 20:55:44 on November 21, 2015 ยท (Permalink)

Can't Eve tell you she's your friend, and send back a box with primes that she knows before your friend can?

MegatonMike ยท 1 points ยท Posted at 21:08:23 on November 21, 2015 ยท (Permalink)

https://youtu.be/U62S8SchxX4

Your explanation was the best explanation of diffie hillman I've seen on text. This is the best video I've seen of it.

wdonnell ยท 1 points ยท Posted at 21:10:11 on November 21, 2015 ยท (Permalink)

Aren't most people smart enough to use Different types of encryption? So one couldn't remove the "locks" out of order. It would corrupt the data

squeeterpig ยท 1 points ยท Posted at 22:01:26 on November 21, 2015 ยท (Permalink)

Is that literally all encryption is or can there be more to the encryption process?

dlp211 ยท 1 points ยท Posted at 00:01:19 on November 22, 2015 ยท (Permalink)

That isn't even close to the tip of the iceberg. Good encryption systems are complex and multi-layered. They require trust, secrecy, and privacy, and what is described above barely falls into secrecy and doesn't cover all the minutia required to get real secrecy.

Nagger_ ยท 1 points ยท Posted at 23:34:27 on November 21, 2015 ยท (Permalink)

So the only reason this works is because eve doesn't know whether the box is in lock 1 or lock 2 stage so she can't guess what primes to divide by?

mallian ยท 1 points ยท Posted at 00:00:35 on November 22, 2015 ยท (Permalink)

In that case, couldn't she just take the first and last messages and multiply them to each other and then divide the result by the middle(chronologically speaking) to get rid of the primes and the square to get the original message(assuming she has all three iterations of the message, anyway)?

rikeus ยท 1 points ยท Posted at 01:39:36 on November 22, 2015 ยท (Permalink)

Why does it need to be multiplied by primes though?

Hotal ยท 1 points ยท Posted at 02:50:36 on November 22, 2015 ยท (Permalink)

The only way for a computer to find the prime factors of a number is to try them (up to the square root of the number). So you'd have to check 2, then 3, then 4, etc until you find a number that is a factor of the number you're trying to crack. If your number is prime, you won't have any success until you actually get to the number. So if your number is prime and very large, it's going to take a very long time to find the factors of it.

If it wasn't prime, for example an even number, you find a factor when you get to 2. Now you can divide the number you're trying to get to by 2 and you've cut the time to find the factors of that number in half.

A trivial example - if you use the number 7. You have to try 2 through 7 before you find the only factor (7). So, 6 steps. If you use the number 8, you find a match on your first try with number 2. Now you have 2 and 4. You hit again with your first try at factoring 4 and you've got all of your factors. 2, 2, and 2. And it only took 2 steps. So using a prime made it take twice as many steps to factor. And in this case, factoring the number is the equivalent to unlocking the lock.

Mdayofearth ยท 1 points ยท Posted at 06:04:09 on November 22, 2015 ยท (Permalink)

All locks are pickable, it just depends on how long it takes, and it takes less time to pick a bunch of small simple locks than a more complex one.

Imagine a large prime number being a large complex key to a large lock. The larger the prime, the more complex the key, and lock. All non-prime numbers can be factored into smaller primes (6 = 2 x 3, 16 = 2 x 2 x 2 x 2). If you did not use primes, you're effectively using a bunch of small less complex keys to a bunch of small locks.

NoSuchAg3ncy ยท 1 points ยท Posted at 02:13:48 on November 22, 2015 ยท (Permalink)

Another similar technique would be to convert the message into a string of ASCII characters. Person A adds a random number to each character in the string and sends it to Person B. B adds a random number to each character and sends it back to A. A subtracts their random numbers and sends the result back to B. (At this point, A could also figure out what random numbers B used by subtracting the message.) B subtracts their random numbers and reads the message.

kingfrito_5005 ยท 1 points ยท Posted at 02:49:14 on November 22, 2015 ยท (Permalink)

but that only proves the original statement is false if you both knew what was going on in advance.

zaturama015 ยท 1 points ยท Posted at 02:52:57 on November 22, 2015 ยท (Permalink)

Use thunderstone, my favorite.

poye ยท 1 points ยท Posted at 02:57:51 on November 22, 2015 ยท (Permalink)

And then Eve gets mad and puts a lock too!

[deleted] ยท 1 points ยท Posted at 03:14:25 on November 22, 2015 ยท (Permalink)

[deleted]

fussylizard ยท 1 points ยท Posted at 08:22:45 on November 22, 2015 ยท (Permalink)

In encryption books you normally use Alice and Bob as examples of people trying to send secure messages to each other. Eve is eavesdropping on the conversation, trying to break the encryption. Mallory is someone who is generally just trying to mess things up for all parties. It makes it easy to use those names when describing things since everyone knows the roles.

theasianpianist ยท 1 points ยท Posted at 04:58:58 on November 22, 2015 ยท (Permalink)

So basically whenever I send something encrypted to somebody else it has to come back to me once before they can actually decrypt it? So basically send to person B, send back to me, Sen back to person B who is now able to read the message?

macye ยท 1 points ยท Posted at 11:17:55 on November 22, 2015 ยท (Permalink)

No. But at the start of an encrypted session, you need to have an exchange like this to exchange private encryption keys.

SarahC ยท 1 points ยท Posted at 07:05:54 on November 22, 2015 ยท (Permalink)

Could this be done with XOR?

tdammers ยท 2 points ยท Posted at 19:24:40 on November 22, 2015 ยท (Permalink)

Yes, but only if the key is at least as long as the message. Otherwise, the encryption is going to be quite weak.

volar92 ยท 1 points ยท Posted at 08:30:44 on November 22, 2015 ยท (Permalink)

But how do you exchange the primes without eve finding out?

Hanse00 ยท 3 points ยท Posted at 11:31:44 on November 22, 2015 ยท (Permalink)

You never exchange the primes.

Only I know the primes I used, and the receiver knows the primes they used.

The whole point is, only I lock and unlock my own lock, and you do the same for yours. But because we do it in the way described by /u/UlyssesSKrunk the package is always locked by at least one of us.

Jevel ยท 1 points ยท Posted at 09:35:02 on November 22, 2015 ยท (Permalink)

But how do you prevent Eve from intercepting the box, putting her own lock on it, and sending it back to your friend?

sammichbitch ยท 1 points ยท Posted at 11:14:11 on November 22, 2015 ยท (Permalink)

You mean she can make copies of that locked box and keep one for herself hint hint-NSA does this^, but she cannot open it. She needs a special locksmith who can break the lock and that lock smith has to be the lockmaker/keymaker themselves. hint hint-microsoft and facebook^

chisleu ยท 1 points ยท Posted at 09:59:14 on November 22, 2015 ยท (Permalink)

Is this DH you are describing? It doesn't seem to translate to what I know of symmetric key encryption, but I only made it a few pages into the Schneier book.

SaltySolomon ยท 1 points ยท Posted at 11:28:53 on November 22, 2015 ยท (Permalink)

You forgot the modulo ;)

old_righty ยท 1 points ยท Posted at 14:10:14 on November 22, 2015 ยท (Permalink)

Correct me if I'm wrong, but that is a key exchange method. That would be for 2 random people trying to exchange a key, that they would use for their ensuing communications. Key exchange is slow, but once you have the key the rest is fast. This is also not public/private key encryption.

Symmetric encryption (AES, 3DES) is fast, but both sides have the same key. You need to get that key to someone, use the method above, or PKI for something like SSL/TLS key exchange.

anacrolix ยท 1 points ยท Posted at 16:18:53 on November 22, 2015 ยท (Permalink)

How do I recognise your lock so that I don't take mine off unless your lock is the other?

dmercer ยท 1 points ยท Posted at 17:50:55 on November 23, 2015 ยท (Permalink)

So if your message is x and your bunch of primes is y, then you send xy. Eve sees xy.

My bunch of primes is z, so I send xyz. Eve sees xyz.

You divide by your bunch of primes and send back xz. Eve sees xz.

Now Eve know what y is: xyz / xz.

And she knows what x is: xy / y.

You might have missed a modulo step in there or somethingโ€ฆ

elpea2k ยท 1 points ยท Posted at 18:45:20 on November 23, 2015 ยท (Permalink)

Why primes? Wouldn't any set of numbers work as long as I know mine and you know yours? There is probably a good reason for this, I just can't see it intuitively...

drinkmorecoffee ยท 1 points ยท Posted at 02:11:02 on November 24, 2015 ยท (Permalink)

One question - why primes?

Couldn't each user just multiply by a bunch of numbers, or just one, as they saw fit? Why constrain them to prime numbers, if they're just going to get divided back out anyway?

Allways_Wrong ยท 1 points ยท Posted at 03:01:33 on November 25, 2015 ยท (Permalink)

I don't get this.

Take your message, treat it as a number and multiply it by a bunch of primes. Send it to me.

Let's call this message A.

I will then multiply by a bunch of primes too. I send it back to you.

And this is message B.

You then divide by all of your primes. Send it back to me.

And this is message C.

If Eve can see messages A, B and C then she can open it; C / (B / A) = the original message.

fgiveme ยท 2 points ยท Posted at 10:10:11 on November 25, 2015 ยท (Permalink)*

We don't actually use multiply in encryption because it's a two way function, you can guess the input if you know the output. y = f(x), if you know y and function f() you can guess what the input x is.

What we use is an one way function. In real encryption: function f(), the encryption protocol is publicly available. Message y is also publicly transfered through the internet. But Eve cannot guess the content x inside without knowing the primes you and I used (called private key)


Edit: Depends on the importance of message x we can use a longer private key or a more complex function f(). Eve can try to guess the private keys with brute computational force of a super computer, but it is ineffective to do so (i.e. she spent 20 years to crack an encrypted email between you and I, which could be worthless by that time).

ohyesbitch ยท 1 points ยท Posted at 21:56:18 on March 6, 2016 ยท (Permalink)

Have you heard the puzzle about the river and the chicken and the fox and the bag of feed? You can transport one at a time, how do you get them all over the river without the fox eating the chicken or the chicken eating the feed. https://en.wikipedia.org/wiki/Fox,_goose_and_bag_of_beans_puzzle

So encryption is basically that?

chamington ยท 1 points ยท Posted at 00:33:47 on April 25, 2016 ยท (Permalink)

Sorry I'm very late to the party, but multiplying by a bunch of primes and sending it back and forth and dividing it won't work.

Let's say Person-A wants to send "Eve is a piece of shit". In binary, then converted to decimal, that is 25988999445895780786907149268430967215050227515418996. P-A multiplies it with 3,7, and 101. That would be 55122667824744951049030063598342081463121532560203690516. That is sent to Person-B. P-B multiplies it by 29 and 13. That would be 20781245769928846545484333976574964711596817775196791324532. That is sent to P-A. P-A divides that number by 3,7, and 101. That would be 9797852791102709356663995274198474640073935773312961492. P-A sends that to P-B. P-B gets it and divides it by 29 and 13. That would be 25988999445895780786907149268430967215050227515418996. P-B converts it into binary, then converts it into ascii (there was a problem, but P-B added a 0 to the beginning, assuming it got lost in the multiplications, and it worked.) P-B reads "Eve is a piece of shit", and thinks "yeah, haha"

So Eve wants to know what was sent. She only knows what was sent after it was multiplied by a bunch of primes.

She knows:

55122667824744951049030063598342081463121532560203690516 was the original text, multiplied by primes, sent from P-A to P-B

20781245769928846545484333976574964711596817775196791324532 is the same thing, but multiplied by more primes

9797852791102709356663995274198474640073935773312961492 is the number above, but divided by the primes that P-A multiplied the original text by.

So she knows that 55122667824744951049030063598342081463121532560203690516 times something is 20781245769928846545484333976574964711596817775196791324532. She divides them accordingly. She gets 377. Since she knows the type of encryption P-A and P-B used, she divides 9797852791102709356663995274198474640073935773312961492 by 377. She now has 25988999445895780786907149268430967215050227515418996. She converts it from decimal to binary, (adds a 0 to the beginning because wolfram alpha said 175 digits, and that's not a multiple of 8, and 176 is), and converts it into ascii. She reads "Eve is a piece of shit", and sues P-A and P-B for talking bad about her.

invisiblelemur88 ยท 1 points ยท Posted at 16:14:16 on November 21, 2015 ยท (Permalink)

I never understood RSA until this explanation. THANK YOU.

lummiester ยท 1 points ยท Posted at 19:07:48 on November 21, 2015 ยท (Permalink)

I'm going to go ahead and nitpick a bit - your explanation assumes two things that aren't necessarily true (but probably are):

  1. Hardness of Factorization.
  2. Eve is polynomial.
rawling ยท 1 points ยท Posted at 10:10:52 on November 22, 2015 ยท (Permalink)

No, it assumes that Eve can't perform two integer divisions.

TotesMessenger ยท 1 points ยท Posted at 14:29:50 on November 21, 2015 ยท (Permalink)*
kilroyshere ยท 1 points ยท Posted at 18:00:47 on November 21, 2015 ยท (Permalink)

Best description I've ever read.

adacmswtf1 ยท 1 points ยท Posted at 18:19:29 on November 21, 2015 ยท (Permalink)

For the even more ELI5 version.

https://youtu.be/MsqqpO9R5Hc?t=177

(also watch the whole series, its good.)

Jmrwacko ยท 1 points ยท Posted at 18:46:54 on November 21, 2015 ยท (Permalink)*

For anyone who's confused about why we need a lock on every step, it's because we don't want to share our keys, since sharing keys would defeat the purpose of encryption. Sure, I could lock the box then send it to you with the key, but then you might turn around and sell that key to someone else, or the key could be intercepted and used to unlock all my other boxes. Through encryption, everyone keeps their own keys to themselves -- only the locks are shared with others. It ensures complete security. The only way a third party (hacker, spook, etc.) can decrypt the file -- open the lock -- is by stealing the sender's or the recipient's key.

Metallkasten ยท 1 points ยท Posted at 18:57:02 on November 21, 2015 ยท (Permalink)

This is beautifully simple.

seiyria ยท 1 points ยท Posted at 19:40:44 on November 21, 2015 ยท (Permalink)

This explanation of encryption is literally perfect and would help simplify explaining it to other people. Thanks!

lenswipe ยท 1 points ยท Posted at 22:01:36 on November 21, 2015 ยท (Permalink)*

Fucking Eve! Why is she ALWAYS snooping on shit?!

unclefire ยท 2 points ยท Posted at 22:54:27 on November 21, 2015 ยท (Permalink)

Really, she should be like Janet in Accounting who doesn't give a fuck.

:-)

lenswipe ยท 1 points ยท Posted at 23:02:18 on November 21, 2015 ยท (Permalink)

Nah, she's Carol now.

[deleted] ยท 1 points ยท Posted at 05:41:06 on November 22, 2015 ยท (Permalink)

What are you talking about? There is no Carol in HR

lenswipe ยท 1 points ยท Posted at 13:26:14 on November 22, 2015 ยท (Permalink)

Ah, did she move to accounting?

SoundChaser83 ยท 1 points ยท Posted at 06:41:28 on November 22, 2015 ยท (Permalink)

Disclaimer: I'm a philosopher, not a mathematician, so please let me know if I make some elementary error.

Perhaps I'm the debbie-downer of the group, but I don't understand why this mathematical counter-example to the claim is especially significant as there are ample non-mathematical counter-examples. I read some other comments and (if I understand correctly) apparently cryptography by way of multiplying prime numbers and such is especially hard to crack, but that's beside the point. To show that the original claim is false, we need only show that there's at least one case in which Eve listens to everything persons A and B say to each other and yet some piece of information slips by Eve. Given that that's the case, I don't see what's so special about the mathematical counter-example other than the fact that it's an especially quirky way among many to prove the claim wrong.

A lot is going to hinge on how we understand the details of original claim. What does it mean for Eve to "listen to absolutely everything" between A and B? Would it be a genuine counter-example if A and B pass notes to each other and Eve doesn't have her reading glasses? If Eve is cognitively impaired (perhaps she had one too many sips of vodka) such that she hears the sentences that A and B say to each other but she lacks the ability to process the meaning of those sentences? If A and B are psychic? Surely we should interpret the original claim in a much more robust fashion...something like this: If Eve has fully functioning cognitive capacities and has full access to all language deployed between A and B, there is no information that A can transmit to B through language (or vice versa) that Eve will not be in a position to accurately interpret. There are still some questions to be asked about this reading of the original claim, but I think it's good enough for our purposes.

This revised interpretation of the claim is still open to non-mathematical counter-examples. Suppose A and B are raised separately in a secret cult in which children are taught by their elders to speak only in a language we can call "Schminglish". When A and B finally meet and start communicating, they speak only in Schminglish to each other for the rest of their friendship. Even though Eve has access to everything A and B will ever say to each other, it will sound like gibberish to her. Eve knows nothing about Schminglish and nothing about the secret cult A and B grew up in.

This appears to be a genuine counter-example to the original claim, but it's not as if we needed math in order to show that the claim was false. I'm not sure if it was ever meant to be implied that math was necessary to show that the claim was false, but I just thought it was worth running by you just in case. We could even show that the claim is false using math (perhaps interpreted loosely) without resorting to prime numbers at all. Imagine Schminglish in my example above is nothing more than a mapping of integers to meaningful thoughts (e.g. "238" means "I miss my chicken and cheese breadsticks", or "524,710" means "I'm going to be late for school today"). Eve could be just as in the dark about the information A and B are trying to convey to each other in this case as well, and there's no need to resort to prime number multiplication.

ErroneousFunk ยท 2 points ยท Posted at 07:30:09 on November 22, 2015 ยท (Permalink)

Sure, if you're speaking a secret language that Eve doesn't understand, then obviously, the statement is false. Don't need four paragraphs to explain that one!

However, since you asked to be corrected if you make an error...

Let's say you're sending an encrypted email, and watching some encrypted video files, and generally sending encrypted information back and forth between your computer and remote servers each day. Your computer is not going to bother with things like "Oh, this looks like an email! I'll just translate it to Schminglish real quick, and send it to this other server that understands Schminglish when no one else does!" That would be crazy. Schminglish translates emails, but might not work with video files or audio. More importantly: Schminglish is required to be understood by one other server, and one other server only, which means that your computer would be required to "know" an entirely different language for each server it talks to AND each server would be required to know an entirely different language for each computer it talks to! As you can imagine, the number of languages would grow astronomically huge. This is actually a well-studied situation in cryptography, and is most often quickly dismissed as impractical :)

So how can you communicate with another remote server, secretly, even when it understands the language but is unable to understand the message? That's what this whole "prime numbers thing" solves.

Hanse00 ยท 1 points ยท Posted at 11:34:29 on November 22, 2015 ยท (Permalink)

Sure, if you're speaking a secret language that Eve doesn't understand, then obviously, the statement is false. Don't need four paragraphs to explain that one!

That doesn't actually work. For us to speak a secret language, we need to first communicate to each other the rules of the language we will speak.

Since Eve is overhearing EVERTHING we say, she'd know the language too.

ErroneousFunk ยท 1 points ยท Posted at 15:35:10 on November 22, 2015 ยท (Permalink)

It does work though! That's the whole point. I can have never talked to you before, Eve is listening to absolutely everything we've ever said to each other in our lives (no opportunity to agree on secret syntaxes), and I send you an encrypted message. Eve knows how the messages are constructed, but cannot decode it. You receive it, and cannot decode it, but you can modify it and send it back to me. That's how this whole thing works. Eve knows the rules, but she's still cut out of communication. If she tries to intercept and modify the message, I can prove, through publicly available information, that she modified it, and not you .

Here's how it works. Numbers composed of two very large primes are really hard to factor. Sure, anyone can say that 21 is composed of "7 and "3" But let's say you have 181521713. That happens to have exactly two prime factors: 13469 and 13477. However, it's not possible to know that unless you run it through all the primes up to 13469. Now let's say you have some 77-digit long prime number. That would take a supercomputer about 650 million years to crack (according to one calculation I found), but they are easy enough to generate.

So, let's use small primes for the sake of simplicity, and say your super secret prime number is "3" and the public prime number that you send around to everyone (including Eve) is "5" Let's say the message you want to send is "2" (in reality, messages are very long numbers -- strings of 1's and 0's -- containing information, but we're sending a very short message today). So you take the message "2" and multiply it by 3*5 (that is, your super secret number times your public number) and send over a "30"

Eve has no idea what the number actually means. So now your friend receives it, divides the 30 by your public key, "5" to get 6. He still has no idea what it means, but multiplies his public and private key to it. Let's say his public key is 7, and his super secret key is 11, so he sends back 6 * 7 * 11 = 462.

Eve has no idea what this 462 means, because she simply doesn't have the ability to factor these "huge" primes.

You receive 462, and now the magic happens! You take 462 and divide it by your friend's public key (7) and your super secret key (3), and send your friend back a "22" Now, the message is simply encoded with his super secret private key, and nothing else. You don't know what the private key is, even, but you just do the math and send it on, knowing that it contains your original message in there, and he'll be able to read it.

Eve still has no idea what the 22 means, because she doesn't know that the secret key is "11" and the message is "2" -- it's just a random encrypted string to her.

Your friend receives the 22, easily divide by his secret key "11" and says "Oh! The message is 2!"

Now, the cool thing is, this message could actually be instructions to encrypt all future communication between you and the friend. Eve can never read these instructions, but once they're known by you and your friend, you can send messages without having to do all the "back and forth," and, in this sense, have sort of established a "secret language," but you've established it having NEVER MET BEFORE, and with Eve watching all your communication.

Now, the real problem is: What if Eve modifies this message en-route, and adds her own keys to the message and sends it back to you, posing as your friend? (Of course, she'll also send the message along, unmodified to your friend, and intercept communication from his side as well) This is called a "man in the middle" attack, and is a problem. This is where the security certificates and stuff come in. It's important to be able to establish who's public key is whose. You can prove that a message was encrypted with the public/private key of your friend, and could not have been encrypted by anyone else who did not know that private key (the explanation here is a little long, and I've already written a lot, but if you follow the transaction above closely, you can probably see why) -- as long as you KNOW that the public key actually really truly does belong to your friend. If you get a public key from Eve and Eve says "this is totally your friend's public key, trust me" well then -- the whole thing doesn't work.

So you look up these public keys in third-party "phonebooks" that you trust, or someone might post their public key on their website, or might even give it to you in person. The cool thing is, you only have to trust that this key is theirs, and that's it. You can even have a huge official agency that says "this set of keys belong to these people that we trust" and each one of those people says "these set of keys belong to these people that we trust," and then that network grows larger and larger, and, as long as each person in the chain trusts everyone else in the chain, it works. That's how we get SSL certificates and such.

But anyway. That's how it works. Eve can overhear ABSOLUTELY everything, and still not be able to crack the message.

Hanse00 ยท 1 points ยท Posted at 16:08:42 on November 22, 2015 ยท (Permalink)

Touchรฉ, my coffee must not have kicked in yet when I wrote that.

DJGandalf ยท 1 points ยท Posted at 11:08:50 on November 22, 2015 ยท (Permalink)

Imo this wouldn't be false because there would be identifiable pattern in the communication, in your example "238" means "I miss my chicken and cheese breadsticks", or "524,710" means "I'm going to be late for school today. So wherever those numbers appear in the structure of the communication they will mean the same thing. This is because Both A and B have learned a language and and understand the structure.

Encryption attempts to remove all structure and render the message indecipherable, using your example "238" means "I miss my chicken and cheese breadsticks" but later In the paragraph "238" could mean "I'm going to be late for school today" making it exponentially harder to decipher. The keys determine the meaning based on position, when the keys reach a certain size you effectively making the task impossible with today's computing powers. The keys define what the numbers mean relative to there position.

The point is that is not the language that makes the communication secure but the process. The locking the box analogy, both A an B both understand that they need to apply there own lock and send it back in order for the other partner to unlock the box. In your example the box had a combination lock and both A and B have been told the code. It possible for the person in the middle to guess the code. But with the locks the need to have both keys. The only thing A and B need to know is the process, and even with the process being publicly available, the person in the middle still can't unlock the box.

stanleyfarnsworth ยท 0 points ยท Posted at 19:58:20 on November 21, 2015 ยท (Permalink)

Holy shit. I... I now understand cryptography!

kspacey ยท 0 points ยท Posted at 18:35:19 on November 21, 2015 ยท (Permalink)

But if it's an easily invertible series of operations then she can still deduce the message. I'm sure there's something else but this isn't a good example.

Natanael_L ยท 1 points ยท Posted at 19:34:55 on November 21, 2015 ยท (Permalink)

Commutative operations is needed when you do three passes like here, just as others said. Except they must be hard in one direction, which multiplication isn't. ECC operations for example are hard in the reverse direction.

kspacey ยท 1 points ยท Posted at 21:34:39 on November 21, 2015 ยท (Permalink)

Hard is not the same as impossible. This doesn't prove that the information can be passed back and forth without an outside observer having all the information he/she needs to crack the code, just that it would take time.

Natanael_L ยท 1 points ยท Posted at 22:08:55 on November 21, 2015 ยท (Permalink)

Only OTP is proven impossible so far, but we have good reason to believe ECC won't fall soon. There's physical limits to how much energy you need to run enough computations

kspacey ยท 1 points ยท Posted at 00:17:15 on November 22, 2015 ยท (Permalink)

Again, hard is very different from impossible and it very much doesn't fit the concept of this thread to say something is 'hard enough'

gagnonca ยท -3 points ยท Posted at 22:09:53 on November 21, 2015 ยท (Permalink)

Not sure why you're being upvoted... Diffie-Hellman uses modular arithmetic, not division. So you basically just did a bad job of explaining a common crypto protocol.

[deleted] ยท 0 points ยท Posted at 07:10:27 on November 21, 2015 ยท (Permalink)

[deleted]

Meliorus ยท 11 points ยท Posted at 07:16:29 on November 21, 2015 ยท (Permalink)

You never exchange keys in this scheme

GbZeKamikaze ยท 0 points ยท Posted at 17:48:15 on November 21, 2015 ยท (Permalink)

Thank you very much. My knowledge of "security and encryption" extends little further than hashed passwords in a database, but this suddenly makes everything so clear. (I mean that should help me understand harder notions later...)

dsquard ยท 0 points ยท Posted at 19:16:56 on November 21, 2015 ยท (Permalink)

Why do you have to use prime numbers?

miroku000 ยท 2 points ยท Posted at 20:01:13 on November 21, 2015 ยท (Permalink)

Well, as Eve, if I could guess what numbers to divide the encrypted message by to get the orginal, then I can decrypt the message. If the numbers are prime, then there are only two possible divisors that would work. If the numbers are not prime then there are more possible answers that will also give the correct result.

SkepticalOfOthers ยท 2 points ยท Posted at 20:37:46 on November 21, 2015 ยท (Permalink)

basically nothing. The protocol he described isn't a good one, as others have pointed out.

One of the simpler algorithms that's actualy used is called the diffie-hellman key exchange. Let's say you and I want to agree on a key or password that we can use to encrypt messages we send to each other.

We start by agreeing on a prime number, that we'll call p, and a special number g, that we call a "generator". next, you and I both pick a random number (a and b respectively) between 1 and p-1. We keep those secret

Next, I take the number ga, divide it by p, and take the remainder (this is called taking ga mod p). You do the same, with gb, and then we exchange ga and gb.

So now I take the gb you sent me, and do (ga )b mod p, and at the end I'm left with gab mod p, and you have gba mod p, and it turns out these two things are equal. So we now have a number between 1 and p-1 that we can use as a password.

Here's why this works. if you have numbers (ga mod p) and (gb mod p), it turns out to be really hard (we think) to compute (gab mod P). This is called the diffie-hellman problem. But, if you have b, and ga mod p, then it's trivially easy, so Bob and Alice can both find the secret key, but Eve cannot.

The reason p has to be prime in this case, is because if p is composite, and Eve can factor p, then it is actually easy to solve this problem.

Natanael_L ยท 1 points ยท Posted at 19:36:40 on November 21, 2015 ยท (Permalink)

RSA needs it. Other algorithms like ECC don't.

dsquard ยท 1 points ยท Posted at 19:40:43 on November 21, 2015 ยท (Permalink)

Let's pretend I don't know what either of those things are...

I'm seriously a rube when it comes to math.

Natanael_L ยท -1 points ยท Posted at 19:43:16 on November 21, 2015 ยท (Permalink)*

Start on Wikipedia, then feel free to check out /r/crypto

Edit: really, downvotes?

Bleeurg ยท 0 points ยท Posted at 21:04:45 on November 21, 2015 ยท (Permalink)

I teach RSA in my discrete math class and I love your analogy.

rawling ยท 1 points ยท Posted at 10:14:20 on November 22, 2015 ยท (Permalink)

Oh God, you actually teach encryption and you can't tell that this scheme is both completely broken and nothing to do with RSA?

liangauge ยท 0 points ยท Posted at 21:33:35 on November 21, 2015 ยท (Permalink)

technically encryption does not prevent the man in the middle from working out the secrets told.

It just makes it extremely difficult to work out the actual message.

veritasserum ยท 0 points ยท Posted at 21:52:31 on November 21, 2015 ยท (Permalink)

For PKI, wouldn't a better analogy be:

I publish plans for a simple to make safebox. The safebox has two locks: One that can only be used to lock it. The other can only be used to unlock it. I also publish the exact dimensions of the first key - the one that locks it. However, I and I alone have the key for the second lock - the one that unlocks it.

Now then, anyone that wants to send me something that no one else can see, only has to build the simple safebox and make a locking key. At that point, the only key in the world that can unlock it is the one I have.

Both the many locking keys and my single unlocking key can be used over and over again for an unlimited number of newly built safeboxes.

The system is secure so long as no one steals my unlocking key, or manages to guess/figure out what its exact shape is.

securitywyrm ยท 0 points ยท Posted at 22:02:11 on November 21, 2015 ยท (Permalink)

I've tried to understand how encryption works before, and this finally made it make sense. Thank you!

Now if we can only get this message to our political leaders so they can understand what encryption is other than "evil secrecy."

BABarracus ยท 0 points ยท Posted at 02:07:52 on November 22, 2015 ยท (Permalink)

Couldn't you do this with differential equation and have it use several equations and at the end of the message the computer will say which equation its switching to and the only way to know is to have a copy of the equations are.

gagnonca ยท -2 points ยท Posted at 19:59:39 on November 21, 2015 ยท (Permalink)

Just google Diffie-Hellman

DoWhile ยท 19 points ยท Posted at 03:09:32 on November 21, 2015 ยท (Permalink)

In the classical, computational-cryptography setting, use key exchange

In the classical, random oracle model without crypto, use merkle puzzles to obtain a quadratic advantage against Eve (which is optimal).

In the quantum world, you can use quantum key exchange and get unconditional security as long as certain quantum physics holds.

OperaSona ยท 5 points ยท Posted at 16:02:06 on November 21, 2015 ยท (Permalink)

Also worth knowing, in the Information Theory world, you have the Wiretap Channel. It works this way:

  • Alice has a noisy communication channel to Bob, for instance with Channel Capacity C.

  • When it's used, Eve receives the message through a noisier channel, with capacity C' < C.

  • Alice and Bob can design a protocol so that they can exchange data reliably at a rate which, in the simplest scenario (where Eve has access to what's called a "degraded version" of the channel from Alice to Bob), can be arbitrarily close to C-C', without letting Eve receive more than an arbitrarily low rate of information, and all of that even if Eve has perfect knowledge of the communication scheme that Alice and Bob are using, including potential keys etc.

Arxiv link to a beautiful paper that achieves this result: http://arxiv.org/abs/1001.0210 (Vardy is a pretty famous guy in the Coding Theory community).

[deleted] ยท 5 points ยท Posted at 14:02:55 on November 21, 2015 ยท (Permalink)

RSA Encryption friend.

Alice (A) Bob (B) Eve (E) Mallory (M)

Are the placeholder names used when teaching this typically.

bradten ยท 2 points ยท Posted at 22:22:36 on November 21, 2015 ยท (Permalink)

Coming out of the lurk for this. /u/UlyssesSKrunk got this right, but let me formalize it a bit for the actual mathematicians out there.

If Alice wants to communicate with Bob, she's more than happy to tell Eve a few things, like the modular group in which she wants to communicate, and a constant g. In fact, Alice is just going to blast that out there for everyone to see.

Now, Alice is going to pick a constant, g, raised to some random prime number in the modular group that we'll call a. So her message is ga.

Bob will receive that and reply with his own random prime number, b, so his message says gb.

Since (ga)b = (gb)a, Bob and Alice can communicate using gab as their key from now on. This can be a symmetrical or asymmetrical key. All the while, Eve, who has heard all of this, is none the wiser. This is because of the discrete logarithm problem, which is widely accepted as being intractable.

[deleted] ยท 1 points ยท Posted at 06:17:10 on November 21, 2015 ยท (Permalink)*

[deleted]

UncleMeat ยท 2 points ยท Posted at 16:02:08 on November 21, 2015 ยท (Permalink)

Not the only asymmetric crypto scheme. Its actually slowly falling out of favor.

hotoatmeal ยท 1 points ยท Posted at 17:12:56 on November 21, 2015 ยท (Permalink)
newyorkzxtc ยท 1 points ยท Posted at 19:42:18 on November 21, 2015 ยท (Permalink)

Save on shipping. Send me your lock, ill attach it to the box and return it.

xereeto ยท 1 points ยท Posted at 22:55:43 on November 21, 2015 ยท (Permalink)

You write them - she's only listening to you, duh. (/s)

fr3ddie ยท 1 points ยท Posted at 12:23:22 on November 22, 2015 ยท (Permalink)

Eve listens with her ears to everything the friends say... but they can just write something down on paper... it doesnt say that Eve is also watching everything they do. if eve WAS watching everything they were doing, wouldnt she then see the discussion of the keys and such?

p00b ยท -6 points ยท Posted at 13:42:08 on November 21, 2015 ยท (Permalink)

To play semantics here, which I suppose is the point, the statement is not precise enough.

Eve is only listening. Since it's stated that she's a girl, there are limits on her listening capabilities - for example she can't map a room with sonar or hear you blinking or what you're writing.

Any non-auditory agreements you make with your friend (e.g. writing down a standard of signals or ciphers to communicate though, then using that for all secrets going forward) will not be able to be observed by Eve.

bairedota ยท 29 points ยท Posted at 03:02:26 on November 21, 2015 ยท (Permalink)

Not too knowledgeable on cryptography, is this still true if Eve has infinite processing power?

Lopsidation ยท 68 points ยท Posted at 03:19:52 on November 21, 2015 ยท (Permalink)*

Nope, it depends on Eve having limited power.

Unless, as DoWhile says, you can use quantum physics. Then you can arrange a protocol whereby you can send your friend a secret one-time pad using quantum bits of information ('qubits'). While you can't stop Eve from intercepting the pad on the way, you can measure the qubits in a certain way to figure out whether or not she did. (This has to do with quantum physics weirdness, where observing a system changes it. You arrange your qubits so that in order for Eve to observe them, she has to change them in a way you can detect.)

If you detect that Eve didn't intercept your one-time pad, then you can use it to absolutely securely encrypt a message. If Eve has infinite processing power and decides to intercept all your qubits... well, you're out of luck.

Snoron ยท 17 points ยท Posted at 03:42:58 on November 21, 2015 ยท (Permalink)

If you detect that Eve didn't intercept your one-time pad

...

If a girl called Eve listens to absolutely everything you and your friend say to each other

While what you say is true, doesn't it go against the original statement that is being discussed?

UncleMeat ยท 6 points ยท Posted at 16:03:26 on November 21, 2015 ยท (Permalink)

The original statement was discussing asymmetric crypto, which does not rely on any secret exchange. OTP doesn't have this nice property.

AntiquatedReality ยท 2 points ยท Posted at 14:04:58 on November 21, 2015 ยท (Permalink)

Allright, you got me curious and this is going to be a dumb question but if Eve's observation could flip the qubits state by observing it, what's keeping your own observation from doing the same thing and creating a false positive(flipping the same one as eve)? While I know the extremely basic concepts behind some of the things related to quantum theory, I don't see a viable safeguard from such a thing. Would your own trigger a different qubit than the one Eve's observation would flip?

How does the superposition of qubits come into play, if they are all both 1 and 0 until observed(wouldn't knowing the state during transit and prior ruin the whole point, if not how so?) and have we demonstrated this as a proof of concept beyond calculations and working to those ends (a la Higgs boson and the standard model's predictions)?

GaryTheKrampus ยท 8 points ยท Posted at 06:07:45 on November 21, 2015 ยท (Permalink)

If Eve has arbitrarily large processing capability, then classical cryptography still holds. For infinite processing power it breaks down.

Meliorus ยท 5 points ยท Posted at 07:18:42 on November 21, 2015 ยท (Permalink)

What scheme holds up to arbitrarily large processing power if yours is fixed?

wintermute93 ยท 2 points ยท Posted at 18:52:30 on November 22, 2015 ยท (Permalink)

I think he meant "for any instance of Eve with fixed computing power, there are instances of this scheme she cannot break".

GaryTheKrampus ยท 2 points ยท Posted at 22:01:03 on November 22, 2015 ยท (Permalink)

Ah, yeah, to expand on that other explanation...

If your adversary (Eve) has arbitrarily great computing capability, you may simply use sufficiently large primes in your classical crypto scheme. Sufficiently large meaning "large enough to retain any arbitrary level of security you desire."

How exactly you get those sufficiently large primes is another question entirely, and certainly relies on your processing power not being fixed. Actually, for practical public-key crypto, we use Fermat pseudoprimes, which can be generated relatively quickly. However, the amount of computation you have to do to use this scheme grows much more slowly than the amount of computation your adversary needs to break it. That's the fundamental idea of practical cryptography -- though exactly how much is "much more slowly" depends on the scheme.

Now, if your adversary has infinite capability, traditional cryptosystems do not hold -- even if you, too, have infinite capability.

wei2912 ยท 1 points ยท Posted at 14:54:21 on November 21, 2015 ยท (Permalink)

Nope. Some other encryption schemes are perfectly secure though, like the one time pad, though they're not used for practical reasons.

grelphy ยท 1 points ยท Posted at 20:22:33 on November 23, 2015 ยท (Permalink)

I'm late to the party, but I'll note that for practical purposes, the upper limit on processing power before you've exhausted all of the energy in the universe isโ€”from a cryptographic brute-force perspective, anywayโ€”pretty low. You can't even count to 2256, much less do actual cryptography on all those values.

jfb1337 ยท 55 points ยท Posted at 11:18:23 on November 21, 2015 ยท (Permalink)

Does it work with people who aren't called Eve?

octatoan ยท 92 points ยท Posted at 15:05:33 on November 21, 2015 ยท (Permalink)

We leave the obvious generalizations to the reader.

-Israel Herstein

[deleted] ยท 29 points ยท Posted at 16:25:09 on November 21, 2015 ยท (Permalink)

We have proof (assuming one-way functions or similar) that it works in the case you and your friend are called Alice and Bob resp., and the listener is called Eve.

It's a longstanding open question in cryptography whether this protocol can be extended to other first names.

teawreckshero ยท 2 points ยท Posted at 19:38:21 on November 21, 2015 ยท (Permalink)

Actually, no. That's why we still find security vulnerabilities all the time. If people would stop making up names like Kyllle and Skylee and Zhaden we could attain perfect security, but alas...

-THE_BIG_BOSS- ยท 6 points ยท Posted at 14:48:01 on November 21, 2015 ยท (Permalink)

And does it work if you're talking to someone other than a friend?

[deleted] ยท 21 points ยท Posted at 18:18:00 on November 21, 2015 ยท (Permalink)*

[deleted]

ztxi ยท 14 points ยท Posted at 19:56:30 on November 21, 2015 ยท (Permalink)

(1650*1820)/300300=10

I_play_elin ยท 7 points ยท Posted at 18:39:28 on November 21, 2015 ยท (Permalink)

Why couldn't you use non-prime numbers?

QuiteRefreshing ยท 6 points ยท Posted at 19:22:21 on November 21, 2015 ยท (Permalink)

There's a StackOverflow post that you might find interesting.

I_play_elin ยท 1 points ยท Posted at 19:24:02 on November 21, 2015 ยท (Permalink)

Thank you.

rawling ยท 1 points ยท Posted at 11:33:28 on November 22, 2015 ยท (Permalink)

... that has no bearing on this example at all.

IAmVeryStupid ยท 0 points ยท Posted at 12:18:53 on November 22, 2015 ยท (Permalink)

Yes it does?

rawling ยท 1 points ยท Posted at 12:56:48 on November 22, 2015 ยท (Permalink)

This example doesn't require you to factor the multiplied primes. The keys could be made of multiplied large primes, or multiplied small primes, or just be prime themselves.

IAmVeryStupid ยท 1 points ยท Posted at 04:26:08 on November 23, 2015 ยท (Permalink)

It would require Eve to factor the multiplied primes to intercept the code.

rawling ยท 1 points ยท Posted at 11:55:44 on November 23, 2015 ยท (Permalink)

It really doesn't. All she needs to do is divide the backwards message by one forwards message, and then divide the other forwards message by the result. The key terms are removed regardless of what primes were used.

Apps4Life ยท 2 points ยท Posted at 21:23:01 on November 21, 2015 ยท (Permalink)

Primes can't be divisible by anything else. So a VERY big prime is only divisible by itself. Whereas a very big regular number (composite) could be divisible by tons of smaller (easier to iterate) numbers over and over.

I_play_elin ยท 1 points ยท Posted at 22:01:15 on November 21, 2015 ยท (Permalink)

Good explanation. Thanks.

Apps4Life ยท 1 points ยท Posted at 01:12:39 on November 22, 2015 ยท (Permalink)

I re-read it and realized I slightly left out why that fact about primes is important haha. It's easy for a computer program to iterate tons of small numbers (called the "prime factors" of any composite) millions of times a second. Very large primes are HUGE numbers that can't be broken into smaller pieces so it takes computers a VERY long time to iterate through.

CaesarTheFirst1 ยท 1 points ยท Posted at 19:45:55 on November 21, 2015 ยท (Permalink)

The main thing is you don't want it to be divisible by small primes (op meant large primes) because it's very easy to decompose (you try dividing by 1, then 2, and boom it's divisible by 3, so you divide by 3 and repeat from the start).

However this isn't how actually the algorithm works (the idea is, not the multiplying by primes, here is a link: https://en.wikipedia.org/wiki/Diffie%E2%80%93Hellman_key_exchange#Description)

SkepticalOfOthers ยท 3 points ยท Posted at 20:47:11 on November 21, 2015 ยท (Permalink)

the larger the primes the harder and more impossible it becomes.

No, it's exactly as easy as it is to encrypt in the first place, you just need to divide.

Lopsidation ยท 7 points ยท Posted at 20:43:52 on November 21, 2015 ยท (Permalink)

This is insecure. Eve can take the gcd of all the numbers sen to recover the original message. Or just compute 1820*1650/300300 to get the original message 10.

Ukrainian_Reaper ยท 2 points ยท Posted at 00:06:45 on November 22, 2015 ยท (Permalink)

Yes except using 4884940623 and 2198800+1 wouldn't have made for a very understandable example.

rawling ยท 2 points ยท Posted at 11:34:10 on November 22, 2015 ยท (Permalink)

However big your primes are, you only have to perform two divisions.

It's not the size of the numbers that matters; OC is using them wrong.

veritasserum ยท 2 points ยท Posted at 20:06:30 on November 21, 2015 ยท (Permalink)

Eve is a lousy friend. You should ditch her.

Meliorus ยท 1 points ยท Posted at 23:18:10 on November 22, 2015 ยท (Permalink)

Eve is not your friend.

higgs_bosoms ยท 2 points ยท Posted at 22:10:34 on November 21, 2015 ยท (Permalink)

eve for eavesdropper? nice

NoSuchAg3ncy ยท 2 points ยท Posted at 02:07:11 on November 22, 2015 ยท (Permalink)

Her last name is Dropper. Her siblings are Name and Eye.

FinFihlman ยท 2 points ยท Posted at 18:21:34 on November 22, 2015 ยท (Permalink)

The key here is that Eve is not actively transmitting your messages, merely listening to them. If she was then a mitm is simple, which is sadly the case.

MadTux ยท 2 points ยท Posted at 13:19:42 on November 21, 2015 ยท (Permalink)

Unless you agree upon a random key beforehand, Eve can always decrypt everything. It might take her a while, but even RSA, etc. aren't perfect.

CaesarTheFirst1 ยท 3 points ยท Posted at 19:09:47 on November 21, 2015 ยท (Permalink)

It's assumed Eve has a limited computing power (if it takes her 600 years to decrypt the message, then it's okay. You can read the actual algorithm here: https://en.wikipedia.org/wiki/Diffie%E2%80%93Hellman_key_exchange#Description It's brilliant.

NoiseMarine ยท 1 points ยท Posted at 20:11:40 on November 21, 2015 ยท (Permalink)

Yeah but at some point Eve has to decide whether what she encrypted is the actual message or just happens to be a decryptable interpretation of the message.

MadTux ยท 1 points ยท Posted at 21:28:49 on November 21, 2015 ยท (Permalink)

I thought that's only relevant for things like the one time pad, not RSA.

NoiseMarine ยท 1 points ยท Posted at 21:38:27 on November 21, 2015 ยท (Permalink)

Yeah I think your right after some research I was thinking of stuff like encryption systems that use a 128-bit random number for the key attached to an account. It also really depends upon what you are using encryption with.

Shakemyears ยท 1 points ยท Posted at 00:46:11 on November 22, 2015 ยท (Permalink)

But why the name "Eve"?

epsilon_negative ยท 59 points ยท Posted at 03:40:30 on November 21, 2015 ยท (Permalink)

Any open set in R containing Q must be all of R, up to a countable complement.

StevenXC ยท 20 points ยท Posted at 05:15:20 on November 21, 2015 ยท (Permalink)

Hint: cover q_n with an open set of size 1/2n.

No1TaylorSwiftFan ยท 7 points ยท Posted at 07:58:26 on November 21, 2015 ยท (Permalink)

Similar to how one shows that Q has Lebesgue measure 0. Or any countable set for that matter.

jimeoptimusprime ยท 6 points ยท Posted at 09:00:40 on November 21, 2015 ยท (Permalink)

Well, to show that, one could also view any countable set as a countable union of singletons and use coubtable additivity.

proto-n ยท 4 points ยท Posted at 10:54:56 on November 21, 2015 ยท (Permalink)*

How do you prove that this does not cover R?

edit: nvm the area is 1

ChrisLomont ยท 4 points ยท Posted at 16:58:09 on November 21, 2015 ยท (Permalink)

Area is < 1. The open sets are not disjoint.

evyllgnome ยท 1 points ยท Posted at 09:40:22 on November 22, 2015 ยท (Permalink)

I see. So we do not count the empty set as a countable set? Can one even call the empty set finite?

epsilon_negative ยท 2 points ยท Posted at 16:17:38 on November 22, 2015 ยท (Permalink)

The empty set is finite, since it has finite cardinality (0); depending on the convention, it's considered either countable or "at most countable". I'm a little confused, how does this relate to the claim about open sets containing Q?

evyllgnome ยท 1 points ยท Posted at 17:40:41 on November 22, 2015 ยท (Permalink)*

I might just be really slow on the up take today. But I think your claim should be true then, no?

Since Q is dense in R, any open set O in R with Q \subseteq O \subseteq R should already be R.

Edit: Yeah. Okay. Again, today is a slow day. Sorry -.- .

epsilon_negative ยท 2 points ยท Posted at 20:31:30 on November 22, 2015 ยท (Permalink)

No worries! StevenXC's construction is the typical counterexample, and it is a pretty strange set.

writesgud ยท -1 points ยท Posted at 23:54:55 on November 21, 2015 ยท (Permalink)

wtf did you just say? was it essentially: all of Q is in R, thererfore all Q is also R?

epsilon_negative ยท 1 points ยท Posted at 21:06:19 on November 22, 2015 ยท (Permalink)

No, the (incorrect) claim is that any open set which contains all rationals is R minus a countable set.

writesgud ยท 1 points ยท Posted at 18:34:52 on November 23, 2015 ยท (Permalink)

Apologies, I am (clearly) not a math person, and did not understand the terms used. is an ELI5 version possible?

epsilon_negative ยท 3 points ยท Posted at 18:47:31 on November 23, 2015 ยท (Permalink)

Let U be any set of real numbers. Any point x in U is called an interior point if U contains some open interval centered at x. For example, the point 0.9 is an interior point of the set [0, 1) since the interval (0.85, 0.95) centered at 0.9 is entirely contained in [0, 1). However, 0 is not an interior point of [0, 1) since no open interval centered at 0 lies entirely within [0, 1) (any such interval contains points less than 0).

A set is called open if all of its elements are interior points. Thus, if U is an open set in R, you can take any point in U and "wiggle around" a bit (i.e. come up with a small open interval about that point) while remaining in U. For example, open intervals are open, but half-open and closed intervals are not, because their endpoints have no "wiggle room".

The incorrect claim is: if a set U is open in R and contains every rational number, it must be (more or less) all of R. (This seems intuitive since the rationals are dense in R.)

Valvino ยท 76 points ยท Posted at 01:47:00 on November 21, 2015 ยท (Permalink)
Shadonra ยท 108 points ยท Posted at 01:33:56 on November 21, 2015 ยท (Permalink)

The additive groups of R and of C are not isomorphic.

houston_eulers ยท 50 points ยท Posted at 01:43:48 on November 21, 2015 ยท (Permalink)*

Maybe I'm alone in this, but that never seemed intuitively obvious to me at all...I mean C under addition is just R2

Edit: Holy craps I'm an idiot. R and C are isomorphic? How did I never learn this?

Lopsidation ยท 28 points ยท Posted at 02:31:09 on November 21, 2015 ยท (Permalink)

Note that just because the additive groups of R and C are isomorphic, doesn't mean that R is isomorphic to C. They aren't isomorphic as fields, because C has a solution to x2+1=0 and R doesn't.

MyOverflow ยท 61 points ยท Posted at 01:59:07 on November 21, 2015 ยท (Permalink)

He means the statement that they aren't isomorphic is false (unless that's what you also mean).

For a proof, we use the Axiom of Choice.

We can make R and C into Q-vector spaces with the obvious addition and scalar multiplication rules. Given a basis of R over Q (which exists by AC), call it B, we can make a basis of C over Q by B x B. Since R is uncountably infinite, this makes B infinite, as well, and so B x B is the same cardinality. Therefore, there is a bijection between B and B x B, which can then be extended linearly into an isomorphism. Now, we forget the scalar multiplication, and this gives a group isomorphism for the additive groups of R and C.

bilog78 ยท 11 points ยท Posted at 06:01:46 on November 21, 2015 ยท (Permalink)

Are there proofs that don't require AC?

ranarwaka ยท 9 points ยท Posted at 07:42:42 on November 21, 2015 ยท (Permalink)

iirc there are models of ZF where R as a vector space over Q doesn't have a base

W_T_Jones ยท 7 points ยท Posted at 10:17:27 on November 21, 2015 ยท (Permalink)

That doesn't imply that R and C are not isomorphic as an additive group though, right?

farmerje ยท 2 points ยท Posted at 03:12:56 on November 22, 2015 ยท (Permalink)

An isomorphism between (โ„,+) and (โ„‚,+) implies the existence of a non-measurable subset of โ„, so you need a fairly strong version of choice to prove it. For example, you couldn't prove they're isomorphic in ZF + the Axiom of Dependent Choice since it's not strong enough to prove the existence of non-measurable subsets of โ„.

MyOverflow ยท 1 points ยท Posted at 05:40:38 on November 22, 2015 ยท (Permalink)*

This is true. Sorry for not replying to the parent comments earlier!

Source: "A Consequence of the Axiom of Choice"*, C.J. Ash. J. Austral. Math. Soc. 19 (Series A). 1975, pages 306-308.

For a good, accessible book to see the funky things that happen because of (or because of the lack of) the Axiom of Choice, check out Axiom of Choice by Horst Herrlich.

* PDF file.

HilbertsHotelManager ยท 1 points ยท Posted at 17:40:03 on November 21, 2015 ยท (Permalink)

There are models of ZF where any arbitrary vector space is not guaranteed to have a basis.

scrumbly ยท 6 points ยท Posted at 06:13:39 on November 21, 2015 ยท (Permalink)

Can you explain why B and B x B have the same cardinality?

MedalsNScars ยท 11 points ยท Posted at 06:54:13 on November 21, 2015 ยท (Permalink)

Not the above poster, but I would guess it's similar to the proof that the rationals are countable but it's like 2 AM and I'm too tired to math now.

scrumbly ยท 1 points ยท Posted at 07:20:44 on November 21, 2015 ยท (Permalink)

Ah I think I can convince myself that B must be countably infinite and from there the result follows as you suggest.

SnailRhymer ยท 3 points ยท Posted at 09:39:00 on November 21, 2015 ยท (Permalink)

B must actually be uncountable, since any Q-vector space with a countable basis must also be countable.

If you accept the continuum hypothesis, then B must have the cardinality of the reals since card([;\mathbb{N};]) < card(B) <= card([;\mathbb{R};]) and CH says there are no cardinalities strictly between these two.

[;\mathbb{R};] and the interval (0,1) have the same cardinality (eg the map 1/(1-x) - 1/x). (0,1) and (0,1)x(0,1) have the same cardinality, which can be seen by interleaving decimal places:

(0.abcdef..., 0.wxyz...) -> 0.awbxcydz....

So card(BxB) = card([;\mathbb{R}\times\mathbb{R};]) = card((0,1)x(0,1)) = card((0,1)) = card([;\mathbb{R};]) = card(B)

Without the continuum hypothesis it's a little trickier to show that B has the cardinality of the reals. I suspect it can be shown that a Q-vector space with basis C has the same cardinality as C when C is infinite, and so B cannot have cardinality smaller than [;\mathbb{R};].

proto-n ยท 2 points ยท Posted at 10:05:27 on November 21, 2015 ยท (Permalink)

Your bijection between (0,1) and (0,1)x(0,1) is not really a bijection in this form, and it's not easily fixed either, see http://math.stackexchange.com/questions/290019/cardinality-of-mathbbr-and-mathbbr2

SnailRhymer ยท 3 points ยท Posted at 12:15:37 on November 21, 2015 ยท (Permalink)*

I didn't notice that you need to choose what to do with a tail of 9s or 0s. Once you choose a representation it's an injection, so card((0,1)x(0,1)) <= card((0,1)). Clearly card((0,1)) <= card((0,1)x(0,1)), and therefore the cardinalities are equal.

MyOverflow ยท 5 points ยท Posted at 11:58:32 on November 21, 2015 ยท (Permalink)*

AC => An infinite set has the same cardinality as any Cartesian product of a finite number of copies of itself.

In fact, Tarski's theorem shows that the converse also holds, so this is just another form of the Axiom of Choice.

chaosmosis ยท 1 points ยท Posted at 00:21:44 on November 22, 2015 ยท (Permalink)

That's a more intuitive than typical way to phrase AC, thanks.

person594 ยท 1 points ยท Posted at 14:37:53 on November 21, 2015 ยท (Permalink)

Assuming B has the same Cardinality as R (I'm pretty sure it does; someone correct me if I'm wrong), there is a rather straightforward bijection between RxR and R. Represent each R as an infinite string of digits, and construct a new infinite string of digits by interleaving the digits from your original two numbers. There is a little bit more involved to account for ambiguous reorientations (The real number 1 can be represented by either "10000000..." or "999999999...", for instance), but that is the gist of it.

mathers101 ยท 1 points ยท Posted at 22:05:35 on November 21, 2015 ยท (Permalink)

So just to be clear, R and C are not isomorphic as vector spaces, just as additive groups? And that's why we're required to "forget" the scalar multiplication, because (presumably) we could find some contradiction using the scalar multiplication?

MyOverflow ยท 1 points ยท Posted at 22:13:19 on November 21, 2015 ยท (Permalink)

They're not isomorphic as vector spaces over R, but they are isomorphic as vector spaces over Q. The reason we forget scalar multiplication is simply because a vector space is just an abelian group where we have the extra structure provided by scalar multiplication, and so to get the result we want, we just realize we're already done by saying "oh wait, we don't need this scalar multiplication business" at the end.

When working more abstractly with vector spaces (usually not done in a first course in linear algebra), you have to be very aware of what field you're working over.

mathers101 ยท 1 points ยท Posted at 22:15:32 on November 21, 2015 ยท (Permalink)

Right, that helps. I asked because I recently learned about invariant basis number of modules, and all fields have IBN, so if they were isomorphic as vector spaces over R I would have been quite confused/worried. Being isomorphic over Q makes sense though because it's clearly not a finite basis.

MegaZambam ยท 1 points ยท Posted at 01:46:42 on November 21, 2015 ยท (Permalink)

This was my thinking as well. Didn't seem intuitive to me.

Exomnium ยท 5 points ยท Posted at 01:41:38 on November 21, 2015 ยท (Permalink)

Well, doesn't the isomorphism require the axiom of choice?

Biermoese ยท 2 points ยท Posted at 15:52:12 on November 21, 2015 ยท (Permalink)

Can you explain this in layman terms (i.e., for a physicist)? :)

CraftyBarbarianKingd ยท 4 points ยท Posted at 19:39:48 on November 21, 2015 ยท (Permalink)

I'll assume that a physicist is quite familiar with complex and real numbers. Assume you went to a first grade math lesson in some alien civilization and these aliens are learning some operation like addition, then what you'll probably hear is the teacher giving two weird words say flemkh and blemkh then the class would respond with only one weird word. After a bit of observation you might be able to translate those "numbers" and you'll understand what they're doing, but actually you can never know if this is really "addition" you just know that your translation works and so whatever number system they are using must have the same algebraic properties as our numbers, the technical term to this is isomorphism. What OP is saying is that if you consider real number and complex numbers and addition alone you can translate the numbers in a similar way such that all the algebraic properties are conserved.

Exomnium ยท 1 points ยท Posted at 00:34:49 on November 22, 2015 ยท (Permalink)

The notion of a vector space is defined over for arbitrary fields. In physics we typically talk about vector spaces over R or C, but it's perfectly fine to talk about a vector space over Q, the field of rational numbers, for instance. Also in physics we always think about vector spaces with some other kind of structure, like an inner product, or at the very least a topology, but this proof is going to absolutely require that you forget about that kind of structure. A vector space is a just set of things which can be added together and which has multiplication by scalars (from some field).

A (Schauder) basis of a vector field is a set of linearly independent vectors such that any vector can be written as a finite linear sum of scalar multiples of the basis vectors (in physics we always implicitly use Hamel bases, in which any vector is a possibly infinite sum of basis vectors, but an arbitrary vector space doesn't necessarily have a topology so the notion of an infinite sum doesn't necessarily make sense).

The dimension of a vector space is simply the cardinality of any basis of it (although you need the axiom of choice to prove that this is well-defined; without choice it's consistent that there are vector spaces with no bases or even a vector space with bases of different cardinalities). The dimension can be infinite, obviously.

It is easy to show that any two vector spaces (over the same field) that are the same dimension are isomorphic (as vector spaces, again there might be other structure which makes them different, like the difference between different Lp spaces, even though they're all the same dimension and isomorphic as abstract vector spaces).

So two perfectly good vector spaces over Q are R and C. Since Q is countable any basis of R or C must have cardinality of the continuum, if the basis were smaller the set of all finite Q-linear combinations would be smaller than R or C. If the basis were bigger then the basis would be bigger than the vector space itself. So R and C are vector spaces over Q with the same dimension, so they must be isomorphic as abstract vector spaces and therefore as groups under addition.

However any such isomorphism (a function f:R->C which is invertible and Q-linear) is very pathological. It's discontinuous obviously and furthermore it's not even a 'measurable function' meaning that even if you construct f to be bounded (i.e. the image of a bounded interval in R is a bounded set in C) the integral of the function from 0 to 1, say, can't be made well-defined, even though there are a lot of everywhere discontinuous functions that have perfectly fine integrals. Also the proof relies on the axiom of choice in an essential way. It's consistent in ZF without choice that R and C are not isomorphic as Q-vector spaces (it's an open problem whether or not they're isomorphic as groups under addition without choice).

PIDomain ยท 92 points ยท Posted at 02:39:50 on November 21, 2015 ยท (Permalink)

Not false, but the statement "If X is smaller in cardinality than Y, then X has fewer subsets than Y" is independent of ZFC.

AsidK ยท 19 points ยท Posted at 06:15:47 on November 21, 2015 ยท (Permalink)

Wow that's pretty cool. Reference?

PIDomain ยท 17 points ยท Posted at 06:52:14 on November 21, 2015 ยท (Permalink)*

It's well known that the Luzin hypothesis, which states that 2aleph_0 = 2aleph_1 , is consistent with ZFC. However, you can deduce the original statement from the generalized continuum hypothesis.

Workaphobia ยท 2 points ยท Posted at 13:54:31 on November 24, 2015 ยท (Permalink)

You know, suddenly I understand the anger that mathematicians felt toward Georg Cantor.

graciousgroob ยท 1 points ยท Posted at 17:46:30 on November 21, 2015 ยท (Permalink)

Wait, doesnt 2aleph_0 = aleph_1 straight up? I thought this was true because of the cardinality of power sets yada yada yada.

ShareDVI ยท 3 points ยท Posted at 18:09:18 on November 21, 2015 ยท (Permalink)

Reread the post above yours

orbital1337 ยท 3 points ยท Posted at 23:05:48 on November 21, 2015 ยท (Permalink)

Certainly not, 2aleph_0 = aleph_1 is the continuum hypothesis and is known to be independent of ZFC. Aleph_1 is the next cardinal after aleph_0 but 2aleph_0 might be equal to aleph_37 or pretty much any other cardinal.

SilchasRuin ยท 3 points ยท Posted at 07:50:21 on November 21, 2015 ยท (Permalink)

There's a much stronger theorem implying this.

OperaSona ยท 4 points ยท Posted at 16:15:03 on November 21, 2015 ยท (Permalink)

Along these lines, I like how complex it is to prove, without the axiom of choice, that if there is a one-to-one correspondance from [;{1,2,3} \times A;] to [;{1,2,3} \times B;], then there is one between A and B. The paper by Doyle and Conway is pretty famous and I'm sure many here have already given it a read, but for anyone who haven't, try to stick through at least the "Division by two" section. It's fun. https://math.dartmouth.edu/~doyle/docs/three/three.pdf

HippieSpider ยท 1 points ยท Posted at 12:20:22 on March 12, 2016 ยท (Permalink)

I assume that this only applies to the case of infinite cardinalities? Similar to how the Axiom of Choice can be deduced from ZF in the finite case?

PIDomain ยท 1 points ยท Posted at 19:42:53 on March 12, 2016 ยท (Permalink)

Yes.

UniformCompletion ยท 21 points ยท Posted at 05:16:52 on November 21, 2015 ยท (Permalink)

If there is an injective homomorphism from a free group on m generators to a free group on n generators, then mโ‰คn.

AsidK ยท 3 points ยท Posted at 06:16:43 on November 21, 2015 ยท (Permalink)

Woah. What's the counterexample?

UniformCompletion ยท 12 points ยท Posted at 06:25:12 on November 21, 2015 ยท (Permalink)

In the free group on two generators, the set {x2 , y2, xy} has no relations, and so it generates a subgroup isomorphic to the free group on three generators.

[deleted] ยท 4 points ยท Posted at 08:44:36 on November 21, 2015 ยท (Permalink)*

[deleted]

[deleted] ยท 1 points ยท Posted at 16:16:26 on November 22, 2015 ยท (Permalink)

Every countable group can be embedded in the free group on two generators

Not true. You mean that every countable group can be embedded in a quotient of the free group on two generators.

[deleted] ยท 1 points ยท Posted at 06:12:23 on November 21, 2015 ยท (Permalink)

[deleted]

UniformCompletion ยท 4 points ยท Posted at 06:22:57 on November 21, 2015 ยท (Permalink)

For me, the intuition is that either such an injection should not exist, or there should be no well-defined rank for a free group. The surprising thing to me is that rank is well-defined, and we have the existence of injections that don't exist for free objects in so many other categories (e.g. sets, abelian groups, rings).

WhiskersForPresident ยท 1 points ยท Posted at 18:50:09 on April 10, 2016 ยท (Permalink)

I'm veeeeery late, but I gotta say this: that map isn't invertible, it isn't even injective. c_3-1 c_2 c_1-1 c_2 is in the kernel.

travvo ยท 2 points ยท Posted at 19:06:44 on April 10, 2016 ยท (Permalink)

Oh good spot.

[deleted] ยท 186 points ยท Posted at 10:13:34 on November 21, 2015 ยท (Permalink)*

[deleted]

Gear5th ยท 76 points ยท Posted at 11:25:38 on November 21, 2015 ยท (Permalink)

Could you please explain why this is untrue?

AcellOfllSpades ยท 166 points ยท Posted at 13:24:45 on November 21, 2015 ยท (Permalink)

Throw a dart at a dartboard. The probability that you'l hit any point is 0, but you're going to hit a point.

qjornt ยท 140 points ยท Posted at 14:10:59 on November 21, 2015 ยท (Permalink)

the probablity that you'll hit any point is 1 (given that you hit the board). the probability that you will hit a specific point is however very close to 0 since dartboards are discrete in a molecular sense, hence each "blunt" point on the board has a finite size, thus a throw can be described by a discrete random variable.

your statement holds true for continious random variables though, as I said somewhere else, "For a continous r.v. P(X=x) = 0 โˆ€ x โˆˆ ฮฉ, but X has to take a value in ฮฉ when an event occurs."

AcellOfllSpades ยท 94 points ยท Posted at 14:28:47 on November 21, 2015 ยท (Permalink)

Yeah, it's not 0 if you look at it on a molecular level - I meant an idealized dartboard, which I should've made more clear.

[deleted] ยท 38 points ยท Posted at 14:45:21 on November 21, 2015 ยท (Permalink)

[deleted]

ChezMere ยท 18 points ยท Posted at 15:14:05 on November 21, 2015 ยท (Permalink)

Do we have reason to believe time is continuous either?

[deleted] ยท 24 points ยท Posted at 15:23:07 on November 21, 2015 ยท (Permalink)

[deleted]

neoandrex ยท 3 points ยท Posted at 14:32:01 on November 22, 2015 ยท (Permalink)

We actually have planck time, which is defined as the time in whick the light goes through a distance of a planck unit, since nothing below that interval of space makes sense. So in a way time IS discrete. I'm on mobile but you should find it on Wikipedia.

oddark ยท 2 points ยท Posted at 16:40:10 on November 21, 2015 ยท (Permalink)

this might not be completely accurate, the the Planck time is believed to be the smallest meaningful unit of time.

ChrisLomont ยท 16 points ยท Posted at 16:55:26 on November 21, 2015 ยท (Permalink)

All the Plank units are basically numerology, and people love when they pop out of equations. Some are values we encounter in everyday life or experiments (Plank mass, Plank impedance, for example).

"Because the Planck time comes from dimensional analysis, which ignores constant factors, there is no reason to believe that exactly one unit of Planck time has any special physical significance"

[1] https://en.wikipedia.org/wiki/Planck_time

rudolfs001 ยท 1 points ยท Posted at 20:53:31 on November 21, 2015 ยท (Permalink)

Look in to Planck time

ChezMere ยท 3 points ยท Posted at 21:00:58 on November 21, 2015 ยท (Permalink)

Well, all that really says is that we also don't have reason to believe time isn't continuous, either...

rudolfs001 ยท 1 points ยท Posted at 21:19:34 on November 21, 2015 ยท (Permalink)

The idea is that the Planck time is the smallest amount of time that we can currently say is proportional to the smallest possible time by a given ratio. The value of the ratio is yet to be determined and needs better theories of quantum gravity.

Fundamentally, time is a measure of change. The question then becomes - what is the smallest increment of change possible?

The simple answer - some quantum bit of information being flipped from 0 to (+-)1 or vice-versa.

Then you ask - what's the smallest/most fundamental information carrying quanta possible?

To answer that, we'd have to delve into M-theory or start from scratch and construct a new model universe. Neither are particularly simple.

ChrisLomont ยท 6 points ยท Posted at 16:52:15 on November 21, 2015 ยท (Permalink)

Space may also be continuous, energy levels (unbound particles) are likely continuous, etc. There are many, many physical things that are not known to be discrete, and for all purposes, are considered continuous until shown otherwise.

[deleted] ยท 2 points ยท Posted at 22:31:41 on November 21, 2015 ยท (Permalink)

P(X=x) = 0 โˆ€ x โˆˆ ฮฉ is kind of unnecessary, but we get your point.

[deleted] ยท 3 points ยท Posted at 23:10:20 on November 21, 2015 ยท (Permalink)

[deleted]

[deleted] ยท 1 points ยท Posted at 02:23:28 on November 22, 2015 ยท (Permalink)

Yeah, I was just lamencing it.

Blaposte ยท 1 points ยท Posted at 00:43:06 on February 10, 2016 ยท (Permalink)

ฮฉ

What set does this represent?

austin101123 ยท 19 points ยท Posted at 14:57:00 on November 21, 2015 ยท (Permalink)

That doesn't sound right. Wouldn't the probability of each point be infitessimal? (Assuming location infinitely more accurate than Planck length, and a tip with area of a point.)

AcellOfllSpades ยท 31 points ยท Posted at 15:11:41 on November 21, 2015 ยท (Permalink)

There are no infinitesimal real numbers except 0. Probability is a real number. (And yeah, I'm ignoring the fact that the tip is blunt, the fact that the dartboard is made out of molecules...)

halfajack ยท 13 points ยท Posted at 17:50:40 on November 21, 2015 ยท (Permalink)

Is it possible to do probability in the hyperreals?

wintermute93 ยท 1 points ยท Posted at 18:54:47 on November 22, 2015 ยท (Permalink)

Probably, but there's no reason to suspect that the physical universe behaves like the hyperreals.

SunAvatar ยท 1 points ยท Posted at 21:02:18 on April 26, 2016 ยท (Permalink)

You probably could, and you could thereby have an infinitesimal hyperreal probability of hitting an infinitesimally small area of the dartboard. But this wouldn't 'fix' anything, because the dart would still hit some point, the point would still have volume zero, and the prior probability of hitting it would still be zero.

AcellOfllSpades ยท 1 points ยท Posted at 19:36:40 on November 21, 2015 ยท (Permalink)

I think it is, but I'm not an expert at all.

Kvothealar ยท 2 points ยท Posted at 02:54:28 on November 22, 2015 ยท (Permalink)

What he means that it is non-zero.

mennovf ยท 1 points ยท Posted at 18:22:15 on November 21, 2015 ยท (Permalink)

But when you're talking about continuous distributions you're talking about probability densities which are infinitesimals, right?

AcellOfllSpades ยท 4 points ยท Posted at 19:36:11 on November 21, 2015 ยท (Permalink)

No, when you work with distributions the only meaningful thing is the integral of the distribution - the probability it'll land in a specific range. You don't work with or need infinitesimals at all.

gottabequick ยท 1 points ยท Posted at 21:06:02 on November 21, 2015 ยท (Permalink)

Even if you include the infinitesimals it doesn't allow regularity.

32363031323031 ยท 1 points ยท Posted at 17:16:19 on November 21, 2015 ยท (Permalink)

Whats the probability of hitting the same point twice?

AcellOfllSpades ยท 2 points ยท Posted at 17:20:27 on November 21, 2015 ยท (Permalink)

Zero.

32363031323031 ยท 1 points ยท Posted at 17:51:24 on November 21, 2015 ยท (Permalink)

So one has the same probability of hitting the same point once, and infinity times?

Krexington_III ยท 2 points ยท Posted at 19:03:07 on November 21, 2015 ยท (Permalink)

Yes, since it can't happen. All of this is just a snickering way of saying that for a continuous event ("hitting a certain place"), only intervals are meaningful - you can draw a tiny circle on a dartboard and calculate exactly the probability of the dart landing within that circle, but as the circle gets smaller the probability goes asymptotically to zero. When the circle has zero radius, the probability is exactly zero to hit that circle - which is to say, "we don't calculate things in the real world this way".

That is why it's as probable to hit a point once as a trillion times. Because it can't happen at all.

[deleted] ยท 1 points ยท Posted at 13:49:40 on November 21, 2015 ยท (Permalink)

[deleted]

anonemouse2010 ยท 4 points ยท Posted at 14:09:19 on November 21, 2015 ยท (Permalink)

You're talking about the real world, he's talking about a random variable.

[deleted] ยท 1 points ยท Posted at 14:43:15 on November 21, 2015 ยท (Permalink)

Or 'an idealized dartboard', which no one has yet to come across, making it a little less interesting.

AcellOfllSpades ยท 4 points ยท Posted at 15:27:38 on November 21, 2015 ยท (Permalink)

Well, everything in mathematics is idealized. You're never gonna see a perfect circle in real life or dig up a 2 in your backyard.

rynvndrp ยท 0 points ยท Posted at 14:12:17 on November 21, 2015 ยท (Permalink)

Every random variable is a distribution and never a single number is the Bayesian approach.

TwoFiveOnes ยท 1 points ยท Posted at 16:17:09 on November 21, 2015 ยท (Permalink)

Is there an example that doesn't use continuous random variables? I feel like that'd make the statement feel less artificial

AcellOfllSpades ยท 7 points ยท Posted at 16:21:26 on November 21, 2015 ยท (Permalink)

How are continuous random variables artificial? And no, there isn't.

TwoFiveOnes ยท 4 points ยท Posted at 16:27:14 on November 21, 2015 ยท (Permalink)

They aren't! Unless you're an extreme constructivist, or something, but that's not my point. I just mean that to the casual reader the dartboard example seems like a convenient oversight of the bluntness of the dart.

[deleted] ยท 0 points ยท Posted at 23:07:53 on November 21, 2015 ยท (Permalink)

[deleted]

AcellOfllSpades ยท 3 points ยท Posted at 23:24:35 on November 21, 2015 ยท (Permalink)

There are no infinitely small real numbers. Probabilities are real numbers. And it's not infinitely small, it's 0.

ENelligan ยท 2 points ยท Posted at 17:02:40 on November 22, 2015 ยท (Permalink)

I prefer the following example to the dartboard one:

If you flip a coin infinitely many time, the probability of having no head in the sequence is 0. But the infinite sequence composed of only tails is element of the set of all infinite sequences. Therefor it's possible that head will never shows up in the sequence.

[deleted] ยท 2 points ยท Posted at 18:07:49 on November 21, 2015 ยท (Permalink)*

[deleted]

Gear5th ยท 3 points ยท Posted at 02:10:07 on November 22, 2015 ยท (Permalink)

Your first example is nice. About selecting a number b/w (0,1), you can't ! Atleast not with a fair distribution.

If you try and select a number fairly, you will keep on giving out digits without ever reaching a number. If you stop at any point, you haven't been fair because selecting fairly, you must always land at an irrational number. Also, you can't say something like "I selected pi", because then too you are being unfair.

unfair = non-zero probability for some numbers.

[deleted] ยท 1 points ยท Posted at 13:15:31 on November 22, 2015 ยท (Permalink)*

[deleted]

Gear5th ยท 1 points ยท Posted at 13:42:17 on November 22, 2015 ยท (Permalink)

Same here. Highschool level maths and some youtube.

If you imagine increasing the number of cards without bound, the probability approaches 0.

Yes, but that is not the same as selecting a real number. Because the real set has larger cardinality than the set of natural numbers.

If you had a set and kept adding one element at a time for all eternity, you will never be able to get more than 0% of all the real numbers b/w 0 and 1 (even if you do it for an infinite time)

tpn86 ยท 1 points ยท Posted at 23:55:39 on November 22, 2015 ยท (Permalink)

I give you an infinity sided dice with numbers (0,1). The probability of any outcome is 1/infinity=0. But if you throw it then some value does come out.

Gear5th ยท 1 points ยท Posted at 04:43:08 on November 23, 2015 ยท (Permalink)*

Isn't that like saying "If you throw a sphere, then when it lands, there will be exactly one point at the top."?

I'm not sure if that will be true, but then again, I'm not a mathematician.

tpn86 ยท 2 points ยท Posted at 08:37:08 on November 23, 2015 ยท (Permalink)

Sounds about right..

NamelessAsOfYet ยท 22 points ยท Posted at 23:07:30 on November 21, 2015 ยท (Permalink)

Isn't this a version of 'almost surely', where an event with a probability of 1 might not happen?

The way it was explained to me was that if you gave a monkey a typewriter and infinite time to write on it, the probability that it will write the works of Shakespeare is 1. But then again, it might also just repeat ADADADADADADADADADADADADAD for eternity.

Kvothealar ยท 2 points ยท Posted at 02:55:30 on November 22, 2015 ยท (Permalink)

In that case though it isn't probability 1.

NamelessAsOfYet ยท 2 points ยท Posted at 11:19:03 on November 22, 2015 ยท (Permalink)

In which case? In the case of 'almost surely' or in the monkey case? As I understand it, in both cases, the probability is 1. It just doesn't necessarily happen.

Kvothealar ยท 3 points ยท Posted at 15:09:21 on November 22, 2015 ยท (Permalink)

Well in both it means that the probability would be approaching 1. It would be a limit question wouldn't it?

But as long as there is one example where it won't happen you could never actually get P=1.

It's like ฮฃ[ยน/โ‚‚]n , n={0,1,2...}

It isn't just = 2, it's limit as n->โˆž = 2. That is the supremum for the set. But it wouldn't be in the set. It "converges" to its limit.

At least, this is what I have got out of all of my undergraduate. Maybe the profs are lying to us to keep us content until we take a more complete course. :p

[deleted] ยท 7 points ยท Posted at 11:10:51 on November 21, 2015 ยท (Permalink)*

[deleted]

xe267 ยท 20 points ยท Posted at 11:46:56 on November 21, 2015 ยท (Permalink)

Throw a dart randomly at a unit square dartboard. The probability that it will land inside a certain region is the area of the region - so the probability it'll land on any one point of the board is 0. But it's got to land somewhere!

zeldn ยท 3 points ยท Posted at 21:45:36 on November 21, 2015 ยท (Permalink)

Is this because there are an infinite number of points it can hit? So if you divided the board into areas of 1x1 Planck lengths, now you'd have a probably greater than 0 of hitting a specific one of the areas

xe267 ยท 1 points ยท Posted at 21:38:05 on November 22, 2015 ยท (Permalink)

Exactly, and the dart tip has non-zero area, and so on - I'm talking about an idealised dartboard and an idealised dart. Essentially what I'm saying is "pick a random point in [0,1] x [0,1]" - but darts and a dartboard feels more intuitive.

Aydoooo ยท 44 points ยท Posted at 19:17:30 on November 21, 2015 ยท (Permalink)*

If the surface of a 3D object is infinitely large, then so is its volume.

Edit: Here is a pretty simple counterexample. You can say that one can fill this horn with a finite amount of liquid paint, yet need an infinite amount to paint the inside.

kingfrito_5005 ยท 16 points ยท Posted at 03:04:40 on November 22, 2015 ยท (Permalink)

Ah yes, the classic 'first day of calc II' example.

super_aardvark ยท 6 points ยท Posted at 22:53:00 on November 21, 2015 ยท (Permalink)

yet need an infinite amount to paint the inside

...with a constant thickness of paint -- but if you hold to that restriction, you'll never be able to fill the volume, as the radius gets too narrow to contain that thickness.

Aydoooo ยท 7 points ยท Posted at 23:00:43 on November 21, 2015 ยท (Permalink)

Of course the example doesn't hold true, since this whole paradoxon is not possible in reality.

LudoRochambo ยท 3 points ยท Posted at 22:00:30 on November 21, 2015 ยท (Permalink)

this is a bad play on words though, isnt it? its better to say if its area is infinite. theres a sort of, like, psychology behind "infinitely large".

anyways anyone interested, area is power of 2, volume is power of 3 so you can make the volume converge quicker than area when dealing with recipricals. that immediately gives you a general idea of what the counter examples look like (curved ice cream cones).

Aydoooo ยท -1 points ยท Posted at 22:13:47 on November 21, 2015 ยท (Permalink)

No play on words. it's just you misinterpreting I guess.

SagaCult ยท 2 points ยท Posted at 22:13:07 on November 21, 2015 ยท (Permalink)

Why can't it be infinitely large in just one of the dimensions?

SirUtnut ยท 1 points ยท Posted at 01:03:07 on November 23, 2015 ยท (Permalink)

The Cantor set, Sierpinski carpet, and Menger sponge all have zero volume and infinite surface area (in their respective dimensions).

I assume this generalizes to as many dimensions as you want, but that seems like a dangerous assumption here.

KnowledgeRuinsFun ยท 51 points ยท Posted at 04:55:11 on November 21, 2015 ยท (Permalink)

The closure of the open ball is the closed ball.

middleman2308 ยท 14 points ยท Posted at 05:07:59 on November 21, 2015 ยท (Permalink)

Care to explain?

AseOdin ยท 28 points ยท Posted at 05:38:59 on November 21, 2015 ยท (Permalink)

You can look at a discrete space, for example, where the open ball is clopen. In this case, the closure of the open ball is still the open ball and could be strictly contained in the closed ball.

Meliorus ยท 6 points ยท Posted at 07:21:08 on November 21, 2015 ยท (Permalink)

So 'the closed ball' isn't equal to the open ball even though the open ball is closed?

[deleted] ยท 10 points ยท Posted at 08:24:47 on November 21, 2015 ยท (Permalink)

I think he's imagining a situation where you consider a set of points and the in a discrete topological space with a metric and say the open ball is the set of points of distance less than 1 from the origin and the closed ball is the set of points of distance <= 1 from the origin. If the points of the space were set up so that there are at least a few points exactly at distance 1 from the origin then his statements follow.

urastarburst ยท 2 points ยท Posted at 13:47:08 on November 21, 2015 ยท (Permalink)

Those accumulation points though.

AlmostNever ยท 5 points ยท Posted at 15:01:05 on November 21, 2015 ยท (Permalink)

I like accumulation and condensation points because they sound like weather forecasts.

Meliorus ยท 2 points ยท Posted at 13:55:55 on November 21, 2015 ยท (Permalink)

Ah, right, thanks

croissantology ยท 1 points ยท Posted at 02:18:20 on November 22, 2015 ยท (Permalink)

Consider the metric space R with the discrete metric: d(x,y)=1 if xโ‰ y, 0 if x=y. Then the open ball B(center 0,radius 1)={0} since it contains points strictly less than 1 unit away. It is closed since for any x not in B(0,1), i.e. any xโ‰ 0, the ball B(x,1)={x} is not contained in B(0,1). So B(0,1) is equal to its closure. But the closed ball CB(0,1) = R since it includes points of distance 1 away.

yoloed ยท 5 points ยท Posted at 05:46:23 on November 21, 2015 ยท (Permalink)

For what class of topological spaces is this true? It is clear that it is true for Euclidean spaces, but what about other spaces?

readsleeprepeat ยท 10 points ยท Posted at 07:43:34 on November 21, 2015 ยท (Permalink)

It's true for any normed vector space. Better, I found a more general characterization on Stackexchange.

13467 ยท 1 points ยท Posted at 14:13:56 on November 22, 2015 ยท (Permalink)

What about an open/closed ball of radius 0? Clearly {} != {0}.

readsleeprepeat ยท 2 points ยท Posted at 21:42:12 on November 22, 2015 ยท (Permalink)

My analysis book defines an open ball only with radius greater than 0 and so does Wikipedia. If we allow r=0, lots of things about open balls don't work.

13467 ยท 1 points ยท Posted at 21:56:43 on November 22, 2015 ยท (Permalink)

Ah, I figured. I don't know all that much about the subject. Could there be a more freaky topology/metric in which an open ball of some radius r > 0 is empty, but the corresponding closed ball isn't?

readsleeprepeat ยท 1 points ยท Posted at 06:06:22 on November 23, 2015 ยท (Permalink)

No, I don't think so, if r>0 for an open ball B around a, then d(a,a)<r => aโˆˆB. So an open ball with r>0 can't be empty. This only uses the existence of a metric, so there really can't be a freaky metric that's different.

PeterPorky ยท 1 points ยท Posted at 05:45:41 on November 22, 2015 ยท (Permalink)

Oh my God.

So that's it.

When the golden snitch in The Deathly Hallows "opened at the close" it opened because it was already closed is this real life

lgastako ยท 85 points ยท Posted at 06:09:25 on November 21, 2015 ยท (Permalink)

1 != 0.999...

oighen ยท 174 points ยท Posted at 13:00:52 on November 21, 2015 ยท (Permalink)

This is true, 1 factorial is definitely equal to 0.999...

chrzan ยท 242 points ยท Posted at 15:15:37 on November 21, 2015 ยท (Permalink)

Actually, 1! is not equal to 0.999. And can we stop trying to sound all smug by adding ellipses to the ends of our sentences?

magicturtle12 ยท 5 points ยท Posted at 15:42:39 on November 21, 2015 ยท (Permalink)*

I believe he is using the unequal notation != , not factorial notation Edit: apparently I get downvoted for trying to politely correct someone who i thought had misinterpreted a previous post... and here I thought the /r/math subreddit was a place of friendly discussion

explorer58 ยท 70 points ยท Posted at 16:07:11 on November 21, 2015 ยท (Permalink)

Yeah I have a hard time with jokes too

alecbenzer ยท 8 points ยท Posted at 16:09:57 on November 21, 2015 ยท (Permalink)

They were just joking.

Krexington_III ยท 9 points ยท Posted at 19:07:35 on November 21, 2015 ยท (Permalink)

I also didn't get /u/chrzan 's joke until I'd looked at it for a while. It was an excellent joke, though!

puffybaba ยท 3 points ยท Posted at 03:31:13 on November 22, 2015 ยท (Permalink)

redditors downvote those who are not in on the joke; it appears it's more important to be funny than helpful to most redditors who upvote or downvote.

[deleted] ยท 1 points ยท Posted at 22:33:05 on November 21, 2015 ยท (Permalink)

Yes but that is rather ambiguous considering "!" as notation in western mathematics strictly refers to permutations, i.e. 5! = 5 x 4 x 3 x 2 x 1

Edit: And in the obvious case of 1! = .000000001 x .000000000000001 etc.

muntoo ยท 1 points ยท Posted at 13:37:15 on November 22, 2015 ยท (Permalink)

Friendly discussion? On reddit? Nay, sir, you must be mistaken...

LudoRochambo ยท 1 points ยท Posted at 00:06:54 on November 22, 2015 ยท (Permalink)

the problem here is you give a damn about fake points

InSearchOfGoodPun ยท 66 points ยท Posted at 19:31:14 on November 21, 2015 ยท (Permalink)

This notation of != for "not equal" should absolutely not be used in /r/math. In math, ! is factorial. Lots of needless confusion here.

somnolent49 ยท 68 points ยท Posted at 21:06:19 on November 21, 2015 ยท (Permalink)

Spacing resolves the issue just fine.

InSearchOfGoodPun ยท 1 points ยท Posted at 05:15:38 on November 22, 2015 ยท (Permalink)

No, it doesn't. I think that several comments in this thread demonstrate this.

As a separate matter, it's a piece of notation from computer programming, not mathematics. In fact, I would wager that more mathematicians would understand \neq better than !=

lgastako ยท 30 points ยท Posted at 19:38:14 on November 21, 2015 ยท (Permalink)

Sorry I'm a programmer, not a mathematician. What's the proper notation? 1 <> 0.999...? I guess unicode always works... 1 โ‰  0.999...?

[deleted] ยท 66 points ยท Posted at 21:22:01 on November 21, 2015 ยท (Permalink)

!= is fine and most people will understand it. Just use white space to make it clear or elaborate if you think its unclear. Another option is 2 =/= 1

Kvothealar ยท 3 points ยท Posted at 02:57:44 on November 22, 2015 ยท (Permalink)

I like this one. LaTeX notation also works.

\neq

muntoo ยท 0 points ยท Posted at 13:39:43 on November 22, 2015 ยท (Permalink)

=/= looks hideous. != works in many contexts.

LudoRochambo ยท 18 points ยท Posted at 21:47:26 on November 21, 2015 ยท (Permalink)

!= is fine, anyone not getting what you mean is just being a pretentious asshat.

because someone really looked at that and thought 1 factorial equals 0.99999.. instead of thinking for a fraction of a second and realizing the comment is just the very common 1 is equal to 0.9999? pfft

Scraendor ยท 1 points ยท Posted at 23:33:30 on November 21, 2015 ยท (Permalink)

~(1 = .999...)

Qbaca ยท 1 points ยท Posted at 03:42:29 on November 23, 2015 ยท (Permalink)

1~=.999?

kblaney ยท 1 points ยท Posted at 20:45:27 on November 21, 2015 ยท (Permalink)

You can use the LaTeX \neq [; \neq ;]

So [; 1 \neq .\overline{9} ;]

wintermute93 ยท 2 points ยท Posted at 18:57:05 on November 22, 2015 ยท (Permalink)

"1! = 1" != "1 != 1"

PositiveAlcoholTaxis ยท 1 points ยท Posted at 23:44:26 on November 21, 2015 ยท (Permalink)

Used to do this in maths class in secondary (high school). Nobody had a clue what it meant :')

chromeless ยท 8 points ยท Posted at 11:14:58 on November 21, 2015 ยท (Permalink)

In the reals, yes.

[deleted] ยท 24 points ยท Posted at 13:02:02 on November 21, 2015 ยท (Permalink)

And in the rationals and the complexes...

IvanTheNotTooBad ยท -9 points ยท Posted at 17:17:57 on November 21, 2015 ยท (Permalink)

0.999... repeating infinitely isn't in the rationals.

lvanneke ยท 25 points ยท Posted at 17:28:43 on November 21, 2015 ยท (Permalink)

It can be expressed as 1/1, so it's in the rationals.

MegaZambam ยท -6 points ยท Posted at 19:24:51 on November 21, 2015 ยท (Permalink)

It's the sum of 9/10n from 1 to infinity. It's a sum of rationals so it would have to be a rational.

CraftyBarbarianKingd ยท 7 points ยท Posted at 19:47:43 on November 21, 2015 ยท (Permalink)

what about 1/0! + 1/1! + 1/2! + .... is that rational too?

mozzarella_past ยท 3 points ยท Posted at 19:48:40 on November 21, 2015 ยท (Permalink)

only true if the sum is finite

[deleted] ยท 3 points ยท Posted at 18:59:04 on November 21, 2015 ยท (Permalink)

Equality of real numbers typically tend to happen "in the reals", yes. 1 and 0.999... are both real numbers by standard convention.

chromeless ยท 0 points ยท Posted at 00:12:00 on November 22, 2015 ยท (Permalink)

1 and 0.999... are both real numbers by standard convention.

Which is my biggest issue whenever this subject is brought up, especially when its explained to laypeople. When people question if 1 = 0.999... they're generally interested in what is essentially a variation on Zeno's paradoxes, the standard conventions and definition of the reals be damned. They typically want to know if and how numbers composed of infinitesimal quantities might be possible. This is actually an interesting problem, one that is usually all to quickly dismissed by assuming we are only discussing the reals and that anything else is an artefact of representation.

https://en.wikipedia.org/wiki/Hyperreal_number

[deleted] ยท 2 points ยท Posted at 17:42:14 on November 22, 2015 ยท (Permalink)

When people question if 1 = 0.999... they're generally interested in what is essentially a variation on Zeno's paradoxes, the standard conventions and definition of the reals be damned.

I doubt that's what's happening. Most people don't think that hard about math, and hold plenty of contradictory beliefs about math at the same time. "Infinity is a number" and "if x is a number then x+1 is a bigger number", "there is a smallest positive ("infinitesimal") number" and "there's always a number between two different numbers", etc. If asked, the same people would be perfectly fine with real numbers.

explorer58 ยท 2 points ยท Posted at 16:08:02 on November 21, 2015 ยท (Permalink)

Can you give an example where you think it isn't true?

ExquisiteViolence ยท 1 points ยท Posted at 19:57:44 on November 28, 2015 ยท (Permalink)

They are different in the hyperreals and other systems that contain infinitesimals, such as the surreal numbers.

GiskardReventlov ยท 1 points ยท Posted at 16:46:21 on November 21, 2015 ยท (Permalink)

Is it true in the p-adics?

[deleted] ยท 2 points ยท Posted at 18:59:23 on November 21, 2015 ยท (Permalink)

0.999... isn't convergent in the p-adics.

viking_ ยท 1 points ยท Posted at 18:03:55 on November 21, 2015 ยท (Permalink)

.999... is true in any base-10 system, is it not? It's a rational number, so extending Q differently doesn't change that fact.

Where it wouldn't be true is in a base other than base 10.

[deleted] ยท 2 points ยท Posted at 08:52:41 on November 21, 2015 ยท (Permalink)*

[deleted]

TwirlySocrates ยท 19 points ยท Posted at 09:11:25 on November 21, 2015 ยท (Permalink)

Because 1 = 0.999...

austin101123 ยท 1 points ยท Posted at 04:26:52 on November 22, 2015 ยท (Permalink)

But that means one factorial would also equal 0.999

I don't see why it would be different.

TwirlySocrates ยท 1 points ยท Posted at 05:30:13 on November 22, 2015 ยท (Permalink)

!= means "not equals" in some programming languages.

austin101123 ยท 1 points ยท Posted at 14:04:16 on November 22, 2015 ยท (Permalink)

Oh shit. I knew that but has never seen it used outside of programming. I just read it is 1! = and not 1 !=

TwirlySocrates ยท 1 points ยท Posted at 17:39:10 on November 22, 2015 ยท (Permalink)

:-)

reduckle ยท 10 points ยท Posted at 09:07:13 on November 21, 2015 ยท (Permalink)

He's saying its true, but intuitively they are not equal.

[deleted] ยท 13 points ยท Posted at 09:11:20 on November 21, 2015 ยท (Permalink)*

[deleted]

reduckle ยท 28 points ยท Posted at 09:31:34 on November 21, 2015 ยท (Permalink)

Yeah. I know its pretty common in programming. not sure about anywhere else.

OceanOfSpiceAndSmoke ยท 6 points ยท Posted at 16:37:12 on November 21, 2015 ยท (Permalink)

It is common anywhere you have/want to write in ASCII. The unequal operators I know of:

!=
<>
~=
/=
=/=
ยฌ=
โ‰ 

Last two aren't ASCII.

nephros ยท 9 points ยท Posted at 18:32:21 on November 21, 2015 ยท (Permalink)

~=

That's ambiguous, it's sometimes is used to mean approximately.

Febris ยท 1 points ยท Posted at 19:16:50 on November 21, 2015 ยท (Permalink)

It comes from the fact that ~ is generally used to negate whatever comes after it.

Krexington_III ยท 1 points ยท Posted at 19:08:10 on November 21, 2015 ยท (Permalink)

Not in MATLAB...

themouseinator ยท 8 points ยท Posted at 23:27:26 on November 21, 2015 ยท (Permalink)

ambiguous

sometimes

elyisgreat ยท 1 points ยท Posted at 20:17:39 on November 21, 2015 ยท (Permalink)

I try to use the last one, because != is not widely recognized outside of programming, and Unicode is well-supported. I prefer != though.

OceanOfSpiceAndSmoke ยท 2 points ยท Posted at 22:29:11 on November 21, 2015 ยท (Permalink)

I guess the reason it is used even when unicode is supported is that people can't be bothered to find the โ‰  character, since it isn't on the keyboard.

noahboddy ยท 0 points ยท Posted at 09:13:18 on November 21, 2015 ยท (Permalink)

1 is 0.1 less than 1.1, and 0.1 greater than 0.9.

1 is 0.01 less than 1.01 and 0.01 greater than 0.99

1 is 0.001 less than 1.001 and 0.001 greater than 0.999

...and so forth.

If 1 = 1.000... (which I assume isn't at issue), and the ... means that you've got infinitely many zeros, so there is no amount which has been added to 1, then likewise 0.999... has infinitely many 9s, and therefore is not less than 1 by any particular amount either.

They are two equivalent notations for the number. The latter is considerably less intuitive, of course.

[deleted] ยท 1 points ยท Posted at 09:51:02 on November 21, 2015 ยท (Permalink)*

[deleted]

Violatic ยท 4 points ยท Posted at 10:18:41 on November 21, 2015 ยท (Permalink)

X = 0.33333333... = 1/3

3x = 0.99999999... = 3/3 =1

There's a few ways to do it, always helpful to be able to explain things different ways to help others understand :)

[deleted] ยท 1 points ยท Posted at 10:22:00 on November 21, 2015 ยท (Permalink)*

[deleted]

Violatic ยท 4 points ยท Posted at 11:04:12 on November 21, 2015 ยท (Permalink)

Let a_1 = 3/10 and r = 1/10

Then just take the infinite geometric series (a_1)/(1-r) = 3/10 * 10/9 = 1/3

More fully on stack exchange: http://math.stackexchange.com/questions/335560/is-1-divided-by-3-equal-to-0-333/335578#335578

oighen ยท 3 points ยท Posted at 15:32:32 on November 21, 2015 ยท (Permalink)

If geometric series are allowed just do the same thing with 9/10n .

Violatic ยท 1 points ยท Posted at 22:14:30 on November 21, 2015 ยท (Permalink)

That's true, but not as simple. Lots of people will accept 1/3 = 0.3333... I don't make the rules ._.

sillypantstoan ยท 2 points ยท Posted at 14:52:20 on November 21, 2015 ยท (Permalink)

There are some subtle issues with this method. Namely, it relies on the fact that 0.999...=1.

How do we know 10*(0.999...) = 9.99...? You're using a pattern that comes from the multiplication algorithm, but you can't use that algorithm with infinite digits, so we have to go deeper. 0.99... is by definition an infinite series (think back to your place value stuff) and you can only multiply through by the 10 like that if you know the series converges, so let's address that first.

We see this as the sum of terms 9*.1n (if n starts at 1), so now we have to backtrack a little further and examine the series .1n (which is the definition of .111....). We recognize this is a geometric sequence which converges to 1/9 (see calculus). So now we have that .111... = .1 + .01 + .001 + ... = 1/9. Since we now have convergence, 9.1 + 9.01 + 9.001+... = 9(.1+.01+.001+...) = 9*1/9 = 1.

Now we could go back to multiplying 10*(0.999...) but we've already proven that 0.999... = 1.

Edit: Fixed the random italicizing caused by my *s

ENelligan ยท 79 points ยท Posted at 02:25:23 on November 21, 2015 ยท (Permalink)

Semantically obvious:

A non-open set is closed.

rampion ยท 90 points ยท Posted at 12:55:12 on November 21, 2015 ยท (Permalink)
WilliamBaronKelvin ยท 17 points ยท Posted at 02:56:55 on November 21, 2015 ยท (Permalink)

Alternatively, a closed set is not open. Or, an open set is not closed.

ranarwaka ยท 16 points ยท Posted at 08:02:58 on November 21, 2015 ยท (Permalink)

"Sets are not doors"

I read it somewhere on MSE, but I forgot the source

TwoFiveOnes ยท 2 points ยท Posted at 16:13:30 on November 21, 2015 ยท (Permalink)

I think it's Munkres

rapgamezezima ยท 8 points ยท Posted at 07:09:05 on November 21, 2015 ยท (Permalink)

God my first analysis course was such a nightmare

ZirconCode ยท 3 points ยท Posted at 11:44:26 on November 21, 2015 ยท (Permalink)

I'm living it now, does it get any better ='(

TheOnlyMeta ยท 5 points ยท Posted at 17:02:34 on November 21, 2015 ยท (Permalink)

Analysis is great fun. I didn't like it at first but the reals just have such weird and interesting structure. I prefer it to algebra or shudder applied.

Orbitir ยท 1 points ยท Posted at 17:24:50 on November 22, 2015 ยท (Permalink)

algebra is where the party is at tho :(

however, on point with applied.

rapgamezezima ยท 2 points ยท Posted at 18:33:04 on November 21, 2015 ยท (Permalink)

It definitely does, but you've gotta get the hang of it, and that took me a little while.

Vainamoinennoumlauts ยท 2 points ยท Posted at 13:51:44 on November 22, 2015 ยท (Permalink)

Specific concepts will be horrible until you get the hang of them and they become trivial. Then you'll be given new, yet again horrible concepts to suffer again. Such is math. :P

ZirconCode ยท 1 points ยท Posted at 13:56:41 on November 22, 2015 ยท (Permalink)

Hahaha, this is what I realized, every week I ask myself why... It's lovely but I can now see why it would drive someone insane xD

Vainamoinennoumlauts ยท 2 points ยท Posted at 23:38:35 on November 22, 2015 ยท (Permalink)
613style ยท 7 points ยท Posted at 06:48:00 on November 21, 2015 ยท (Permalink)

"... if and only if f is a closed map (or equivalently, an open map)."

moschles ยท 2 points ยท Posted at 06:36:26 on November 22, 2015 ยท (Permalink)

Mein Furhrer, "Closed" does not imply "not open".

ranarwaka ยท 1 points ยท Posted at 15:33:42 on November 21, 2015 ยท (Permalink)

Another semantically obvious statement which is false in maths:

"Forests aren't trees"

mattsains ยท 1 points ยท Posted at 11:22:58 on November 22, 2015 ยท (Permalink)

A set which is both non-open and non-closed is simply ajar

belleberstinge ยท 1 points ยท Posted at 19:49:40 on November 22, 2015 ยท (Permalink)

And "half-open" intervals in R like (0, 1] are neither open nor closed.

[deleted] ยท 12 points ยท Posted at 05:23:47 on November 21, 2015 ยท (Permalink)

[deleted]

Exomnium ยท 4 points ยท Posted at 05:50:31 on November 21, 2015 ยท (Permalink)

Wait isn't this impossible by the Lรถwenheimโ€“Skolem theorem?

[deleted] ยท 13 points ยท Posted at 06:06:32 on November 21, 2015 ยท (Permalink)

[deleted]

Exomnium ยท 0 points ยท Posted at 10:22:20 on November 21, 2015 ยท (Permalink)

Oh I get it. You're talking about naively interpreting ZFC with second order logic, right?

[deleted] ยท 6 points ยท Posted at 11:10:48 on November 21, 2015 ยท (Permalink)

[deleted]

Exomnium ยท 1 points ยท Posted at 23:16:39 on November 21, 2015 ยท (Permalink)

I said it badly. I was trying to say that it's like the situation where you can prove that the second order axioms of the real numbers (ordered field with least upper bound property) have a unique model up to isomorphism (in a given set theory) so you might assume that there's a first order set of statements about the real numbers that uniquely specifies the model too, but it doesn't because of Lรถwenheimโ€“Skolem. Similarly you might think that the axioms of ZFC are enough to ensure that, say, there isn't a bijection between N and P(N) (because ZFC can prove that), but really it's only proving that the model doesn't contain a bijection, there still might be one (such as when the entire model is countable).

Krexington_III ยท 1 points ยท Posted at 19:09:19 on November 21, 2015 ยท (Permalink)

I've seen ZFC twice in this thread, and despite me having a master's in math I've never seen it before. What is it? I tried googling but found nothing :/

maffzlel ยท 3 points ยท Posted at 19:30:38 on November 21, 2015 ยท (Permalink)

Try googling Zermelo-Fraenkel

[deleted] ยท 22 points ยท Posted at 17:57:06 on November 21, 2015 ยท (Permalink)*

A curve shape with finite volume must have finite surface area.

Counter-example

DCarrier ยท 6 points ยท Posted at 19:21:22 on November 21, 2015 ยท (Permalink)

Isn't a curve one-dimensional?

[deleted] ยท 1 points ยท Posted at 19:44:32 on November 21, 2015 ยท (Permalink)

Should've used shape. Editing.

elyisgreat ยท 2 points ยท Posted at 20:20:49 on November 21, 2015 ยท (Permalink)

Koch curve is a good 2D example.

No1TaylorSwiftFan ยท 34 points ยท Posted at 08:04:32 on November 21, 2015 ยท (Permalink)

The integral of the derivative of a function is that same function.

There is a good MathOverflow thread about this.

Krexington_III ยท 25 points ยท Posted at 19:10:43 on November 21, 2015 ยท (Permalink)

This seems completely obvious to me -

d/dx(x^2) = 2x
int(2x) = x^2 + C

, C being any constant. Set C =/= 0 and your statement is proven to be correct.

No1TaylorSwiftFan ยท 27 points ยท Posted at 20:53:36 on November 21, 2015 ยท (Permalink)

'The integral of the derivative of a function is that same function, up to an additive constant.' Is also not true in general.

Krexington_III ยท 14 points ยท Posted at 22:50:21 on November 21, 2015 ยท (Permalink)

Really? That's fascinating!

flexmuzik ยท 3 points ยท Posted at 03:40:51 on November 22, 2015 ยท (Permalink)

How come? Are you talking about functions with nasty bits like discontinuity or something?

No1TaylorSwiftFan ยท 7 points ยท Posted at 03:49:49 on November 22, 2015 ยท (Permalink)

Cantor's function is the canonical counter example. It turns out that Cantor's function is continuous everywhere.

kingfrito_5005 ยท 1 points ยท Posted at 03:03:54 on November 22, 2015 ยท (Permalink)

Oh right c. I always forget about c.

almightySapling ยท 14 points ยท Posted at 10:09:30 on November 21, 2015 ยท (Permalink)

The integral of the derivative of a function is that same function.

Do you mean this the other way around? "The integral" is a fairly imprecise concept, and I think we can agree that if f = 1 and g = 2 then the integral of the derivative of f is the integral of the derivative of g but f โ‰  g.

No1TaylorSwiftFan ยท 5 points ยท Posted at 23:33:31 on November 21, 2015 ยท (Permalink)

Even up to an additive constant and almost everywhere equality, there are functions f and g such that f != g but the integral of the derivative of g-f is always 0. See Cantor's function.

themasterofallthngs ยท 5 points ยท Posted at 08:27:17 on November 21, 2015 ยท (Permalink)

How? Isn't that the fundamental theorem of Calculus?

Ex:

Integral of d/dx[x2] = x2

No1TaylorSwiftFan ยท 13 points ยท Posted at 08:44:01 on November 21, 2015 ยท (Permalink)*

See Volterra's function. Another example is Cantor's function.

themasterofallthngs ยท 3 points ยท Posted at 09:45:26 on November 21, 2015 ยท (Permalink)

I think this is beyond my current understanding of math (hopefully not for long), but thank you anyway for replying.

Edit: Actually not that much beyond, now that I think about it.

Krexington_III ยท 5 points ยท Posted at 19:11:19 on November 21, 2015 ยท (Permalink)

Integral of d/dx[x2] = x2 + C

austin101123 ยท 1 points ยท Posted at 04:30:53 on November 22, 2015 ยท (Permalink)

No because you get a +c because you don't know the constant. So you get back something slightly different.

dxtfyuh ยท 1 points ยท Posted at 20:17:19 on November 22, 2015 ยท (Permalink)

The fundamental theorem of calculus requires a continuous derivative IIRC.

LudoRochambo ยท 1 points ยท Posted at 21:55:25 on November 21, 2015 ยท (Permalink)

well this is kind of a broken comment. the integral doesnt end up being valid in the regular sense.

randomdragoon ยท 42 points ยท Posted at 03:25:41 on November 21, 2015 ยท (Permalink)

You can rearrange the terms of an infinite sum and the result will be the same.

Okay, okay, you got me. You can rearrange the terms of an infinite sum that converges to a finite value and the result will be the same.

[deleted] ยท 20 points ยท Posted at 09:26:11 on November 21, 2015 ยท (Permalink)

Doesn't that go against the fact that addition is commutative?

almightySapling ยท 52 points ยท Posted at 10:18:56 on November 21, 2015 ยท (Permalink)

The problem is, a lot of things that work on 2 values can be extending to working on n values for any n, but this doesn't mean that they work on infinite values.

So, what we get is that infinite sums aren't exactly the same as "addition". The notation looks like addition. In spirit it is really close to addition. Addition is a core part of the definition. But really it's a limit, and by rearranging the terms of the series you are looking at limits of completely different sequences of numbers.

[deleted] ยท 2 points ยท Posted at 14:33:09 on November 21, 2015 ยท (Permalink)

I see. Thanks!

almightySapling ยท 6 points ยท Posted at 20:33:31 on November 21, 2015 ยท (Permalink)

Also, you can't just rearrange a few terms and and get a different sum. In order to produce a different sum from a series, you have to rearrange infinitely many terms.

If you only rearrange the first hundred, or million, or billion terms, then commutativity kicks in and the sums converge to the same thing.

ice109 ยท 11 points ยท Posted at 03:31:23 on November 21, 2015 ยท (Permalink)*

Absolutely convergent. And there's no such thing as converging to an infinite value.

meepwn53 ยท 2 points ยท Posted at 20:22:08 on November 21, 2015 ยท (Permalink)

can you please give, or point me to, an example of such infinite sum that can be rearranged to converge to a different value?

EDIT: nvm, here is an example https://en.wikipedia.org/wiki/Riemann_series_theorem#Changing_the_sum

belleberstinge ยท 1 points ยท Posted at 19:54:47 on November 22, 2015 ยท (Permalink)

In a certain sense, given a certain way infinite sums are defined, i.e. as the limit of its partial sums, infinite sums don't have terms.

[deleted] ยท 0 points ยท Posted at 11:15:18 on November 21, 2015 ยท (Permalink)*

[deleted]

jmt222 ยท 1 points ยท Posted at 16:00:20 on November 21, 2015 ยท (Permalink)*

The original statement in quantified form: For any series which converges to a finite value, any rearrangement of the series converges to the same value. OP is saying that this is false and this is indeed so.

If one wants to then claim that this is true, then its truth is not given by a specific example. The example you gave is absolutely convergent so every rearrangement gives the same sum. Also, what you have described is not a rearrangement. A rearrangement of a series is given by a permutation of the indices of its terms.

The original statement is false, and any series which is not absolutely convergent serves as a counterexample. Consider the altenating harmonic series. This converges to ln(2).

We can rearrange the alternating harmonic series as follows: Take the first n positive terms such that their sum is >1, then add the first negative term, which is -1/2 as the (n+1)th term of the rearranged series. Do this again for next m positive terms and then follow it with -1/4, the second negative term.

This is a bona fide rearrangement as each term from the original series appears once. However, this series is bounded below by 1/2+1/2+1/2+... since 1-1/2k>1/2 for every k so the series given this rearrangement diverges to positive infinity, which is certainly not ln 2.

explorer58 ยท 1 points ยท Posted at 16:17:30 on November 21, 2015 ยท (Permalink)

Infinite sums are always defined as a limit, its the only way they make sense. Also, your example is different from what OP was talking about

[deleted] ยท -7 points ยท Posted at 08:19:51 on November 21, 2015 ยท (Permalink)

[deleted]

archiecstll ยท 11 points ยท Posted at 09:33:45 on November 21, 2015 ยท (Permalink)

Even your statement is false. An alternating series which is absolutely convergent cannot be rearranged to converge to a different value.

But the point is that reading the title of the thread is important.

UlyssesSKrunk ยท 200 points ยท Posted at 03:10:09 on November 21, 2015 ยท (Permalink)

It's a commonly believed myth that 1*1 = 1

This, of course, is absurd. It should be obvious, not only to the mathematical elite, but also to the casual observer, that 1*1 = 2.

http://www.independent.co.uk/news/people/terrence-howard-thinks-1x1-2-has-a-secret-system-called-terryology-and-spends-17-hours-a-day-making-10502365.html

OperaSona ยท 102 points ยท Posted at 16:37:22 on November 21, 2015 ยท (Permalink)

"I was always wondering, you know, why does a bubble take the shape of a ball? Why not a triangle or a square? I figured it out."

Should we tell him, guys?

Cyrus296 ยท 13 points ยท Posted at 20:33:08 on November 22, 2015 ยท (Permalink)

A sphere CAN'T have the lowest surface area to volume because the radius is five times the circumference.

OperaSona ยท 8 points ยท Posted at 20:35:08 on November 22, 2015 ยท (Permalink)

And 5*pi=10.

Cyrus296 ยท 11 points ยท Posted at 22:16:10 on November 22, 2015 ยท (Permalink)

Oh my god, pi is the square root of 2

bipocni ยท 200 points ยท Posted at 07:18:40 on November 21, 2015 ยท (Permalink)

My favourite part:

One times one equals two because the square root of four is two, so what's the square root of two? Should be one, but we're told its two, and that cannot be.

How did this guy ever get accepted into a chemical engineering course?

joelschlosberg ยท 173 points ยท Posted at 19:22:18 on November 21, 2015 ยท (Permalink)

Sounds like he has plenty of experience with engineered chemicals.

helfiskaw ยท 18 points ยท Posted at 04:19:41 on November 22, 2015 ยท (Permalink)

The banter is strong in /r/math today

Death_Soup ยท 32 points ยท Posted at 01:45:35 on November 22, 2015 ยท (Permalink)

BY READING YOUR COMMENT AND USING CONTEXT CLUES I CAN INFER THAT YOU ARE IMPLYING THAT HE USES A LARGE AMOUNT OF RECREATIONAL DRUGS

Xeno87 ยท 7 points ยท Posted at 10:11:31 on November 22, 2015 ยท (Permalink)

He probbably didn't:

Howard's account of his educational history has not been confirmed; Pratt Institute, which he says he attended, closed its engineering degree program in 1993.

And he is definitely lying:

On February 26, 2013, Howard said on Jimmy Kimmel Live! that he had earned a Ph.D. in chemical engineering from South Carolina State University that year. Although he was awarded a Doctorate of Humane Letters from SCSU in 2012, he never attended the university and never earned a degree in chemical engineering

That guy is probbably just a crank that can't stand the idea of not being regarded as educated and therefore makes shit up.

jerryFrankson ยท 12 points ยท Posted at 10:43:17 on November 22, 2015 ยท (Permalink)

He is definitely lying

You mean you couldn't figure that out from this:

We're told [the square root of two] is two

No, we're not. We're told the square root of two is 1,41421... because, you know, it is.

Xeno87 ยท 9 points ยท Posted at 11:05:54 on November 22, 2015 ยท (Permalink)

Well there's a serious difference between sucking at high school math and feigning an educational history. I can tolerate the first, but definitely not the latter.

59ekim ยท 82 points ยท Posted at 04:24:25 on November 21, 2015 ยท (Permalink)

What the hell.

UlyssesSKrunk ยท 79 points ยท Posted at 04:25:51 on November 21, 2015 ยท (Permalink)

You just have to believe in your heart.

artifex93 ยท 15 points ยท Posted at 06:41:07 on November 21, 2015 ยท (Permalink)

Heard about this in the Rooster Teeth podcast, haha.

octatoan ยท 46 points ยท Posted at 05:45:29 on November 21, 2015 ยท (Permalink)

So he's a Jaden Smith disciple?

SpaceTimePudding ยท 5 points ยท Posted at 22:09:11 on November 21, 2015 ยท (Permalink)

Sounds more like Jaden Smith's mentor, or maybe they're the same person O.o

octatoan ยท 6 points ยท Posted at 01:36:27 on November 22, 2015 ยท (Permalink)

Well, 1 * 1 is 2 . . .

Pseudoradius ยท 42 points ยท Posted at 22:27:35 on November 21, 2015 ยท (Permalink)

[...] if Pythagoras were around to see this discovery "he would lose his mindโ€.

He got at least one thing right ...

xereeto ยท 35 points ยท Posted at 23:07:02 on November 21, 2015 ยท (Permalink)

One times one equals two because the square root of four is two, so what's the square root of two? Should be one, but we're told its two, and that cannot be.

WHAT

[deleted] ยท 59 points ยท Posted at 15:51:03 on November 21, 2015 ยท (Permalink)

I have 1 pen. If I have 1 lot of 1 pen, how many pens do I have?

Seriously, this guy is actually retarded. How do you even go about making "new logic"?

slower_than_explorer ยท 57 points ยท Posted at 19:15:14 on November 21, 2015 ยท (Permalink)

2 pen

[deleted] ยท 2 points ยท Posted at 19:48:39 on November 21, 2015 ยท (Permalink)*

i don't understand how illogical you have to be for 11 to equal 2. Does 21=2? 3? 4?

fucking reddit formatting screwing my *'s

domuseid ยท 3 points ยท Posted at 21:25:13 on November 21, 2015 ยท (Permalink)

It's very clearly 3!

LordTardus ยท 3 points ยท Posted at 13:24:25 on November 22, 2015 ยท (Permalink)

So 6?

xereeto ยท 2 points ยท Posted at 23:05:23 on November 21, 2015 ยท (Permalink)

you have to escape them with a backslash.

1 * 1 = 2

^ was typed using 1 \* 1 = 2

[deleted] ยท 2 points ยท Posted at 23:14:34 on November 21, 2015 ยท (Permalink)

yeah, I know. But I forgot

buzzabuzza ยท 2 points ยท Posted at 11:53:26 on November 22, 2015 ยท (Permalink)
[deleted] ยท 0 points ยท Posted at 11:56:45 on November 22, 2015 ยท (Permalink)

yes but all logic is logical ergo it derives from the simplest of things, like all maths.

buzzabuzza ยท 1 points ยท Posted at 12:06:49 on November 22, 2015 ยท (Permalink)

I don't get it.

[deleted] ยท 3 points ยท Posted at 12:22:34 on November 22, 2015 ยท (Permalink)

Everything in logic is logical, if it's illogical then it's not logic, ergo everything is derived from the simplest things like 1*1=1. Logic, huh.

Making a "new logic" based on the premise 1*1=2 is inherently illogical.

buzzabuzza ยท 3 points ยท Posted at 12:43:31 on November 22, 2015 ยท (Permalink)

Making a "new logic" based on the premise 1*1=2 is inherently illogical.

Why not? You can make new geometries by changing some axioms (like rejecting Euclid's fifth and making non-euclidean geometries) or new logical systems by rejecting some axiom, for example fuzzy logic that rejects the law of the excluded middle.

rrussell1 ยท 21 points ยท Posted at 11:27:34 on November 21, 2015 ยท (Permalink)

I feel bad for the downvotes, this is hilarious.

agentyoda ยท 14 points ยท Posted at 01:56:34 on November 22, 2015 ยท (Permalink)*

I totally thought this was going to be some clever group theory substitution within an additive ring.

Instead, I get this.

(For those interested, 1*1 does equal 1 in certain groups, I'm pretty sure; I'd have to crack open the algebra book to double check group definition. But that statement in that system means something different from the real number 1 multiplied by the real number 1, so it's a bit of a misnomer.)

EDIT: I meant to say 1*1 = 2 in certain groups, not = 1.

UlyssesSKrunk ยท 6 points ยท Posted at 03:01:21 on November 22, 2015 ยท (Permalink)

1*1 does equal 1 in certain groups

Nope, not buying it

agentyoda ยท 2 points ยท Posted at 03:05:39 on November 22, 2015 ยท (Permalink)

Whoops, typo.

I meant 1*1 = 2 in certain groups, haha. Edited.

ziggurism ยท 2 points ยท Posted at 14:21:40 on November 22, 2015 ยท (Permalink)

Still not buying it.

agentyoda ยท 1 points ยท Posted at 15:13:19 on November 22, 2015 ยท (Permalink)*

First of all, a*b is notation for an operation on a group. It just so happens that we associate it also with the multiplicative operation for the real numbers (also for rings).

If we define a group over the integers with the following operation: a*b = a + b, then we have:

0 as the identity element: x*0 = x for all x

1*1 = 2

Really basic example. You can make it more complicated, if you want:

a*b = 2ab yields the same thing.

Like I said, its a misnomer, since 1*1 would not be referring to multiplication of the real numbers 1 and 1 but an operation on a group.

However, that's how the notation generally looks, so we really can say 1*1 = 2 in the additive group of integers. Since you're just adding them together.

ziggurism ยท 4 points ยท Posted at 15:20:56 on November 22, 2015 ยท (Permalink)

As long as * is a group operation, and 1 is identity element with respect to that operation, 1*1=1. But yes, if you do not follow that convention, you may redefine anything you want, including mixing multiplicative notation for the operation, with additive notation for the identity element. May as well just define 2 to be another name for 1, then the equation is true in all groups.

agentyoda ยท 2 points ยท Posted at 15:52:21 on November 22, 2015 ยท (Permalink)

and 1 is identity element with respect to that operation

Right, but there are groups whose identity element equal zero, as shown above. Remember that groups are defined around a single operation *, which can be anything that fits the axioms: doesn't need to be the multiplicative operation. It's only when you get to rings that 1 is guaranteed to be the identity element, since * means strictly the multiplicative operation in rings.

May as well just define 2 to be another name for 1, then the equation is true in all groups.

All groups where 1 is the identity element, yes.

ziggurism ยท 2 points ยท Posted at 16:42:54 on November 22, 2015 ยท (Permalink)

I think it's perverse that you use the multiplication symbol to represent addition. But I concede that you can use any symbol to represent anything you want, as long as you define your terms.

In that light, I dispute your statement that in rings 1 is guaranteed to be the multiplicative identity. I can define a ring where 0 is the multiplicative identity and 1 is the additive identity.

agentyoda ยท 2 points ยท Posted at 17:08:17 on November 22, 2015 ยท (Permalink)

I think it's perverse that you use the multiplication symbol to represent addition.

Group theory textbooks (the two I read, anyway) use the symbol * to denote "the operation on the group." a*b thus represents a (operation) b for the elements a, b in the group. There's nothing perverse about it if said operation is addition. Happened in plenty of additive group examples. Not to mention I also gave another example of a non-additive operation which still yields the result 1 operation 1 = 2 for the group operation.

In that light, I dispute your statement that in rings 1 is guaranteed to be the multiplicative identity.

You're right, I'm sure; didn't think the statement through. I simply wanted to emphasize that there are groups with an identity element not equal to one.

ziggurism ยท 1 points ยท Posted at 18:33:08 on November 23, 2015 ยท (Permalink)

Yeah, if you don't know anything about the group operation (for example in a group theory textbook where you want to make statements that apply to all groups), then you may use * for the group operation. But if you use *, you must also use 1 for the identity element! No group theory textbook is going to use * for the operation but 0 for the identity, or use 1 for anything other than the identity. Together (* for operation, 1 for identity) this is called multiplicative notation. Alternatively, you may use + for the operation, and 0 for the identity (preferably for a commutative operation), but what you should not do is mix and match.

agentyoda ยท 1 points ยท Posted at 20:33:31 on November 23, 2015 ยท (Permalink)

But if you use *, you must also use 1 for the identity element! No group theory textbook is going to use * for the operation but 0 for the identity, or use 1 for anything other than the identity.

Demonstrably false. My abstract algebra textbook (I'll link it when I find it at home) used * for the group operation and e for the identity element in a group, even if that group had an identity element different from one. This included groups like this one. The identity element e is, as he stated, 1/2 in that example.

ziggurism ยท 1 points ยท Posted at 16:46:38 on November 22, 2015 ยท (Permalink)*

All groups where 1 is the identity element, yes.

1*1=2 is not an equation that's true in all groups where 1 is the identity element. For example, it's not true in the multiplicative group of rational numbers. It is true in all groups where we've redefined the symbol 2 to be another name for the symbol 1 (which was my point), and also in groups where we've redefined the multiplication symbol to be another name for addition (which was apparently your point).

agentyoda ยท 1 points ยท Posted at 17:05:26 on November 22, 2015 ยท (Permalink)

It is true in all groups where we've redefined the symbol 2 to be another name for the symbol 1

...which was what I meant by the passage you quoted. I think we're on the same page here, haha.

frog971007 ยท 1 points ยท Posted at 04:09:55 on November 23, 2015 ยท (Permalink)

At that point wouldn't you be able to define anything to be true just by renaming every symbol?

agentyoda ยท 1 points ยท Posted at 04:56:27 on November 23, 2015 ยท (Permalink)

Yes, but I think the main point of this is getting mixed.

When people see (a)(b), they think it means "a" multiplied by "b". But what is actually means is an operation of "a" and "b" in whatever group "a" and "b" are in. It's just that, when most people are talking about the real numbers, they're using the multiplicative operation. In reality, you can use any number of operations for that, so that (1)(1) = 2 or (0)(5) = 5 can legitimately be true in a group defined on that operation.

In other words, we're not talking about renaming symbols. We're talking about different kinds of groups of numbers instead of the real numbers on the multiplicative operation.

That was the point of the "intuitively obvious mathematical statement" joke, but it's been long since muddled up.

Gwodmajmu ยท 3 points ยท Posted at 02:59:12 on November 23, 2015 ยท (Permalink)

Sure, take (Q\{0},*) where a*b=2ab. This is closed and associative. For the identity, we want an e such that a*e = a. So 2ae = a, which implies e = 1/2. Additionally, any element a has an inverse by solving a*x = e. We have a*x = 2ax = 1/2, so x=1/(4a). This is defined for all nonzero rational numbers.

In this group, 1*1 = 2(1)(1) = 2.

Of course, if by "1", they meant the identity element, then we always have e*e = e in any group.

jerkandletjerk ยท 4 points ยท Posted at 07:38:44 on November 22, 2015 ยท (Permalink)

I expected it to be something like Ramanujan's summation. But this was far more beautiful!

59ekim ยท 0 points ยท Posted at 22:52:55 on November 21, 2015 ยท (Permalink)

Watching the last episode of south park, I think that's just something they made up to spark up attention for that tv show.

_jbass ยท 7 points ยท Posted at 00:49:32 on November 22, 2015 ยท (Permalink)

You cannot turn a sphere inside out without puncturing it.

5225225 ยท 2 points ยท Posted at 19:13:07 on April 6, 2016 ยท (Permalink)

Assuming you have a material that can pass through itself.

I think you can safely say you can't if you don't have that condition.

orbital1337 ยท 13 points ยท Posted at 16:36:13 on November 21, 2015 ยท (Permalink)

Every continuous function is somewhere differentiable.

Parzival_Watts ยท 4 points ยท Posted at 13:08:55 on November 22, 2015 ยท (Permalink)

I love that one. The counter example (the Weierstrass Function) is what spawned my interest in math.

_God____ ยท 2 points ยท Posted at 19:14:52 on November 21, 2015 ยท (Permalink)

Where is there a counterexample?

DCarrier ยท 7 points ยท Posted at 19:23:08 on November 21, 2015 ยท (Permalink)
archiecstll ยท 6 points ยท Posted at 09:29:13 on November 21, 2015 ยท (Permalink)

The compliment of any embedding of the Cantor set into S3 has trivial fundamental group.

Mayer-Vietoris ยท 4 points ยท Posted at 15:12:03 on November 21, 2015 ยท (Permalink)

Wait that seems intuitively false to me...

archiecstll ยท 1 points ยท Posted at 16:58:59 on November 21, 2015 ยท (Permalink)

In nontechnical terms, my thought for why some may find it intuitively obvious was that one may reason that any circle which loops around the embedded Cantor set may be removed from it through the gaps between the points.

However, one's intuition is distinct from another's.

Mayer-Vietoris ยท 1 points ยท Posted at 05:21:48 on November 22, 2015 ยท (Permalink)

Ahh fair enough. I suppose that the 'normal' embedding has the stated property. I just think of the cantor set as a universal compact metric space. If there is a continuous surjection of the cantor set onto any compact metric space...

oighen ยท 1 points ยท Posted at 00:06:43 on November 22, 2015 ยท (Permalink)

So, you take the classic "one" dimensional cantor set, put it in S3 and it's fundamental group becomes non trivial? How does it happen? What group does it have?

Exomnium ยท 1 points ยท Posted at 00:44:44 on November 22, 2015 ยท (Permalink)

The fundamental group in these cases is complicated but here's an example.

archiecstll ยท 1 points ยท Posted at 00:51:41 on November 22, 2015 ยท (Permalink)

It's not my area of expertise, but one construction I know is with a sequence of nested chains of solid tori. For the first step, embed a solid 1-holed torus into S3. At the (k+1)-th step, for each solid torus T in the k-th step, take a circular chain of linked solid 1-holed tori (think of something like a physical chain where the links in the chain form a circle) and embed the chain into T so that the chain wraps around the hole of T once. Let A_n be the union of all the solid tori in the n-th stage. The embedded Cantor set C is the intersection of the A_n. (There may be a requirement that the diameter of the tori decrease to zero, but again, this is not my area of expertise.)

A simple loop in S3-C which passes once through the hole of the solid torus in the first step of the construction of C cannot be contracted to a point in S3 without passing through C.

DCarrier ยท 5 points ยท Posted at 19:27:36 on November 21, 2015 ยท (Permalink)

If you take a ball and cut it into five pieces, and reassemble them into two balls, they're not going to be as big as the first ball.

Every theorem can be either proven or disproven.

A number can't be both real and imaginary.

Dave37 ยท 3 points ยท Posted at 20:35:29 on November 21, 2015 ยท (Permalink)

A number can't be both real and imaginary.

How is that not true? Could you give me an example?

person594 ยท 7 points ยท Posted at 21:01:14 on November 21, 2015 ยท (Permalink)

0

DCarrier ยท 3 points ยท Posted at 21:19:04 on November 21, 2015 ยท (Permalink)

Zero.

GaryMutherFuckinOak ยท 2 points ยท Posted at 20:38:38 on November 21, 2015 ยท (Permalink)

I believe that 0 is both an imaginary and a real number

Dave37 ยท 2 points ยท Posted at 20:45:46 on November 21, 2015 ยท (Permalink)

Yea that's true. So it's not as much A number as it is One number. :)

LudoRochambo ยท 1 points ยท Posted at 22:09:45 on November 21, 2015 ยท (Permalink)

"give me a counter-example"

does not mean give me the ONE counter-example. unless you were making a joke with One being Zero-ne (0ne).

Dave37 ยท 1 points ยท Posted at 02:23:43 on November 22, 2015 ยท (Permalink)

I just meant that there isn't any other number apart from 0 that's both imaginary and real.

LudoRochambo ยท 1 points ยท Posted at 02:25:22 on November 22, 2015 ยท (Permalink)

sure there is, any real number.

Dave37 ยท 3 points ยท Posted at 02:27:40 on November 22, 2015 ยท (Permalink)

No...(?) 2 is both real and complex, but it's not real and imaginary.

LudoRochambo ยท 1 points ยท Posted at 02:51:12 on November 22, 2015 ยท (Permalink)

ah you do it that way. ive always gone by imaginary/complex are the same, and imaginary doesnt necessarily need a non-zero i term.

guess that was just lazy instruction :D

Dave37 ยท 2 points ยท Posted at 11:21:06 on November 22, 2015 ยท (Permalink)

According to wikipedia, a imaginary number is any real number multiplied by i.

moschles ยท 6 points ยท Posted at 07:08:18 on November 22, 2015 ยท (Permalink)

"Transcendental numbers are special, and therefore appear very rarely on the real number line."

Daimanta ยท 14 points ยท Posted at 22:17:51 on November 21, 2015 ยท (Permalink)

There are more fractions than whole numbers.

plumpvirgin ยท 5 points ยท Posted at 02:44:10 on November 22, 2015 ยท (Permalink)

That's just because cardinality isn't the notion of size that most people have in mind when they talk about the "size" of a set (before taking a set theory course, anyway). There are many different (all very valid) ways of comparing the sizes of infinite sets.

I would argue that when people think things like "there are more rationals than integers" or "there are more integers than even integers", they have something like natural density (not cardinality) in mind, and that's absolutely fine. Telling them that they're "wrong" and that cardinality is the only measure of size is very counter-productive.

[deleted] ยท 1 points ยท Posted at 04:46:25 on November 22, 2015 ยท (Permalink)

How is this not true? Of course there are.

CraftyBarbarianKingd ยท 2 points ยท Posted at 05:37:24 on November 22, 2015 ยท (Permalink)

This depends on how we "count" sets. For example take the sets {a,b,c} and {1,2,3} you can see they have the same amount of elements just by counting, but that strategy doesn't hold up well with infinite sets so what you can do is you match the two sets. That is you could match a with 1, b with 2 and c with 3, this way you know they have the same size, if you can match all the elements from both sets together. Going back to integers, or the positive integers for now, if you want to match a set with the integers this mean you assign some element to 1, another for 2, 3,etc that is you put them in a list. So anything that can be put in a infinite list without repeating elements has the same amount of elements as the positive integers. here is how you can list all of the rational numbers with the integers. What's even more mind blowing is that actually you can't list all the real numbers, here is a video showing why.

jerkandletjerk ยท 2 points ยท Posted at 07:41:53 on November 22, 2015 ยท (Permalink)

Integer, fraction, and natural number sets have a 1 to 1 correspondence, which is why they are 'equal'.

[deleted] ยท 1 points ยท Posted at 12:59:18 on November 22, 2015 ยท (Permalink)

Why put quotes around equal? Because you're using a pretty unintuitive definition of equal.

jerkandletjerk ยท 2 points ยท Posted at 14:44:31 on November 22, 2015 ยท (Permalink)

Because intuitively, there's infinite fractions between two integers, yet the sets of integers and fractions are 'equal.' There, I put quotes again.

[deleted] ยท 1 points ยท Posted at 20:37:52 on November 22, 2015 ยท (Permalink)

So then you're simply using an unintuitive definition of equal. So this is kind of ridiculous to use as something that intuitively seems true but is actually false. I'm sure I could easily come up with a definition of many or equal that results in there being more rationals.

On an unrelated note, is there a way to show 1-1 correspondence between integers and rationals that includes numbers above 1? The only way I have seen is:

1, 1/2, 1/3, 2/3, 1/4, 3/4, etc

[deleted] ยท 1 points ยท Posted at 11:34:03 on February 10, 2016 ยท (Permalink)

This is two months late, but there is:
If you have a sequence containing all the rationals below 1, you can take their inverses to get the rationals above 1. Then you can simply zip the two into a single sequence.

If I take your sequence 1, 1/2, 1/3, 2/3, 1/4, 3/4, ...
We can take the inverses: 1, 2, 3, 3/2, 4, 4/3, ...
Then mix them together (I'll remove 1, since it's in both of them, and add 0)
0, 1, 1/2, 2, 1/3, 3, 2/3, 3/2, 3/4, 4/3, ...

You can also add the negative rationals the same way if you want.

And if you don't like the fact that the enumeration isn't 'explicit', you may find the Stern-Brocot sequence interesting.

The sequence is defined as:
a(0) = 0; a(1) = 1
for n>0: a(2*n) = a(n); a(2*n+1) = a(n) + a(n+1)

Then the sequence a(n)/a(n+1) is a bijection between the integers and the non negative rationals. (no missing rationals, and no repeats, either.)

The sequence is closely related to the Stern-Brocot Tree, which enumerates the rationals using the same idea.

[deleted] ยท 2 points ยท Posted at 12:35:58 on February 10, 2016 ยท (Permalink)

If you have a sequence containing all the rationals below 1, you can take their inverses to get the rationals above 1.

This is incredible. Thanks for the post, really appreciate it.

[deleted] ยท 1 points ยท Posted at 12:35:43 on February 10, 2016 ยท (Permalink)

If you have a sequence containing all the rationals below 1, you can take their inverses to get the rationals above 1.

This is incredible. Thanks for the post, really appreciate it.

Mayer-Vietoris ยท 2 points ยท Posted at 22:28:22 on November 22, 2015 ยท (Permalink)

The cardinality of a set is a fairly standard notion in the math community of size for infinite sets. It's a nice generalization of the notion of the 'number of elements' in a finite set. You are, of course, free to define your own, non-standard notion of the size of an infinite set which makes the rationals and the integers have different sizes as sets, but you would have to specify that you were using that notion before hand. It turns out that the rationals and the naturals have the same cardinality though.

One of the reasons mathematicians are happy with the use of cardinality is because it very simply states when you can use one set to list off the elements of another set. Because they are in one-to-one correspondence each element of one of the sets corresponds to an element of the other set, so you can use the one set to label the elements of the other set uniquely.

moschles ยท -1 points ยท Posted at 06:49:09 on November 22, 2015 ยท (Permalink)

This beautiful example should be voted to the top.

[deleted] ยท 4 points ยท Posted at 19:22:03 on November 21, 2015 ยท (Permalink)*

[deleted]

boarhog ยท 4 points ยท Posted at 21:14:26 on November 21, 2015 ยท (Permalink)

It's intuitively possible to take one sphere and make two, they will simply end up smaller.

[deleted] ยท 1 points ยท Posted at 23:34:49 on November 21, 2015 ยท (Permalink)*

[deleted]

boarhog ยท 2 points ยท Posted at 06:26:40 on November 22, 2015 ยท (Permalink)

I know, good old Banach-Tarski paradox

Mayer-Vietoris ยท 1 points ยท Posted at 22:30:23 on November 22, 2015 ยท (Permalink)

Even with only rigid motions? You aren't allowed to bend or stretch any part of the sphere, only rotate and translate. I would intuitively expect sharp angles if you were trying to make a smaller sphere.

jam11249 ยท 1 points ยท Posted at 02:55:56 on November 30, 2015 ยท (Permalink)

Trivially, if you let yourself cut up the sphere into uncountably many sets (i.e. singletons) you could turn a sphere into two smaller ones with only rigid motions.

Mayer-Vietoris ยท 1 points ยท Posted at 03:47:34 on November 30, 2015 ยท (Permalink)

I was assuming that we were only cutting into finitely many pieces. If you do take singletons why are the spheres smaller?

jam11249 ยท 1 points ยท Posted at 04:07:21 on November 30, 2015 ยท (Permalink)

Well they don't have to be smaller. Two spheres have the same cardinality as one, so by rigidly deforming each point separately there's no issues. This is why I've never found Banach-Tarski massively outrageous. If you want to take finitely many sets, the sets aren't measurable, so that's "cheating" just the same as using uncountably many sets.

darksingularity1 ยท 3 points ยท Posted at 15:23:22 on November 22, 2015 ยท (Permalink)

There are more numbers between 1.0 and 100.0 than there are between 1.0 and 2.0. They are both technically infinite.

PurelyApplied ยท 7 points ยท Posted at 21:25:19 on November 21, 2015 ยท (Permalink)

If a function f is continuous on [a,b] and f(a) < f(b), then there exists some point c in [a,b] where f'(c) > 0.

It's, like, a corollary to the Mean Value Theorem or something.

[Counterexample: The Devil's Staircase]

LudoRochambo ยท 1 points ยท Posted at 22:14:56 on November 21, 2015 ยท (Permalink)

never heard of this, but its so clear as something to come up with.

fuck that shit, ugh. that made me angry, lol.

ice109 ยท 1 points ยท Posted at 23:19:11 on November 21, 2015 ยท (Permalink)

Umm there do exist points for which f'(c)>0, it's just that their collection has measure zero.

magus145 ยท 5 points ยท Posted at 23:42:30 on November 21, 2015 ยท (Permalink)

The derivative is 0 at any point not in the Cantor Set. The function is not differentiable anywhere on the Cantor Set.

[deleted] ยท 1 points ยท Posted at 16:04:14 on November 22, 2015 ยท (Permalink)

This is a good one.

dxtfyuh ยท 1 points ยท Posted at 20:19:53 on November 22, 2015 ยท (Permalink)

True is the function is differentiable though.

imaami ยท 12 points ยท Posted at 10:38:27 on November 21, 2015 ยท (Permalink)

-1 == UINT_MAX

...I'll get my coat.

cdsmith ยท 7 points ยท Posted at 10:58:47 on November 21, 2015 ยท (Permalink)

(mod UINT_MAX + 1)

cockmongler ยท 9 points ยท Posted at 12:36:11 on November 21, 2015 ยท (Permalink)

In C signed integers aren't guaranteed to overflow like unsigned integers are.

FUZxxl ยท 2 points ยท Posted at 15:26:02 on November 21, 2015 ยท (Permalink)

In conversion between signed and unsigned integers the overflow that might occur is somewhat well-defined, OPs statement is actually true.

FUZxxl ยท 1 points ยท Posted at 15:25:31 on November 21, 2015 ยท (Permalink)

AT least in C this alsways holds.

PPewt ยท 2 points ยท Posted at 17:31:35 on November 21, 2015 ยท (Permalink)*

There are lots of examples in real analysis. To follow up on the "closure of the open ball" one, some others that come to mind:

If you have two balls B_1 and B_2 such that the radius of B_1 is strictly larger than that of B_2, then you can't contain B_1 strictly in B_2.

If a sequence converges componentwise then it also converges under your favourite norm.

If you have a nested sequence of closed balls in a complete metric space with strictly decreasing radius, their intersection is nonempty.

oighen ยท 1 points ยท Posted at 00:12:20 on November 22, 2015 ยท (Permalink)

Care to explain them?

PPewt ยท 1 points ยท Posted at 01:23:56 on November 22, 2015 ยท (Permalink)

Counterexample for #1: Let your space be the naturals with the usual distance. Take B_1 = B( 1 ; 1.9 ) and B_2 = B( 2 ; 1.1 ). Then B_1 = { 1, 2 } and B_2 = { 1, 2, 3 }

Counterexample for #2: Let your space be the union from n=1 to infinity of Rn, where every vector is followed by infinitely many zeroes. This is a vector space (verifying this is easy). Take the sequence {x_n} s.t. the nth coordinate of x_n is 1 and all other coordinates are zero. This sequence converges componentwise to the zero vector, but the distance between each term and the zero vector stays the same under most norms.

Counterexample for #3: Take the naturals for your set and use the distance function d(x,y) = 1 + min(x,y)-1 (it's immediate that this space is complete, given that every point is isolated), and consider a sequence of closed balls where B_n has radius equal to 1 + n-1. These balls contain all numbers m in the naturals s.t. m โ‰ฅ n. Therefore they're clearly nested and yet their intersection is empty.

AluminiumSandworm ยท 2 points ยท Posted at 08:29:44 on November 22, 2015 ยท (Permalink)

If something with a finite probablity of happining is given enough time, it almost certainly will happen. Another calc II example.

speedyjohn ยท 2 points ยท Posted at 21:47:02 on November 22, 2015 ยท (Permalink)

Is this not true? I mean, it's not guaranteed to happen, but doesn't the probability approach 1?

AluminiumSandworm ยท 1 points ยท Posted at 22:17:15 on November 22, 2015 ยท (Permalink)

Not necessarily. It's possible to have the limit of a series go to infinity, but the series itself adds up to a finite number. That's when the series is considered convergent. Note that this only works for whole numbers, but you can measure time in units of planks, which is a discrete quantity, so it holds for probablity in the universe.

speedyjohn ยท 1 points ยท Posted at 00:56:24 on November 23, 2015 ยท (Permalink)

So the probability of this event is not constant?

AluminiumSandworm ยท 1 points ยท Posted at 05:50:34 on November 23, 2015 ยท (Permalink)

Yes. If it is constant, then it does approach 1.

speedyjohn ยท 2 points ยท Posted at 06:01:34 on November 23, 2015 ยท (Permalink)

Well if the probability decreases with time it doesn't seem so shocking that the event isn't sure (or even likely) to happen.

barbadosslim ยท 1 points ยท Posted at 07:20:54 on February 21, 2016 ยท (Permalink)

A 3D random walk is a pretty good example of this

entp_adrone ยท 4 points ยท Posted at 14:46:45 on November 21, 2015 ยท (Permalink)

A non negative number is positive, or similarly, a non positive number is negative

OceanOfSpiceAndSmoke ยท 6 points ยท Posted at 16:47:53 on November 21, 2015 ยท (Permalink)

This is wrong because zero is neither positive or negative?

boarhog ยท 4 points ยท Posted at 21:15:53 on November 21, 2015 ยท (Permalink)

Then why does my thermometer sometimes show -0ยฐC?

LudoRochambo ยท 11 points ยท Posted at 22:16:24 on November 21, 2015 ยท (Permalink)

thanks obama.

rileyrulesu ยท 1 points ยท Posted at 22:17:39 on November 21, 2015 ยท (Permalink)

Also complex and imaginary numbers.

[deleted] ยท 1 points ยท Posted at 17:58:59 on November 21, 2015 ยท (Permalink)

All sets in R can be assigned a Lesbegue measure.

jwktiger ยท 5 points ยท Posted at 22:24:06 on November 21, 2015 ยท (Permalink)

non measurable sets are messed up

Pseudoradius ยท 3 points ยท Posted at 22:45:24 on November 21, 2015 ยท (Permalink)

Of course they are. After all, they don't conform to any standards.

Mayer-Vietoris ยท 2 points ยท Posted at 22:37:03 on November 22, 2015 ยท (Permalink)

There are some models for ZF in which all sets of real numbers are Lesbegue measurable. The existence of non-measurable sets seems to depend pretty critically on the axiom of choice.

sonicandfffan ยท 1 points ยท Posted at 15:23:03 on November 28, 2015 ยท (Permalink)

There are more fractions than whole numbers

GreenfromThat70s ยท 1 points ยท Posted at 00:16:14 on December 24, 2015 ยท (Permalink)

In infinite dimensional Hilbert spaces, the unit sphere is not a deformation retract of the closed unit ball.

Ostrololo ยท -1 points ยท Posted at 10:34:46 on November 21, 2015 ยท (Permalink)

It is obvious that the only meaningful value you can assign to the infinite sum 1+2+3+4+... is infinity.

Vietoris ยท 21 points ยท Posted at 15:19:07 on November 21, 2015 ยท (Permalink)

I understand what you want to say, but the word "meaningful" is too vague to be ... meaningful.

I can describe a useful method to associate values to some objects. I can call the value associated to a particular object the "infinite sum" of that object. That does not mean that it has anything to do with the intuitive meaning of the word "sum".

samloveshummus ยท 3 points ยท Posted at 15:45:41 on November 21, 2015 ยท (Permalink)

But various procedures for summing divergent series are generalizations of procedures that give the expected answers in the uncontroversial cases. They're not just arbitrary assignments of meaning to the summation symbol. Thus, I don't think it's right to say that the rules have nothing to do with honest-to-God summation.

grizzlebritches ยท 6 points ยท Posted at 14:40:29 on November 21, 2015 ยท (Permalink)

I still don't see where it becomes -1/12. Seems more like a "magic, got it" answer

meem1029 ยท 1 points ยท Posted at 21:39:33 on November 21, 2015 ยท (Permalink)*

There's a certain set of infinite sequences that are useful sometimes. To save time from adding an infinite number of elements for them each time, we define a function based on a parameter that tells us which sequence of this family we are looking at. 1+2+3+... takes the form of this family, but is not a member because it doesn't make sense to sum it to infinity and have it converge. If we completely ignore sanity and pretend that the function works here, we get -1/12.

To go slightly more in depth, the family of functions is the sums of the form 1/(ns ) summed from 1 to infinity, with s as the parameter. 1+2+3+... can be written as such with s = -1. The function we look at for this is the Riemann zeta function. The primary problem here is assuming that just because the function that gives the right value for nicely formed members of the set extends to anything means that it's meaningful.

[deleted] ยท 1 points ยท Posted at 21:41:59 on November 21, 2015 ยท (Permalink)

http://math.stackexchange.com/questions/437883/what-is-the-analytic-continuation-of-the-riemann-zeta-function

Basically you make the function Z that gives you the sum of all natural numbers to the input power. So Z(2) = 1 + 1/4 + 1/9.... And Z(-1) = 1 + 2 +3 + 4..., but that's not defined, so what we do is "connect the dots" based on the other values of the function that make sense. There's a natural way to connect the dots which then gives us Z(-1) = -1/12

Blieque ยท -10 points ยท Posted at 16:09:06 on November 21, 2015 ยท (Permalink)

There's a good Numberphile video about it. There's also a link in the description to an alternative proof using the zeta function.

TheOnlyMeta ยท 7 points ยท Posted at 17:27:59 on November 21, 2015 ยท (Permalink)

That video is the reason there is so much misunderstanding about it. Just terribly explained. Uses magic (read: incorrect logic) to get the answer he wants.

The "alternative proof" using the zeta function is the only proof. Because the sum doesn't converge, we only assign it the value -1/12 via the analytic continuation of the zeta function. If zeta isn't there you're being fooled.

Blieque ยท -2 points ยท Posted at 18:33:58 on November 21, 2015 ยท (Permalink)

Could you explain further? I've watched both videos, and the one I linked made sense to me. What about it is invalid? What part of it is magic?

TheOnlyMeta ยท 6 points ยท Posted at 20:11:30 on November 21, 2015 ยท (Permalink)

Well he starts off saying 1 - 1 + 1 - ... = 1/2 which isn't Kosher anyway. But the important point is that addition isn't commutative in infinite series (this is even a parent comment in this thread!), so when he goes adding and subtracting stuff it's meaningless for convergent series, let alone non-convergent series.

grizzlebritches ยท 2 points ยท Posted at 19:09:12 on November 21, 2015 ยท (Permalink)

Watched both. Still think it's voodoo

LudoRochambo ยท 2 points ยท Posted at 22:07:14 on November 21, 2015 ยท (Permalink)

the harmonic series is famous for being divergent, but super super slow. (1+1/2+1/3+1/4+1/5+1/6+....) is not bounded.

if you alternate the signs, it does converge (1-1/2+1/3-1/4+1/5-1/6+...). it actually converges to log2, but whatever, irrelevant.

youre always taught that if you rearrange numbers, it stays the same. 1+2+3 = 1+3+2 = 2+3+1, etc

back to 1-1/2+1/3-1/4+1/5-1/6+... if you start rearranging this like (1+1/3+1/5)-1/2+(1/7+1/9+1/11+1/13)-1/4 except in better ways then you can change what it converges to. in THIS particular example its based off knowing the harmonic diverges but it sucks typing out an explanation.

the jist is you can't simply rearrange alternating infinite sums. the "voodoo" in the video is because they do, so theyre free to basically conclude anything they want. its broken logic. the ONLY way to justify the video is through the analytic continuation of the zeta function which forces the sum to be -1/12. the zeta function is the harmonic, but for complex numbers and powers. so, the zeta function generalized the harmonic sum.

its not voodoo so much as someone rambling about nonsense. they start with 1-1+1-1+1-....=1/2 because they are unsure if its 0 or 1. you can argue -1+1-1+1-....should be -1/2 and derive a totally different number. in fact you can take this farther and say the general population would begin writing this with a +1 over a -1 as the first term, lets say 75% of the people. therefore probabilistically, 1-1+1-...=0.75 = 3/4 and derive a different sum.

its just crap

CrashOverride_ ยท 1 points ยท Posted at 02:40:19 on November 23, 2015 ยท (Permalink)

You could do Ramanujan_summation.

entp_adrone ยท 2 points ยท Posted at 14:48:35 on November 21, 2015 ยท (Permalink)

That a number divided by zero is infinity. This seems to make intuitive sense, because as the denominator approaches zero the value increases. However infinity is not an algebraically manipulable number like the statement would imply.

entp_adrone ยท 1 points ยท Posted at 22:36:23 on November 21, 2015 ยท (Permalink)

Getting downvotes here with no explanation. I heard this idea on numberphile here, could somebody explain why 1/0 does equal infinity?

[deleted] ยท 1 points ยท Posted at 02:50:59 on November 30, 2015 ยท (Permalink)

The if you take the limit it is infinity

entp_adrone ยท 1 points ยท Posted at 03:47:42 on November 30, 2015 ยท (Permalink)

1/0 is not equal to the limit of 1/0. The value itself is not infinity. We wouldn't say that in other situation, like that (sin0)/0 equals 1. Only the limit of (sin0)/0 equals 1. The difference matters

Money_bigshotxx ยท 1 points ยท Posted at 23:21:08 on November 21, 2015 ยท (Permalink)

0.9 repeating doesn't equal 1

Kotsoumpis ยท 0 points ยท Posted at 13:52:30 on November 21, 2015 ยท (Permalink)*

ฮง0 is 0. When I first saw that I was sure it was true! Edit: Grammar

LudoRochambo ยท 1 points ยท Posted at 22:16:05 on November 21, 2015 ยท (Permalink)

x0 or do you mean 00 ? why would x0 be 0?

Kotsoumpis ยท 1 points ยท Posted at 22:22:33 on November 21, 2015 ยท (Permalink)

Well when I first saw x0 I was like" hmm so we have x multiplied by it self 0 times so we have no x therefore x0 = 0". This was MY intuition and according to a fairly small amount of people around me their's too.

xereeto ยท 1 points ยท Posted at 23:15:32 on November 21, 2015 ยท (Permalink)

That would only make sense if X1 meant X was multiplied by itself 1 time...

Kotsoumpis ยท 2 points ยท Posted at 23:36:54 on November 21, 2015 ยท (Permalink)

Touche. Let me rephrase."hmm so we have "x" 0 times so we have no x therefore x0 = 0". How about now?

xereeto ยท 2 points ยท Posted at 23:38:25 on November 21, 2015 ยท (Permalink)

Makes way more sense, and I can totally see where you're coming from. I had the same misconception too.

ck2839 ยท 1 points ยท Posted at 02:35:39 on November 22, 2015 ยท (Permalink)

By going down x2, x1, etc, you keep dividing by x, so x0 = x1 / x =1 should make more sense intuitively.

[deleted] ยท -1 points ยท Posted at 00:48:39 on November 22, 2015 ยท (Permalink)

[deleted]

imstarlordman ยท 3 points ยท Posted at 01:09:28 on November 22, 2015 ยท (Permalink)

How does this show that "there are as many even numbers as odds" is false? I do not follow.

criangulien ยท 2 points ยท Posted at 01:48:01 on November 22, 2015 ยท (Permalink)

f(n)=f(n+1) is a one-to-function from the even numbers to the odd numbers. I don't get your point ?

TheNarrMaster ยท 1 points ยท Posted at 01:40:40 on November 22, 2015 ยท (Permalink)

You can always double a number and then add one to it, even or not, so there are as many odd numbers as there are numbers.

StevenXC ยท 0 points ยท Posted at 05:24:34 on November 21, 2015 ยท (Permalink)

Let a_n,b_n be distinct sequences of nonnegative integers less than 10. The sum of the series a_n/10n is less than the sum of the series b_n/10n if and only if a_N<b_N where N is the smallest n for which the sequences differ. For example, 3.62<3.9486 because 6<9.

VanMisanthrope ยท 2 points ยท Posted at 07:39:22 on November 21, 2015 ยท (Permalink)

3.59999... = 3.6 is an obvious counter example, (in decimal, of course). N=1 is the smallest natural for which a_n and b_n differ, a_1 = 5 < 6 = b_5, and thus 3.599.. < 3.6 (if this claim were true). And yet, we all know that this inequality is false.

duskhat ยท 1 points ยท Posted at 08:41:27 on November 21, 2015 ยท (Permalink)

I would think that seeing this is false is decently trivial

[deleted] ยท 0 points ยท Posted at 17:30:56 on November 21, 2015 ยท (Permalink)

[deleted]

[deleted] ยท 4 points ยท Posted at 19:03:53 on November 21, 2015 ยท (Permalink)

I don't see how that's intuitively obvious at all, unless you consider all statements of the form "the foo of the bar is the bar of the foo" intuitively true. As in "the brother of the mother is the mother of the brother".

[deleted] ยท 0 points ยท Posted at 23:05:11 on November 21, 2015 ยท (Permalink)

[deleted]

Ukrainian_Reaper ยท -1 points ยท Posted at 00:29:25 on November 22, 2015 ยท (Permalink)

Wouldn't 5 have infinitely many complex solutions?

Mayer-Vietoris ยท 1 points ยท Posted at 22:20:09 on November 22, 2015 ยท (Permalink)

Nope! A quadratic equation has at most 2 solutions. Similarly a polynomial of degree n has at most n solutions in the complex plane. There is always one solution (which is the content of the fundamental theorem of algebra), and if you count solutions with 'multiplicity' meaning that sometimes you count solutions more than once, you will get exactly n solutions. So with multiplicity a quadratic equation always has 2 solutions over the complex numbers.

SmArtilect ยท 1 points ยท Posted at 09:55:33 on November 23, 2015 ยท (Permalink)

Look up fundamental theorem of algebra.

every non-zero, single-variable, degree n polynomial with complex coefficients has, counted with multiplicity, exactly n roots

Huguet57 ยท 0 points ยท Posted at 11:09:32 on November 22, 2015 ยท (Permalink)

There are more whole numbers than pair numbers.

goiken ยท 0 points ยท Posted at 11:45:03 on November 22, 2015 ยท (Permalink)*
if true; then echo "yes"อพ fi

will not print โ€˜yesโ€™ in bash.

Synclicity ยท 1 points ยท Posted at 14:23:24 on November 22, 2015 ยท (Permalink)

Is that because a call to true returns exit success, aka 0, so you will need a negation or test before it?

goiken ยท 0 points ยท Posted at 14:46:07 on November 22, 2015 ยท (Permalink)

No

graboy ยท 1 points ยท Posted at 04:09:46 on November 23, 2015 ยท (Permalink)
graboy@computer ~ $ if true; then echo "yes"; fi
yes
graboy@computer ~ $

what

goiken ยท 1 points ยท Posted at 11:03:17 on November 23, 2015 ยท (Permalink)

You retyped it instead of copying. Hereโ€™s the trick.

graboy ยท 1 points ยท Posted at 20:28:33 on November 23, 2015 ยท (Permalink)

Aha!

bazir03 ยท -5 points ยท Posted at 11:40:18 on November 21, 2015 ยท (Permalink)

-1 = (Sqrt(-1))2 = Sqrt(-1) * Sqrt (-1) = Sqrt((-1) * (-1)) = Sqrt(1) = 1

-1 = 1

jachymb ยท -3 points ยท Posted at 14:40:21 on November 21, 2015 ยท (Permalink)

|Q| = |R \ Q|

daturkel ยท 6 points ยท Posted at 16:46:18 on November 21, 2015 ยท (Permalink)

I certainly hope this isn't commonly believed!

criangulien ยท 2 points ยท Posted at 02:29:23 on November 22, 2015 ยท (Permalink)

Care to explain?

Mayer-Vietoris ยท 2 points ยท Posted at 22:40:10 on November 22, 2015 ยท (Permalink)

Translation: There are the same number of rational numbers as there are irrational numbers.

When using the notion of cardinality to measure the size of an infinite set, there are some infinite sets which are bigger than others. In particular the rationals have the same cardinality (size) as the integers, and the irrationals have the same cardinality (size) as all real numbers.

spodek ยท -5 points ยท Posted at 13:34:48 on November 21, 2015 ยท (Permalink)

"Math is easy."

or maybe "Math is hard."

Probably one of those two statements is true.

DCarrier ยท 1 points ยท Posted at 19:25:32 on November 21, 2015 ยท (Permalink)

Buddhism has the same thing with "The mind is Buddha" and "The mind is not Buddha."